De Chuyen Car Nuoc

You might also like

Download as pdf or txt
Download as pdf or txt
You are on page 1of 129

PHẦN I: TỔNG HỢP ĐỀ THI CHUYÊN + HSG TRÊN CẢ NƯỚC

TEST 1
SECTION 1: LISTENING
Part 1: You will hear five short extracts in which people talk about habits they find difficult to
control. For questions 1-5, choose from the list (A–H) the habit each person has. Use the letters
only once. There are three extra letters which you do not need to use. [You will need to play this
recording twice.]
A. eating unhealthy food
B. doing too much exercise
C. buying unnecessary items
D. watching too much television
E. spending too much time online
F. oversleeping
G. working too hard
H. arriving late for everything
Speaker 1 ____________
Speaker 2 ____________
Speaker 3 ____________
Speaker 4 ____________
Speaker 5 ____________
Part 2: You will hear Leonie Steiner talking to an interviewer about her work as a music teacher
in a school. For questions 6–12, choose the best answer (A, B or C). [You will need to play this
recording twice.
6. Leonie first started learning the piano
A. with a relative.
B. at primary school.
C. with a private teacher.
7. Leonie started giving music lessons
A. for the pleasure of seeing others learn.
B. because she needed some extra money.
C. to see if she was suited to teaching.
8. Leonie most likes to teach students who
A. have a great natural talent at an early age.
B. need good teaching to develop their talent.
C. have previously been taught badly.
9. Leonie thinks that schools should
A. employ far more music teachers.
B. buy good musical instruments.
C. ensure that all their pupils pass music exams.
10. Leonie thinks the problem with singing in schools is that
A. many students are too embarrassed to sing.
B. few students want to learn how to sing.
C. singing is not often taught in them nowadays.
11. Leonie believes her success as a music teacher is a result of
A. choosing a particular age group of children to teach.
B. the training she received as a student teacher.
C. a natural ability to communicate with young people.
12. What decision did Leonie find difficult to make?
A. to turn down the offer of a job abroad
B. to refuse promotion in the school
C. to continue teaching when she felt tired
Part 3: For each question, write the correct answer in the gap. Write one or two words or a
number or a date or a time. You will hear a student giving a review of a new video game to his
class.
Video game review
The action takes place in: a (13) _______________
Players answer questions about: (14) _______________
Name of the most difficult level of the game: the (15) _______________
The best thing about the game: the (16) _______________
Maximum number of players: (17) _______________
Website for more details: www.(18) _______________.com

SECTION 3: LEXICO – GRAMMAR


I. Choose the word or phrase which best completes each of the following sentences. Write
your answers A, B, C, or D in the space provided.
Question 11 : Please help yourself to the food at the buffet. You__________ be starving after
your long journey.
A. could B. would C. might D. must
Question 12 : Luckily, I __________a new pair of sunglasses as I found mine at the bottom of a
bag.
A. needn’t have bought B. mustn’t have bought
C. didn’t need to buy D. can’t have bought
Question 13 : On the stairs __________ a small dark-haired girl.
A. was sitting B. were sitting C. sitting D. having sat
Question 14 __________ visit the British Museum while you’re in London, you mustn’t forget
to look at the famous Rosetta Stone.
A. In case you B. For fear that you
C. If you should D. When you might
Question 15 I‘m __________for Jeff at tomorrow’s meeting as he has to go to a conference in
Manchester.
A. copying in B. filling in C. picking up D. taking up
Question 16 I do take my hat __________ to the new director for having pulled the company
back from bankruptcy.
A. By B. on C. to D. off
Question 17 Since he’s left the army, he misses the excitement and danger and still wants to
live life in the fast __________
A. track B. path C. street D. lane
Question 18 My parents had to see the headmaster because of my bad __________in class.
A. manner B. behavior C. mood D. attitude
Question 19 The winner of the marathon had to __________ his breath before speaking to the
journalist.
A. run B. hold C. catch D. keep
Question 20 The poor grasshopper __________ he had spent the whole summer enjoying
himself singing and dancing.
A. confused B. confessed C. continued D. contented
Question 21 He’s so nervous. I wish for once he’d stop being such a __________
A. fish B. rabbit C. chicken D. mouse
Question 22 Before Tom headed into the forest, he made sure he had packed his bow
and__________
A. shotgun B. weapons C. backpack D. arrows
Question 23 Pesticides can be__________harmful to any creature living in the vicinity of where
they are used.
A. highly B. extremely C. completely D. absolutely
Question 24 Sylvia had been dieting for a long time, so I didn’t __________ to tell her that she
didn’t appear to have lost any weight
A. cross my heart B. have the heart
C. keep a straight face D. slip of the tongue
II. Choose the letter A, B, C, or D to indicate the sentence that best completes each of the
following exchanges.
Question 25 Frank and Mike are talking in the office
-Frank: “What are your thoughts on yesterday’s meeting?”
-Mike: “__________”
A. I think the president will be there.
B. I’m glad we covered the human resources’ issue.
C. We should bring up the workers’ concerns.
D. You are warmly welcome.
Question 26 Katie and Jimmy are in a restaurant.
-Katie: “ Waiter, we’re not happy with the taste of the soup.”
- Jimmy: “__________, Katie. There’s nothing wrong with the soup.”
A. Speak up B. Talk about it
C. Talk nonsense D. Speak for yourself
III. Choose the letter A, B, C or D to indicate the word(s) CLOSET in meaning to the
underlined word(s) in each of the following sentences.
Question 27 I can’t stomach those silly people who always go around with their mobile phones
glued to their ears.
A. protest B. accept C. argue D. criticise
Question 28 I simply don’t believe the scheme will ever succeed, it’s just pie in the sky
A. likely to swing B. unlikely to happen
C. possible to fly D. impossible to devise
IV. Choose the letter A, B, C, or D to indicate the word(s) OPPOSITE in meaning to the
underlined word(s) in each of the following sentences.
Question 29 After many months of grueling work and painful injuries to her shoulder and back,
Susan realized that her dream of swimming the English Channel was unattainable.
A. impossible B. unachievable C. realistic D. confused
Question 30 Most of Kim’s teachers wanted her to use her noodle more when it came to exam
time.
A. act up B. get on C. try out D. give up
V. Identify the underlined word or phrase that must be changed to make the sentence correct
by writing your answer A, B, C, or D in the space provided.
Question 31 If watering and taking care of carefully, the little pot plant will grow
A B C
to become a beautiful tree in some months.
D
Question 32 Never in the history of humanity has there been more people living
AB C
on this relatively small planet.
D
Question 33 That guy, who was considerable and turned down the stereo when
A B C
we asked him to, created a good impression on me.
D
Question 34 The most visible remind of the close relationship between the
A B
United States and France is the famous Statue of Liberty, which stands in New York harbour.
C D
Question 35 The final of the show was a fantastic musical number where the
A B C
entire cast danced to a live band.
D
VI. Give the correct forms of the verbs in brackets. Write your answers in the space provided.
36. (leave) __________ out in the rain for months, the bicycle was covered in rust. 37. The
suggestion was that all first-year university students (give) __________a room on campus.
38. Angela was upset (omit) __________from the short list of candidates for the science prize.
39. The new teacher allowed (write) __________ the exam papers in pencil. 40. Why didn’t you
tell me that you were coming home with David? Then I (not drive) __________ all the way into
town to pick you up!
Write your answers here:
36.__________ 37. __________ 38. __________ 39. __________ 40. __________ \
VII. Fill in each blank with the correct form of the word in brackets. Write your answers in
the space provided.
41. The interviewee wasn’t very __________ to questioning, and failed to get the job she
applied for. (RESPOND)
42. People who sleep on their backs with their arms stretched out typically awake feeling
__________ and eager for the day ahead. (VITAL)
43. Having done a great amount of research into the mobile phone market, Robert insists that the
most expensive phones are not __________ the best. (NEED)
44. Your __________ towards my new job bothers me. (DIFFER)
45. The change in Oliver’s attitude towards pay cuts was __________. As our superior, he had
always listened to and supported our demands. (EXPLAIN)

SECTION 4: READING
Read the following passage and fill each numbered blank with ONE suitable word. Write your
answers in the space provided.
MALNUTRITION GOALS IN AFRICA “VERY FAR AWAY”
A new report shows that no country in Africa will meet the goals set to end childhood
malnutrition by the year 2030. That target was set by the United Nations in 2015 (1) __________
a Sustainable Development Goal. The UN adopted a set (2) __________ goals, "to end poverty,
protect the planet and ensure prosperity for all as part of a new sustainable development agenda".
The new report is published in the journal Nature. It identifies poor child nutrition and low levels
of education across 51 African countries. These were key factors in countries battling to (3)
__________ children with sufficient food. Researcher Simon Hay said the goal of ending
childhood malnutrition was always an "aspirational" target. He said: "This aspiration is very, very
far away."
There was some good news in the report. It highlighted the fact that many African nations,
(4) __________ Ghana and Nigeria, have shown signs of improvement in childhood development
(5) __________since the year 2000. (6) __________, it is a different story for countries like Chad,
Central African Republic and Eritrea. The report indicates that malnutrition (7) __________
remained "persistently high" in 14 countries between Senegal in the west and Somalia in the east.
Many of these countries have (8) __________ war, famine and mass migration, all of (9)
__________have put massive strains on health and agriculture. One researcher said the
considerable investment was needed (10) __________ health and infrastructure in order to
address "serious inequalities".
Read the following passage and choose the word or phrase ( A, B, C, or D) that best fits the
blank in the following passage. Write your answers in the space provided Why do we smile?
Every human being is born with the ability to smile. We know this because even children
who are (1) __________ to see can still grin. A smile is a natural reaction (2) __________ a
positive moment, like a friendly face or a clever joke. But this isn’t the first reason we smile. Most
babies first smile when they (3) __________ the age of six to eight weeks. It’s usually an accident:
an expression (4) __________ when exercising their facial muscles (or passing wind!). But once
they realize a smile (5) __________ them a lot of attention- huge smiles and happy noises in
return- they learn to try it again. And it doesn’t (6) __________ too much effort either we need
43 muscles to frown but only 17 smiles.
But not every creature can smile. Although your pet dog may sometimes look like it’s smiling
back at you, it’s not. Animals do (7) __________some of the same emotions as us, but they don’t
smile. A dog’s cheerful face is more (8) __________to be a sign of confusion, while dolphins (9)
__________ the muscles for facial expressions (their mouths are permanently turned upwards
which makes them appear to be smiling). Animals express their joy through other movements,
like a (10) __________ tail or a backward flip. Some, such as chimpanzees and rats, can even
laugh.
Question 1 : A. incapable B. useless C. unable D. helpless
Question 2 : A. of B. for C. with D. to
Question 3 : A. reach B. make C. achieve D. accomplish
Question 4 : A. done B. given C. made D. pushed
Question 5 : A. gets B. pays C. draws D. points
Question 6 : A. use B. cost C. take D. make
Question 7 : A. sense B. experiment C. release D. experience
Question 8 : A. possible B. probable C. likely D. kindly
Question 9 : A. lose B. lack C. miss D. need
Question 10 : A. wagging B. revolving C. shifting D. stirring
Read the following passage and choose the correct answer ( A, B, C, or D) for each of the
questions below. Write your answers in the space provided.
The Mystery of the Fortune Cookie
To many people, particularly in America, every good Chinese meal should end with a
fortune cookie. So would you believe that one place you won’t see a fortune cookie is China?
These cookies have a long and mysterious history―one that doesn’t begin in China. According
to researcher Yasuko Nakamachi, fortune cookies actually originated in Japan! Ms. Nakamachi
first saw Japanese fortune cookies at a bakery while visiting a popular temple outside Kyoto in
the 1990s. However, the baker was folding a paper fortune into a fold on the outside of the
cookie, not the inside, like the fortune cookies we are used to. Ms. Nakamachi was very curious
about this, and decided to do her own research. After spending six years going through
thousands of old documents and drawings, and interviewing bakers around the country, she
realized that fortune cookies used to be very popular in Japan. The reason that Takeshi
Matsuhisa, the baker, puts the fortune on the outside of the cookie is to make sure that people
don’t accidentally eat the paper
Ms. Nakamachi found a drawing that went as far back as 1878, showing a Japanese man
making the same kind of cookies as Matsuhisa’s bakery. This is interesting because a number of
people claimed to have invented fortune cookies in California in the 1920s.
If these cookies are a Japanese invention, then why are they served in American Chinese
restaurants? After interviewing many Japanese and Chinese American families, Ms. Nakamachi
suggested that it’s likely that Japanese people first started serving fortune cookies in their
restaurants when they moved to the United States. Then Chinese restaurant owners borrowed the
idea and began making their own fortune cookies, beginning the now traditional practice of
serving fortune cookies at the end of each meal.
Today, about three billion of these cookies are made annually in the U.S., and are served in
restaurants all over the world. Although fortune cookies might not be a traditional snack in China,
they have become one for people in many other countries.
Question 11 This passage is mainly about__________?
A. how fortune cookies became popular
B. how and where fortune cookies began
C. why Chinese people make fortune cookies
D. how fortune cookies can predict the future
Question 12 What is important about Ms. Nakamachi’s discovery of the 1878 drawing?
A. It shows that fortune cookies were popular in Japan and California.
B. It proves that Matsuhisa’s bakery was the first to make fortune cookies.
C. It shows that people wanted to draw and write about fortune cookies.
D. It proves that fortune cookies were first made in Japan, not California
Question 13 The word their in the passage refer to __________.
A. American Chinese restaurants
B. Chinese American families
C. Japanese people
D. fortune cookies
Question 14 The fortune cookie in the passage is best described as__________.
A. A biscuit with the frosting on it
B. a biscuit made from many kinds of ingredients
C. a biscuit containing a message, usually about someone’s future
D. the most popular kind of biscuit all over the world
Question 15 Which of these statements is NOT true?
A. Fortune cookies are popular in the U.S.
B. In Japan, the fortunes are put on the outside of the cookie.
C. Fortune cookies were brought to the U.S by the Chinese.
D. American cookie makers made the fortune cookie popular.

SECTION 5: WRITING
I. Finish each of the following sentences in such a way that it has a similar meaning to the
sentence printed before.
Question 1 The value of dollars has fallen considerably in the past week.
→ There __________________________________________________________.
Question 2 It was to be another ten years before my cousin returned to his hometown.
→ Not until ________________________________________________________________.
Question 3 Our science teacher never remembers to correct our homework!
→ Our science teacher is _____________________________________________.
Question 4 San made a good impression on the judges, and she earned a place on the talent
show.
→ If _________________________________________________________________.
Question 5 The conductor who had devoted her life to studying Bach was honoured at the
ceremony.
→ The conductor whose ___________________________________________.
II. Complete the second sentence so that it has a similar meaning to the first sentence using
the word in bold given brackets. DO NOT CHANGE THE WORD GIVEN.
Question 6 The drunken man was driving so fast that there was no way he was going to stop.
(SUCH)
→ The drunken man _______________________________________________.
Question 7 I’ve considered the advantages and disadvantages, and I’ve decided not to go.
(WEIGHED)
→ I’ve ___________________________________________________________. Question 8
The young actress was very nervous before the audition. (BUTTERFLIES)
→ The young actress had ________________________________________________________.
Question 9 I didn’t particularly like either of the two laptops that the assistant showed me.
(NEITHER)
→ The assistant showed me ________________________________________.

Question 10 My sister was against my taking her motorbike to school yesterday. (SOONER)
→ My sister ____________________________________________________________
III. Cycling to different parts of the country is becoming a trend among many people. Write a
paragraph (from 120 to 150 words) about the benefits of a tour by bicycle.
TEST 2
PHẦN LISTENING
Part 1: For each question, choose the correct answer.
1. You will hear two friends talking about a film they’ve just seen. Why didn’t the boy enjoy the
film?
A. It was very frightening.
B. It lasted too long.
C. It had terrible acting.
2. You will hear two friends talking about some biology homework. The girl suggests that the
boy should
A. ask his teacher for help.
B. get information from the internet.
C. look in the biology textbook.
3. You will hear two friends talking about an interview with a singer they’ve seen on TV. They
agree that
A. the singer’s answers were interesting.
B. the interviewer was quite rude.
C. the questions were confusing.
4. You will hear a girl telling her friend about a diving trip. How did the girl feel about it?
A. sure she will go again
B. glad she went with a relative
C. pleased with her diving skills
5. You will hear a girl talking to a friend about basketball. The girl is trying to
A. explain the rules of the game.
B. describe a game she took part in.
C. encourage the boy to start playing.
6. You will hear a boy talking about a trip to a city with his family. Why did the boy’s family
get lost?
A. They couldn’t understand their map.
B. Someone gave them the wrong directions.
C. The guidebook contained incorrect information.
Part 2: You will hear a man called Markus Fischer talking about mooncake, a traditional Asian
bakery product. For questions 7–16, complete the sentences with a word or short phrase.
Mooncake
Markus was staying in (7) __________ when he first tried mooncake.
Markus’s favourite kind of mooncake has a filling made of (8)
__________. Markus says that he likes to have (9) __________ with
mooncake.
Mooncake is popular during the Moon Festival, which last year was held in (10) __________.
The mooncake Marcus was given during the festival had the shape of a (11) __________ on the
top.
Markus was surprised to learn that mooncake is rarely made (12) __________ in China. The
people Markus was staying with received mooncake from their (13) __________. His host
family gave Markus a mooncake that had a slightly (14) __________ flavour. Markus was told
that, many years ago, people used mooncakes to send (15) __________ to each other.
After people had read what was written on the mooncake, they (16) __________ it. Part 3:
Complete the notes below. Write ONE WORD AND/OR A NUMBER for each answer.
Children’s Engineering Workshops
Tiny Engineers (ages 4-5)
Activities
- Create a cover for an (17) __________ so they can drop it from a height without breaking
it. - Take part in a competition to build the tallest (18) __________
- Make a (19) __________powered by a balloon.
Junior Engineers (ages 6-8)
Activities:
- Build model cars, trucks and (20) __________ and learn how to program them so they can
move.
- Take part in a competition to build the longest (21) __________ using card and wood.
- Create a short (22) __________ with special software.
- Build, (23) __________ and program a humanoid robot.
Cost for a five-week block: £50
Held on (24) __________ from 10 am to 11 am
Location
Building 10A, (25) __________ Industrial Estate, Grasford
Plenty of (26) __________ is available.
Choose the correct answer to complete each of the following questions from 3 to 17.
Question 3 Look! There's no point in trying to overturn the decision. It's all __________.
A. cut and dried B. head and shoulder
C. tooth and nail D. chop and change
Question 4 Buy me a newspaper on your way back, __________?
A. don't you B. have you C. do you D. will you
Question 5 Mary gave me a __________ box on my last birthday.
A. Jewelry metal small square B. metal small jewelry square
C. small square metal jewelry D. square small jewelry metal
Question 6 I was disappointed when I saw the film. It was a real __________.
A. lay-by B. setback C. let-down D. knockout
Question 7 In the early 20th century, physicians discovered that blood transfusions often failed
because the blood type of the recipient was not compatible __________ the donor.
A. to that of B. with that of C. to those of D. with those of
Question 8 She could sleep well __________ the loud music the neighbour played last night.
A. although B. despite C. regardless D. but
Question 9 Our prices are __________ of all flights and accommodation, and represent
excellent value.
A. self-catering B. economical C. inclusive D. packed
Question 10 If the level of VAT is __________ this year, small businesses will be affected.
A. raised B. arisen C. risen D. raising
Question 11 They managed to __________ doing the work by pretending to be busy.
A. get out of B. get away with C. make up to D. make off with
Question 12 It was extremely extravagant of us to stay in a luxurious hotel, but we wanted to
__________ ourselves.
A. pride B. pledge C. justify D. treat
Question 13 By appearing in the soap powder commercials, she became a __________ name.
A. housewife B. housekeeper C. house D. household
Question 14 I wish I hadn't __________ him for his brother.
A. thought B. considered C. confused D. mistaken
Question 15 Make sure that the memo is sent to __________.
A. all the involved people B. all who are involved people
C. all the people involved D. all of people involved
Question 16 The guest tasted the dessert __________.
A. appreciatively B. appreciative C. appreciated D. appreciation
Question 17 It's true that older people are a bit __________ when it comes to things like
technology, but on the whole, I think they're probably more open-minded than they used to be.
A. out of work B. out of touch C. out of control D. out of order
Choose the word(s) CLOSEST in meaning to the underlined word(s) in each of the following
questions from 21 to 22.
Question 21 Did she get the better of you in the argument as to whether milk is good for our
health?
A. gain an advantage over B. gain a disadvantage over
C. try to beat D. try to be better than
Question 22 Once I realized I was not cut out for the job, I decided to hand in my notice.
A. was laid off by the company
B. did not have the necessary qualities and abilities
C. did not take the necessary action to resolve
D. lacked relevant working experience
Read the text and decide which answer best fits each gap of the following questions from 23 to
32.
Why people laugh
Sunday May 4th will be World Laughter Day. Dr Madan Kataria, who introduced this annual
event, says we need more laughter in our lives to (0) combat the global rise of stress and
loneliness. But surely that strange sound that we make periodically can't be the (23) __________
to such problems.
If an alien (24) __________ to land on our planet and a take stroll among a crowd of
earthlings, it would hear a lot of 'ha-ha' noises. It might wonder what purpose this strange habit
(25) __________. If we ask ourselves what (26) __________ a good laugh, the obvious answer is
that it is a response to something funny. But one scientist, Rober Provine, says humour has
surprisingly (27) __________ to do with that. Instead, it lies at the (28) __________ of such issues
as the perception of self and the evolution of language and social behaviour.
Rober Provine realised that you cannot capture (29) __________ laughter in the lab because
as soon as you place it under scrutiny, it vanishes. So, instead, he gathered (30) __________ by
hanging around groups of people, noting when they laughed.
He collected 1,200 laugh episodes - an episode being defined as the comment immediately
(31) __________ we laughter and the laughter itself. His analysis of this data revealed some
important facts about laughter. “It's a message we send to other people - it virtually reveals when
we're by ourselves," he says. "And it's a choice. Ask someone to laugh and they'll either try to fake
a laugh or say they can't do it on (32) __________”.
Question 23 A. answer B. means C. response D. question
Question 24 A. should B. were C. had D. are
Question 25 A. served B. asked C. caused D. made
Question 26 A. results B. concludes C. leads D. prompts
Question 27 A. few B. much C. little D. many
Question 28 A. head B. root C. back D. stem
Question 29 A. authentic B. current C. artificial D. contemporary
Question 30 A. noise B. data C. fun D. news
Question 31 A. defining B. following C. paralleling D. preceding
Question 32 A. intention B. support C. command D. accident
Choose the word(s) OPPOSITE in meaning to the underlined word(s) in each of the following
questions from 33 to 34.
Question 33 A celebrated psychologist who expanded our knowledge of how children think
and develop was a Swiss named Jean Piaget.
A. obscure B. renowned C. prominent D. conservative
Question 34 Downpours had drenched Hanoi all day, but that did not dampen the enthusiasm
of red-clad spectators.
A. irrigate B. flood C. replenish D. kindle
Read an article about the effects of digital media on people's minds and choose the best
answer to the following questions from 35 to 40.
Is the Internet making us stupid?
In an article in Science, Patricia Greenfield, a developmental psychologist who runs UCLA's
Children's Digital Media Center, reviewed dozens of studies on how different media technologies
influence our cognitive abilities. Some of the studies indicated that certain computer tasks, like
playing video games, increase the speed at which people can shift their focus among icons and
other images on screens. Other studies, however, found that such rapid shifts in focus, even if
performed adeptly, result in less rigorous and more ‘automatic’ thinking.
In one experiment at an American university, half a class of students was allowed to use
internet-connected laptops during a lecture, while the other half had to keep their computers shut.
Those who browsed the web performed much worse on a subsequent test of how well they retained
the lecture's content. Earlier experiments revealed that as the number of links in an online
document goes up, reading comprehension falls, and as more types of information are placed on
a screen, we remember less of what we see.
Greenfield concluded that 'every medium develops some cognitive skills at the expense of
others'. Our growing use of screen-based media, she said, has strengthened visual-spatial
intelligence, which can strengthen the ability to do jobs that involve keeping track of lots of
rapidly changing signals, like piloting a plane or monitoring a patient during surgery. However,
that has been accompanied by 'new weakness in higher-order cognitive processes', including
'abstract vocabulary, mindfulness, reflection, inductive problem solving, critical thinking and
imagination'. We're becoming, in a word, shallower.
Studies of our behaviour online support this conclusion. German researchers found that web
browsers usually spend less than ten seconds looking at a page. Even people doing academic
research online tend to ‘bounce' rapidly between documents, rarely reading more than a page or
two, according to University College London study. Such mental juggling takes a big toll. In a
recent experiment at Stanford University, researchers gave various cognitive tests to 49 people
who do a lot of media multitasking and 52 people who multitask much less frequently. The heavy
multitaskers performed poorly on all the tests. They were more easily distracted, had less control
over their attention, and were much less able to distinguish important information from trivia. The
researchers were surprised by the results. They expected the intensive multitaskers to have gained
some mental advantages. That wasn't the case, though. In fact, the multitaskers weren't even good
at multitasking. 'Everything distracts them,' said Clifford Nass, one of the researchers.
It would be one thing if the ill effects went away as soon as we turned off our computers and
mobile, but they don't. The cellular structure of the human brain, scientists have discovered, adapts
readily to the tools we use to find, store and share information. By changing our habits of mind,
each new technology strengthens certain neural pathways and weakens others. The alterations
shape the way we think even when we're not using the technology. The pioneering neuroscientist
Michael Merzenich believes our brains are being 'massively remodelled' by our ever-intensifying
use of the web and related media. In 2009, he said that he was profoundly worried about the
cognitive consequences of the constant distractions and interruptions the internet bombards us
with. The long-term effect on the quality of our intellectual lives, he said could be ‘deadly'.
Not all distractions are bad. As most of us know, if we concentrate too intensively on a tough
problem, we can get stuck in a mental rut. However, if we let the problem sit unattended for a
time, we often return to it with a fresh perspective and a burst of creativity. Research by Dutch
psychologist Ap Dijksterhuis indicates that such breaks in our attention give our unconscious mind
time to grapple with a problem, bringing to bear information and cognitive processes unavailable
to conscious deliberation. We usually make better decisions, his experiments reveal, if we shift
our attention away from a mental challenge for a time.
But Dijksterhuis's work also shows that our unconscious thought processes don't engage with
a problem until we've clearly and consciously defined what the problem is. If we don't have a
particular goal in mind, he writes, 'unconscious thought does not occur'. The constant
distractedness that the Net encourages is very different from the kind of temporary, purposeful
diversion of our mind that refreshes our thinking. What we seem to be sacrificing in our surfing
and searching is our capacity to engage in the quieter, attentive modes of thought that underpin
contemplation, reflection and introspection.

Question 35 What do we learn about Patricia Greenfield's research in the first paragraph?
A. It did not produce consistent patterns in connection with computer use.
B. It focused on problems resulting from use of media technologies.
C. It involved collating the results of work done by other people.
D. It highlighted differences between people when using computers.
Question 36 Two of the experiments mentioned in the second paragraph
concerned__________.
A. the amount of attention people pay to what they see on computers.
B. the connection between computer use and memory.
C. changes that happen if people's computer use increases.
D. the use and non-use of computers for studying.
Question 37 One of Greenfield's conclusions was that__________.
A. too much emphasis has been placed on the benefits of computer use.
B. people do not care about the effects of computer use on their minds.
C. computer use has reduced a large number of mental abilities.
D. certain claims about the advantages of computer use are false.
Question 38 One of the pieces of research mentioned in the fourth paragraph indicated
that__________.
A. people read online material less carefully than other material.
B. beliefs about the effectiveness of multitasking are false.
C. some people are better at multitasking than others.
D. 'mental juggling' increases the mental abilities of only a few people.
Question 39 What is the writer's purpose in the fifth paragraph?
A. to present opposing views on the consequences of use of new media technology
B. to advise on how to avoid the bad effects of new media technology
C. to summarise the findings of the previously-mentioned research
D. to warn about the damage done by use of new media technology
Question 40 The writer mentions Ap Dijksterhuis's research in order to make the point
that__________.
A. problem-solving can involve very complex mental processes.
B. not all research supports beliefs about the dangers of computer use.
C. the mind functions in ways that computers cannot.
D. uninterrupted concentration on something is not always a good thing

PHẦN II: TỰ LUẬN

Read the article about teenagers. Five paragraphs have been removed. Choose from the
paragraphs (A-F) the ones which best fit each gap of the following questions from 41 to 45.
There is one extra which you do not need to use.
Rebels with a Cause
Here's to the crazy ones. The misfits. The rebels. The troublemakers. The round pegs in the
square holes. The ones who see things differently. They're not fond of rules. You can quote them,
disagree with them, glorify or vilify them. About the only thing you can't do is to ignore them.
And while some may see them as crazy ones, we see genius. Because the people, who are crazy
enough to think they can change the world, are the ones who do."
41. __________
Typically, there is not a lot of praise to be heard for teenagers. But teenage rebellion and
risk-taking is a natural result of the development of the brain, and a necessary step to reach
adulthood. Our teenage years are like no other period in our lives, and we should view this stage
as a unique opportunity rather than only a time of drama and danger.
42. __________
For example, one of the best things about being a teen is having an amazing ability to learn,
and this is a positive side effect of being willing to take risks. Teens are less afraid of failure, and
one of the biggest limitations people face in life is the tendency not to try something new because
they might fail. Teens, however, are wide open to trying new things.
43. __________
This sensitivity can make teens highly motivated at tasks that interest them. And while the
task that interests them might be playing video games, it could also be playing the guitar, writing
poetry, or even learning about physics. It is the age when obsessive practice is the easiest it will
ever be, and that practice is one aspect of genius.
44. __________
Adults might view this as naivety, after all, making changes happen in society is extremely
difficult, could be a waste of time, and would probably upset people. However, we can be sure
that if no one bothers to try, change is impossible. And while change might be scary, and risky, it
is not always bad. In fact, sometimes it is essential.
45. __________
So perhaps it is time to start celebrating teenage rebellion rather than dreading it. Of course,
parents and teachers need to set safe limits, but they should also feel proud of teens' newfound
opinions, positive risk-taking and creativity. After all, they will need those skills in the future in
order to build a better world.
A. In addition, teens' brains are very sensitive to rewards, and this also helps them learn.
Succeeding at a task gives teens a powerful incentive for repeating, and remembering, rewarding
behaviours.
B. Essentially, what we can learn from this isn't that teens necessarily have better memories
than adults, but rather the way in which they remember is different. They are able to connect
different things and build a richer understanding of the world.
C. And, although not essential, even teenage rebellion against family authority has a good
side. Researchers have found that teens who argued with their parents were more likely to resist
peer pressure to drink. It turns out that the family home is a safe place for teens to practise
standing up for what they believe in.
D. This quote could easily be used to describe teens that rebel against authority, are
emotional and make impulsive decisions. But, like the crazy ones, they are also creative,
committed to doing things their own way, and are often agents of social change.
E. It's true that it's a dangerous age. For example, teen drivers are three times more likely
than adults to be involved in a fatal accident. And while this issue is real, let's not overlook the
fact that not all risk-taking is negative.
F. Another positive aspect of teenage risk-taking is their belief that they can change society.
They are more likely than adults to speak out for what they think is right or find innovative
solutions to problems.
Read the following passage and answer the following questions from 46 to 55.
YOUTH WORKS
As the pace of today's working life blurs the line between personal time and work time, so it
increasingly mixes personal lifestyle and work style. And as companies concentrate on attracting
and keeping a younger workforce for its technical skills and enthusiasm for change, office culture
is becoming an extension of youth culture. This may be no bad thing. Along with the company
games room come things that matter deeply to young people: opportunity, responsibility, respect.
For most of human history the middle-aged have ruled. With years came wisdom, experience,
connections and influence. Rarely did they change jobs, years of loyal service counted most.
However, in the future, older workers will not disappear, or even reduce in numbers, but they will
have to share power with fresh-faced youths.
There have been a number of reasons for this change; the most dramatic of these is
technology. Children have always been more expert than their parents at something, but usually a
game or a fashion, not the century's most important business tool. The Internet has triggered the
first industrial revolution in history to be led by the young. This is the age group that created
Netscape, the first commercial web browser; Napster, the music-sharing technology that shocked
the music industry; Yahoo! and many of the other web giants. Though there have been youth
revolutions before, none of them made the leap from teen bedroom to boardroom the way the
Internet has. Throughout the twentieth century, had a young person wanted to enter corporate
America they needed to leave their youth behind. They got a haircut, and probably a suit or at
least a tie. Now the same hair, same clothes, even nearly the same hours apply to office and home.
Had it not been for the Internet, this change could not have happened. However, it did not
happen because of the Internet only, the corporate restructuring of the 1980s and 90s broke down
traditional hierarchies. In many companies, rigid seniority-based hierarchies have given way to
hierarchies based on merit. No longer are the abilities to navigate internal bureaucracies and please
your superiors the most valued skills. Today's employees are free agents who stay with companies
only as long as they feel challenged and rewarded; moving from job to job is now a sign of
ambition and initiative. Today's young people are valued as workers for different reasons than
their predecessors: they welcome change; they think differently; they are independent; they are
entrepreneurial; they want opportunity more than money and security and finally, they demand
respect.
This revolution is not just about the young. Youth itself is being redefined. Increasingly, 35-
yearolds listen to the same music as 20-year-olds, dress like them and even look almost like them.
Never before has there been a time when there was so little difference between age groups.
Imagine a society converging on an age somewhere between 20 and 30, and you have a fair picture
of New York or San Francisco now, with other American cities not far behind.
The rise of the young is a good thing, not least because it gives people at their most creative
stage in life more opportunity to put their ideas and energy into practice. But will there be a
takeover by the young? A good place to look for an answer is Microsoft. Microsoft's most
important employees are not its managers, but individual programmers. They have great
independence in choosing how to do their job. By and large, the managers' task is not to tell the
programmers what to do, but to clear obstacles from the path they choose. Microsoft workers are
valued most for their ability to think for themselves, they are trusted to find their own solutions to
business problems. Managers hold back, knowing that the more specific their order, the more it is
likely to undermine their employees' ability to find creative solutions. So they concentrate on the
diplomatic tasks that most of the independent young programmers are not much good at: co-
ordinating with other teams, resolving conflicts, motivating people and ensuring that everybody
is happy. Microsoft starts to look like a model for the workplace of the future: programmers tend
to bem their twenties and early thirties, whereas the managers are about a decade older. Many of
the managers are former programmers who reached a point where they no longer wanted to sleep
under their desk. The effect of all this is that youth and youth qualities apparently dominate, but
the experience and maturity of older employees is put to good use too.
Decide whether the statements reflect the claims of the writer.
Write
YES if the statement reflects the claims of the writer.
NO if the statement contradicts the claims of the writer.
NOT GIVEN if it is impossible to say what the writer thinks about this.
46. The number of older workers in companies will decline.
47. The Internet is the most important development since the industrial revolution.
48. In many companies, the ability to make the superiors pleased is not one of the most valued
skills any longer.
49. Microsoft's most important employees are individual programmers.
Complete the summary below. Use NO MORE THAN THREE words or a number.
In today's workplace (50) __________ and work are becoming mixed and older workers are
losing power in their companies. The most important reason for this is (51) __________ which
has allowed fresh-faced youths to enter the workplace and make changes. A second reason was
the changes made to company (52) __________ in the 80s and 90s which emphasized (53)
__________ over seniority. The final reason is that values have changed. Today's workers want
opportunity more than (54) __________. Another effect is that older people are behaving like
younger people with society's average age between 20 and 30 in some US cities. At Microsoft
the manager's role is not to give workers orders but to (55) __________ from their way and help
them discover solutions to business problems.
Read the text below and use ONE word which best fits each gap of the following questions
from 56 to 65.
Britain has gone mad for bananas. Over the past 12 months Britons have consumed an
unprecedented 3.5 billion pieces of the tropical fruit, forcing the (56) __________ apple into
second place.
The nation's banana boom is one of the most remarkable nutritional trends of recent years, a
guide not only to the growing (57) __________ consciousness of the British people but also to the
country's economic health. (58) __________ is amazing is that bananas were virtually unheard
of during the 19th century and even up until the end of the 1920s (59)__________ anyone in
Britain had tasted or (60) __________ seen them. Early attempts to introduce them to northern
countries had met with failure because by the time they had been shipped to Britain, they had
rotted (61) __________ recognition. However, thanks to the development of refrigerated
shipping, all this changed. Refrigerated shipping meant that then, as now, bunches of imported
bananas could arrive in good condition at (62) __________ houses in dockyards where they were
stored. The first commercial refrigerated shipment arrived 100 years ago, triggering enthusiasm
from (63) __________ Britons have never looked back.
"The banana has everything going for it”, says Jeanette Scott of the Banana Group marketing
organization. "It's easy to open it is packed (64) __________ energy and vitamins and is low in
calories. It is also a first-class cure for upset stomachs and it stabilises blood (65) __________, so
its popularity should not be seen as that surprising."
Read the text below. Use the word given in capitals to form a word that fits in the gap of the
following questions from 66 to 75.
The American painter George Wesley Bellows (1882-1925) was the only son of an elderly
couple who (66. EXAMPLE) __________ the Midwestern values of honest business practice and
strict morality. From earliest childhood he seemed determined to become an artist. Before
graduating from Ohio State University, and in the face of stiff parental (67. OPPOSE)
__________ he moved to New York to study art. There he was strongly influenced by "The Eight",
or American Ashcan School. For the (68. REMAIN) __________ of his life, his work was
characterized by realist subject matter, (69. LIE) __________ which was a traditional approach
to composition. He was also fascinated by the various systems of colour (70. RELATE)
__________ that painters were using at the time, and studied them in detail. The truly outstanding
work that he produced in these early days (71. SHADOW) __________ and contributed to much
of his later painting.
Despite his identification with common, even low-life themes, he was elected an associate of
the (72. PRESTIGE) __________ National Academy at the exceptionally early age of 27. One
of the reasons the Academy honoured Bellows, while (73. HOLD) __________ approval from
many of the other members of “The Eight", was the fact that there were unmistakable references
to the old masters in Bellows' work. He was one of the few artists who (74. INSTINCT)
__________ combined a modern verve and energy with an appreciation of (75. ART)
__________ tradition, and his almost universal appeal was therefore not surprising.
Finish each of the following sentences from 76 to 80 in such a way that it means exactly the
same as the sentence printed before it.
Question 76 It was such a difficult task that expert assistance was required.
→ So ___________________________________________________________________.
Question 77 He is unlikely to be promoted.
→ There
_______________________________________________________________________.
Question 78 Susan tries hard, but she doesn't get anywhere.

However______________________________________________________________________
. Question 79 They believe that the manager absconded with the company's pension fund
money.
→ The manager ______________________________________________________________.
Question 80 “Please, please don't tell anyone you've seen me!" the boy said to me.
→ The boy begged ____________________________________________________________.
Complete the second sentences from 81 to 85 so that it has a similar meaning to the first one,
using the word given. Do not change the word given. You must use between THREE and SIX
words, including the word given.
Question 81 She is certainly not a good cook. (MEANS)
→ She ____________________________________________________ good cook.
Question 82 I could tell by the tone of his voice how serious the situation was. (HOME)
→ The tone of his voice ___________________________ how serious the situation was.
Question 83 [ No one stands a chance of beating him in this year's championship.
(FOREGONE)
→ It's _____________________________________________ that he will win this year's
championship.
Question 84 We ask travellers not to use their mobile phones when they pass through security.
(REFRAIN)
→ Travellers _________________________________ their mobile phones when they pass
through security.
Question 85 [The festival was so well organized that everything went smoothly.
(CLOCKWORK)
→ Everything at the festival _________________________ thanks to the excellent
organization.

Recently young people are said to be "Welcome Generation" as they are willing to face any
difficulties Write a paragraph of about 140 words about how people of your age in your
country deal with challenges in their life.
TEST 3
PHẦN 1: LUYỆN NGHE

Part 1: For each question, write the correct answer in the gap. Write one or two words or a
number or a date or a time. You will hear a student called Matilde giving a presentation to her
class about a science museum she visited.
Science museum visit
Matilde’s favourite room contained displays about (1) __________.
The (2) __________ exhibition is currently closed.
Matilde made a (3) __________ at the museum.
The shop had a really good range of (4) __________.
The guided tour lasts for (5) __________ minutes.
Visitors must use the entrance on (6) __________ Road.
Part 2: You will hear five short extracts in which people are talking about how to give good
presentations. For questions 7–11, choose from the list (A–H) what advice each person gives.
Use the letters only once. There are three extra letters which you do not need to use.
A. Keep your presentation short.
B. Remember to repeat your main point.
C. Support your presentation with visuals.
D. Add some humour.
E. Practise giving your presentation.
F. Try to relax during your presentation.
G. Don’t try to memorise every word.
H. Find out about your audience.
Question 7 Speaker 1 __________
Question 8 Speaker 2 __________
Question 9 Speaker 3__________
Question 10 Speaker 4 __________
Question 11 Speaker 5 __________
Part 3:
Questions 12 and 13
Which TWO points does Adam make about his experiment on artificial sweeteners?
A. The results were what he had predicted.
B. The experiment was simple to set up
C. A large sample of people was tested.
D. The subjects were unaware of what they were drinking.
E. The test was repeated several times for each person.
Questions 14 and 15
Choose TWO letters, A-E.
Which TWO problems did Rosie have when measuring the fat content of nuts?
A. She used the wrong sort of nuts.
B. She used an unsuitable chemical.
C. She did not grind the nuts finely enough.
D. The information on the nut package was incorrect.
E. The weighing scales may have been unsuitable.
Questions 16-21. Choose the correct letter, A, B or C.
16. Adam suggests that restaurants could reduce obesity if their menus
A. offered fewer options.
B. had more low-calorie foods.
C. were organised in a particular way.
17. The students agree that food manufacturers deliberately
A. make calorie counts hard to understand.
B. fail to provide accurate calorie counts.
C. use ineffective methods to reduce calories.
18. What does Rosie say about levels of exercise in England?
A. The amount recommended is much too low.
B. Most people overestimate how much they do.
C. Women now exercise more than they used to.
19. Adam refers to the location and width of stairs in a train station to illustrate
A. practical changes that can influence people’s behaviour.
B. methods of helping people who have mobility problems.
C. ways of preventing accidents by controlling crowd movement.
20. What do the students agree about including reference to exercise in their presentation?
A. They should probably leave it out.
B. They need to do more research on it.
C. They should discuss this with their tutor.
21. What are the students going to do next for their presentation?
A. prepare some slides for it
B. find out how long they have for it
C. decide on its content and organisation

PHẦN II:

Questions 1 - 2. Choose the word whose underlined part is pronounced differently from the
other three in each of the following questions. (2 ms)
Question 1 A. sought B. drought C. bought D. fought
Question 2 A. reservoir B. domestic C. optimist D. nursery
Questions 3 - 4. Choose the word that differs from the other three in the position of primary
stress in each of the following questions. (2 ms)
Question 3 A. advocate B. disallow C. interrupt D. overcome
Question 4 A. comparable B. benevolent C. devastating D. charitable
Questions 5 - 37. Choose the correct answer to each of the following questions. (33 ms)
Question 5 I'd rather you __________ anything about the garden until the weather improves.
A. hadn't done B. wouldn't have done
C. won't do D. didn't do
Question 6 They have arranged special insurance to cover medical __________ in the event of
an accident.
A. numbers B. expenses C. money D. amounts
Question 7 I really object __________ being charged for parking.
A. on B. for C. to D. against
Question 8 Now, don't tell anyone else what I've just told you. Remember, it is __________.
A. confident B. confidence
C. confiding D. confidential
Question 9 Most people had moved to safe areas when the storm broke. Only __________
minor injuries were reported.
A. a little B. little C. few D. a few
Question 10 It will be more effective if we all __________ and present a united front when we
ask the coach to change his training techniques.
A. speak out against B. speak with one voice
C. tell the difference D. lower our voice
Question 11 Phone me before ten; __________ I'll be too busy to talk to you.
A. unless B. whether C. if D. otherwise
Question 12 The bank is reported in the local newspaper __________ in broad daylight
yesterday.
A. to be robbed B. having been robbed
C. to have been robbed D. robbed
Question 13 He __________ have watered the plants. If he had, they wouldn't have died.
A. shouldn't B. needn't C. wouldn't D. can't
Question 14 The brochure says that the hotel has a great __________ of the sea.
A. appearance B. look C. sight D. view
Question 15 The harder you try, __________ you will get.
A. better results B. the better results
C. more better results D. the best results
Question 16 The webmaster will decide whether to __________ the web page or make the
required changes.
A. write down B. move down C. pull down D. take down
Question 17 Carl was injured last week, and the doctor recommended that he __________ in
the next match.
A. can't play B. not play C. won't play D. might not play
Question 18 __________ saying was so important that I asked everyone to stop talking and
listen.
A. What the woman was B. That was the woman
C. The woman was D. How was the woman
Question 19 Everyone knows about pollution problems, but not many people have __________
any solutions.
A. come up with B. thought over C. got round to D. looked out
Question 20 Unlike most birds, __________.
A. the heads and necks of vultures lack feathers
B. feathers are not found on the heads and necks of vultures
C. there are no feathers on vultures' heads and necks
D. vultures do not have feathers on their heads and necks
Question 21 Even if you are rich, you should save some money for a __________ day.
A. windy B. foggy C. rainy D. snowy
Question 22 __________ the ringing phone when it went dead.
A. Hardly I reached B. I had no sooner reached
C. Just as I reached D. Scarcely had I reached
Question 23 __________, he agreed to help his wife with the household chores.
A. So tired did he feel B. Such was his tiredness
C. No matter how tired he felt D. But for his tiredness
Question 24 Although we argued with him for a long time, he __________ his ground.
A. remained B. stood C. kept D. persisted
Question 25 We regret to inform you that the materials you ordered are __________.
A. out of work B. out of reach C. out of stock D. out of practice
Question 26 It's often said that there's a __________ line between genius and madness.
A. long B. short C. fine D. thick
Question 27 Before you start cooking, you should gather together all the necessary
__________.
A. factors B. substances C. elements D. ingredients
Question 28 I am writing __________ your advertisement for interns, which is currently
posted on your website.
A. by all means of B. with regard to C. in place of D. in honor of
Question 29 The interviewer asked me a lot of professional questions, __________ I couldn't
answer.
A. of which most B. most of which C. most of that D. most of them
Question 30 Shelley disagreed with the board's decision so she __________ and went to work
for another company.
A. retired B. sacked C. fired D. resigned
Question 31 When his alarm went off, he shut it off and slept for __________ 15 minutes.
A. other B. others C. the others D. another
Question 32 Meet Gabriel, she's a __________ cat that we adopted a month ago.
A. British cute white tiny B. cute tiny white British
C. cute white tiny British D. tiny cute white British
Question 33 The mother was afraid to let the boy __________ the street alone.
A. risk to cross B. risk crossing C. to risk crossing D. to risk to cross
Question 34 I disapprove __________ people whispering to each other in a meeting.
A. with B. of C. at D. on
Question 35 In football, a penalty shot is awarded when a major foul is __________ inside the
16.5-meter line.
A. prevented B. presented C. committed D. contacted
Question 36 He retired early __________ his ill health.
A. on behalf of B. in front of C. on account of D. apart from
Question 37Don't smoke in the forest. Fires __________ easily at this time of the year.
A. catch up B. break out C. set off D. trigger off
Questions 38 - 40. Choose the underlined part that needs correction in each of the following
sentences. (3 ms)
Question 38 Once having provided with sufficient information, the freshmen will
A B C

feel more confident to start the new course.


D
Question 39 The aim of the award in this competition is to encourage
A B C
participants' creation and innovation.
D
Question 40 It's high time the government take tougher measures to reduce the
A B C
pollution in this city.
D
Questions 41 - 50. Read the following passage and choose the correct word or phrase that
best fits each of the numbered blanks. (10 ms)
IS TRADITION AN OBSTACLE TO PROGRESS?
Most of what we do in our daily lives follows procedures that come from the past. They are
habits and customs (41) __________ throughout the centuries, and they have a profound relation
to the unchanging natural order of things. When people or institutions (42) __________ a custom,
it is because its efficiency had already been tested and approved for generations as the best way
to act. (43) __________, people and institutions have a large stock of these wise customs and a
natural tendency to continue them. They confer stability. Things become fixed in tradition because
they work well that way: they aren't seen as a straitjacket, limiting movement. One may adjust the
custom, but only to improve it, not destroy it. Of course, if one was to call every practice
"tradition" and refuse to change anything for its (44) __________, society wouldn't move forwards
at all.
Some object that tradition produces (45) __________ because nothing changes. This is a rash
judgment. Traditions don't become mouldy; they constitute an inheritance of wise practices and
customs that prevent us from having to waste time figuring out (46) __________ our ancestors
have already figured out - Where's the point in trying to reinvent the wheel? It is constantly starting
again that produces sluggishness. Leave (47) __________ enough alone. When one accepts
established solutions for problems, one is free to accomplish creative additions in other
unexplored fields. This is the way societies progress and rise (48) __________ a higher level of
civilisation. To constantly change everything is to condemn society to instability, perpetually
remaining at the same level of civilisation, or even (49) __________ into decay. Important
traditions which are held sacred don't interfere with progress: they give us a (50) __________ of
belonging! When we belong, we care enough to think about how we can improve our own lives
and the lives of those around us.
Question 41 A. posed B. caused C. created D. raised
Question 42 A. break B. adopt C. win D. gain
Question 43 A. Therefore B. However C. By contrast D. Even if
Question 44 A. right B. sake C. role D. self
Question 45 A. stagnation B. difference C. revolution D. improvement
Question 46 A. when B. why C. what D. where
Question 47 A. any B. some C. just D. well
Question 48 A. off B. to C. on D. with
Question 49 A. dropping B. moving C. falling D. holding
Question 50 A. sense B. mind C. brain D. head
Question 51 - 57. Read the passage and choose the correct answer to each of the following
questions. (7 ms)
ME AND MY BRAINS
To paint a clearer picture, we should first familiarize ourselves with the different parts of the
brain. Did you know, for instance, that our brains are made up of around 100 billion nerve cells
called neurons? And stemming from these neurons are several branch-like structures for sending
and receiving electrical signals? Every time we do or think anything, a signal is transmitted. The
signal travels down a long structure called axon and, at the end it passes across tiny gaps called
synapses to the dendrites of another neuron, which receive the signal. In this way, messages are
sent across our neural network.
Our brain structure changes dramatically as we grow up. Newborn babies have almost all
their neurons but few connections between them, which is why they can't do very much. After a
few months however, the number of connections explodes, which in turn helps tiny tots master a
whole range of new skills such as walking and talking. Despite earlier myths that, most brain
development is completed in the first few years, we know that our brains continue to develop
throughout our lives and perhaps the most dramatic time of change and development is during
puberty.
During this period of reorganization, the brain witnesses a sudden increase in neurons not
dissimilar to a plant growing uncontrollably in spring. Just as we prune a plant to make it stronger
and healthier, we prune our brains. The connections that are used become stronger, whereas those
which aren't used wither and die. So, the more frequently an action or thought is activated, the
stronger the connections become between the neurons, which in turn strengthens the part of the
brain being used. This explains why the more you do something, the better you become at it,
reinforcing the old adage "practice makes perfect". In fact, it would seem that the teenage brain
provides optimum conditions for perfecting skills such as playing a musical instrument, speaking
another language or learning a complex computer game. li could therefore be argued that teenagers
determine the development of their own grey matter through the activities and experiences they
engage in.
It may also be unsurprising to many to learn that the last part of the adolescent brain to
develop is the frontal cortex, responsible for self-control, problem solving and decision making.
Consequently, long before teens become adept at rational, abstract thinking and logical decision
making, they rely on the emotional centre of the brain to make choices and think. So perhaps
unpredictable, volatile, risk-taking teenage behaviour, often put down to hormones, may
actually have more to do with what's going on inside our brain.
Question 51 What do we learn about the structure of the brain?
A. All neurons are activated when we think.
B. The structure of a brain cell is compared to a tree.
C. The structure of a brain cell changes when a message is sent.
D. Neurons come into contact with one another to allow a message to travel around the brain.
Question 52 What does the second paragraph tell us?
A. Brain development is an ongoing process.
B. Babies are born with the capacity to do anything.
C. A reduction in connections between neurons helps babies learn how to speak.
D. It is no longer believed that most changes in the brain occur before adulthood.
Question 53 What does the text tell us about brain reorganisation?
A. It normally occurs in the spring.
B. When we engage in an activity, we strengthen connections in the brain.
C. As the number of neurons increases, the brain gets stronger.
D. People who play a musical instrument have stronger brain connections than those who play
sport.
Question 54 What does the writer imply in the third paragraph?
A. All teens should play a musical instrument.
B. It is just as difficult to speak another language as it is to play computer games.
C. If we don't practise an activity in our teenage years, we won't be able to do it as an adult.
D. Teens can influence their own brain development.
Question 55 During adolescence, __________.
A. all parts of the brain develop simultaneously
B. we make emotional decisions because of our hormones
C. we gradually improve our ability to think in abstract terms
D. we are unable to carry out problem-solving tasks
Question 56 In the final paragraph, the word "volatile" is closest in meaning to __________.
A. unstable B. bad-tempered C. aggressive D. lunatic
Question 57 Overall, the writer thinks teenagers __________.
A. are misunderstood
B. want to learn more about the changes in their brains
C. are more intelligent than they used to be
D. can benefit from understanding the changes that happen in their brains
Questions 58 - 62. Read the text. Five sentences have been removed from the text. Choose from
sentences A-T the one that best fits each gap. There is ONE extra sentence that you do not need.
(5 ms)
TREES IN TROUBLES
What is causing the decline of the world's giant forests?
Big trees are incredibly important ecologically. For a start, they sustain countless other
species (58) __________. The trunks and branches can become gardens, hung with green ferns,
orchids and bromeliads, coated with mosses and draped with vines. With their tall canopies
basking in the sun, they capture vast amounts of energy. This allows them to produce crops of
fruit, flowers and foliage that sustain much of the animal life in the forest.
Only a small number of tree species have the genetic capacity to grow really big. The
mightiest are native to North American, but big trees grow all over the globe, from the tropics to
the boreal forests of the high latitudes. To achieve giant stature, a tree needs three things: the right
place to establish its seedling, good growing conditions and lots of time with low adult mortality.
(59) __________.
In some parts of the world, populations of big trees are dwindling because their seedlings
cannot survive or grow. In southern India, for instance, an aggressive non-native shrub, Lantana
camara, is invading the floor of many forests. Lantana grows so thickly that young trees often fail
to take roof. With no young trees to replace them, it is only a matter of time before most of the
big trees disappear. Across much of northern Australia, gamba grass from Africa is overturning
native savannah woodlands. (60) __________. This creates super-hot fires that cause catastrophic
tree mortality.
(61) __________. There is some evidence to suggest tree growth could slow in a warmer
world, particularly in environments that are already warm. Having worked for decades at La Selva
Biological Station in Puerto Viejo de Sarapiqui, Costa Rica, David and Deborah Clark and
colleagues have shown that tree growth there declines markedly in warmer years. "During the
day, their photosynthesis shuts down when it gets too warm, and at night they consume more
energy because their metabolic rate increases, much as a reptile's would when it gets warmer,"
explains David Clark. With less energy produced in warmer years and more being consumed just
to survive, there is even less energy available for growth.
The Clark's hypothesis is correct, which means tropical forests would shrink over time. (62)
__________. According to the Clarks, this might trigger a destabilization of the climate; as older
trees die, forests would release some of their stored carbon into the atmosphere, prompting a
vicious cycle of further warming, forest shrinkage and carbon emissions.
A. The loss of big trees has had impacts on the wider environment
B. The grass grows up to four meters tall and bums fiercely
C. The largest, oldest trees would progressively die off and tend not to be replaced
D. Without the right growing conditions, trees cannot get really big
E. They provide shelter for many animals.
F. Disrupt any of these, and you can lose your biggest trees
Questions 63 - 70. Use one of the words listed in the box to form a new word and fill in each
blank. (8 ms)
attendance competitor identify office popularity present hoist motive

The 31st Southeast Asian Games (SEA Games 31), the region's biggest sporting event, (63)
__________ began with a grand opening ceremony held at My Dinh National Stadium in Hanoi
on May 12th evening. President Nguyen Xuan Phuc, National Assembly Chairman Vuong Dinh
Hue, and Deputy Prime Minister Vu Duc Dam were among the Vietnamese leaders (64)
__________ the ceremony. International guests include Speaker of the Singaporean Parliament
Tan Chuan-Jin, ministers and heads of the sports delegations of the 11 participating countries,
chief (65) __________ of international organisations, and foreign ambassadors. The opening
ceremony saw the presence of a large number of athletes and coaches from the 11 delegations who
have been (66) __________ in 40 sports since May 4. The Vietnamese delegation comprises more
than 1,300 members. The event began with the Vietnamese flag (67) __________ ceremony,
which was followed by special art performances by over 1,000 artists and athletes to introduce the
Vietnamese culture and honour the cultural (68) __________ of the countries in Southeast Asia,
aiming to strengthen solidarity and friendship, promote the spirit of contribution, dedication and
honesty, and (69) __________ people's inner strength through sports competition. The ceremony
looked to not only (70) __________ images of Vietnam but also express the country's readiness
for integration and contribution "For a Stronger Southeast Asia" as the theme of the Games goes.
Questions 71 - 80. Read the following passage and think of a word that best fits each gap. Use
ONLY ONE word in each gap. (10 ms)
There is emerging evidence that some people who develop long Covid have similar
symptoms to people with a condition known (71) _________ postural tachycardia syndrome, or
postural orthostatic tachycardia syndrome (PoTS). This condition can cause dizziness when
moving to an upright position and can be triggered by infections. The most common (72)
__________ are feeling lightheaded, palpitations (being aware of your heartbeat) and fatigue.
There have been several case reports in medical journals of people who had continuing
symptoms following Covid-19 infection, and some were found to have PoTS. PoTS happens
because your body's ways of avoiding a drop in blood pressure when you stand up aren't working
(73) __________. Normally when you sit up or stand up, gravity makes some of your blood flow
downwards, (74) __________ can cause a fall in blood pressure. Your body (75) __________ to
prevent a fall in blood pressure by narrowing your blood vessels and slightly increasing your heart
rate. But (76) __________ you have PoTS, these automatic changes don't happen. When you
move to an upright position, the supply of blood to your heart and brain drops and your heart starts
(77) __________ faster to compensate.
While there is (78) __________ cure for Pots, it can be managed by diet, exercise and
medication, and the researchers say it is important to get an accurate diagnosis so patients can be
treated and advised on how to manage their symptoms.
If this sounds (79) __________ your symptoms after having Covid-19, you should (80)
__________ your doctor know to ensure you are correctly diagnosed and can receive effective
treatment.
Questions 81 - 90. Complete the second sentence using the word given, so that it has a similar
meaning to the first sentence. DON'T CHANGE THE WORD GIVEN. Write between THREE
and SIX words. (20 ms)
Question 81 Danny found it difficult to control his bicycle on the steep slope. (UNDER)
→ Danny found it difficult __________ on the steep slope.
Question 82 You should think about the price before you decide whether to buy it or not.
(CONSIDERATION)
→ You should __________ before you decide whether to buy it or not.
Question 83 Don't forget you are due to see the dentist at 10 o'clock. (APPOINTMENT)
→ Don't forget that __________ the dentist at 10 o'clock.
Question 84 That dress is a third of the cost of the pink one. (MUCH) → The
pink dress is __________ that one.
Question 85 The two theories appear to be completely different. (COMMON) → The two
theories seem __________.
Question 86 68611]: Jake was the person who started my interest in collecting pottery. (GOT)
→ It was __________ collecting pottery.
Question 87 I wish people wouldn't talk during a performance - it makes me really uptight!
(NERVES)
→ It really __________ people talk during a performance.
Question 88 We didn't understand how challenging the programme would be. (DEMANDS)
→ Little __________ of the programme would be.
Question 89 You have to use logic and lateral thinking in equal measure in this job.
(STRIKE)
→ You have to __________ logic and lateral thinking in this job.
Question 90 Being in prison seems to have changed Kevin's behaviour for the better. (LEAF)
→ Kevin has __________ since he got out of prison.
TEST 4
PHẦN I – LISTENING
Part 1: For each question, write the correct answer in the gap. Write one or two words or a
number or a date or a time. You will hear a student giving some information to his class about
an acting club he’s a member of.
My acting club
Teacher:
Appeared on TV as a (1) __________
Has mostly appeared in (2) __________ shows
Sessions:
First part – using your (3) __________ well
Second part – practising performance skills
Take place at the (4) __________
Acting Club play:
Called (5) __________
First performance – on (6) __________
Part 2: You will hear five short extracts in which people talk about deciding to take up new
interests. For questions 7–11, choose from the list (A–H) the reason each speaker gives for
deciding to take up their new interest. Use the letters only once. There are three extra letters
which you do not need to use.
A. to help other people
B. to make some money
C. to pass the time
D. to meet new people
E. to try to win a prize
F. to overcome a fear
G. to get fit
H. to make a change from their work
Question 7. Speaker 1 __________
Question 8. Speaker 2 __________
Question 9. Speaker 3 __________
Question 10. Speaker 4 __________
Question 11. Speaker 5 __________
Part 3: Choose the correct letter, A, B or C.
12. Dartfield House school used to be
A. a tourist information centre.
B. a private home.
C. a local council building.
13. What is planned with regard to the lower school?
A. All buildings on the main site will be improved.
B. The lower school site will be used for new homes.
C. Additional school buildings will be constructed on the lower school site. 14.
The catering has been changed because of
A. long queuing times.
B. changes to the school timetable.
C. dissatisfaction with the menus.
15. Parents are asked to
A. help their children to decide in advance which serving point to use. B.
make sure their children have enough money for food.
C. advise their children on healthy food to eat.
16. What does the speaker say about the existing canteen?
A. Food will still be served there.
B. Only staff will have access to it.
C. Pupils can take their food into it.
PHẦN II-– PHẦN TRẮC NGHIỆM

PHONETICS
Part 1. Circle the letter A, B, C or D to indicate the word whose underlined part is
pronounced differently from that of the rest in each of the following questions.
Question 1 A. staple B. versatile C. buffalo D. career
Question 2 A. conserve B. reserve C. preserve D. observe
Part 2. Circle the letter A, B, C or D to indicate the word that differs from the other three in
the position of primary stress in each of the following questions.
Question 3 A. regretful B. denial C. sarcastic D. secular
Question 4 A. event B. igloo C. vehicle D. pastime
Question 5 A. variety B. marinate C. imperial D. magnificence
VOCABULARY AND GRAMMAR
Part 1. Circle the best option A, B, C or D to complete the following sentences.
Question 6 The thieves cleared __________ when they heard the guard approaching.
A. off B. away C. out D. up
Question 7 If you catch them stealing again, there will be __________ for it but call the police.
A. no help B. no choice C. no knowing D. saying
Question 8 __________ we not got lost along the way, it would have been a great journey.
A. Might B. Would C. Should D. Had
Question 9 They are __________ the best employees in the company.
A. so far B. as far C. far better D. by far
Question 10 __________ I'd like to help you out, I'm afraid I just haven't got any spare money
at the moment.
A. Even B. Despite C. Try as D. Much as
Question 11 Do not take it as __________ that anyone applying for the course would have the
necessary qualifications.
A. heard B. written C. spoken D. read
Question 12 You shouldn't accept everything politicians say at face __________ as they are
often trying to win votes.
A. merit B. value C. worth D. benefit
Question 13 If a diamond is heated without oxygen, it will turn to graphite, a form of
__________ that it is used as a lubricant.
A. carbon is so soft B. is carbon so soft C. carbon so soft D. so soft the carbon
Question 14 It is important that you __________ there when he gets off the plane.
A. be standing B. are standing C. have stood D. will be standing
Question 15 After years of research, scholars have finally __________ this anonymous play to
Christopher Marlowe.
A. subscribed B. described C. ascribed D. prescribed
Part 2. Mark the letter A, B, C, or D on your answer sheet to indicate the word(s) CLOSEST
in meaning to the underlined word(s) in the following question.
Question 16 The security will always be on hand in case of emergency.
A. ready to help B. able to respond promptly
C. completely busy D. professional
Mark the letter A, B, C, or D on your answer sheet to indicate the word(s) OPPOSITE in
meaning to the underlined word(s) in the following question.
Question 17 It's time you pulled your socks up and started working seriously. Your final
exams are coming.
A. got furious B. studied harder C. stayed focus D. became lazy
READING
Read the passage and decide which answer (A, B, C or D) best fits each gap.
We have all heard that good study habits involve sitting somewhere quiet, sticking to a schedule
and setting yourself targets. However, such habits don't work for everyone and cognitive
scientists have come up with some new findings. In fact, the brain makes (18) __________
associations between what it is studying and the background sensations it has at the time. (19)
__________ study to one place may therefore have (20) __________ effects because when the
context is varied, the information being studied is enriched and forgetting is slowed down. In
(21) __________, the information is given more support from the multiple associations.
Evidence also shows that varying the type of material studied in a single sitting leaves a deeper
impression on the brain than does concentrating on one skill at a time. Musicians have known
this for years, as have athletes who routinely (22) __________ strength, speed and skill drills
into their workouts. These ideas seem to work when applied to any subject so it seems time for
this (23) __________ to studying to be taken more seriously.
Question 18 A. thoughtful B. subtle C. sensitive D. influential
Question 19 A. Restricting B. Modifying C. Controlling D. Enclosing
Question 20 A. hurtful B. detrimental C. misleading D. punishing
Question 21 A. honesty B. force C. origin D. effect
Question 22 A. incorporate B. include C. comprise D. unite
Question 23 A. procedure B. theory C. concept D. approach
PHẦN II PHẦN TỰ LUẬN VOCABULARY AND GRAMMAR
Part 1. Think of ONE word which can be used appropriately in all THREE sentences.
Question 1 : __________
The long journey completely __________ me.
I've already __________ the prawns, Mum.
The long war had __________ the resources of both countries.
Question 2: __________
The teacher said he would not __________ such rude behavior in his class. We do not
__________ smoking in the hall.
These websites __________ consumers to compare the prices of all telephone providers.
Question 3 : __________
It's a very stylish and beautiful film, but it lacks __________.
Women are no longer __________ with staying at home, in the sole role of a housewife. You've
got a whole week to yourself and you can read to your heart's __________.
Question 4 : __________
When Lily heard a strange noise upstairs, she suddenly __________ and listened attentively.
When the company was going bankrupt, the government __________ all their assets. We
__________ the leftovers from last night's meal in the fridge to keep them from going bad.
Question 5 : __________
Her hair was __________ brown.
They spent years travelling around in a __________ old van.
You may be a little __________, but past experience and teaching skills won't have been lost.
Part 2. Give the correct form of the words in brackets to complete the passage. On the
outskirts of Vietnam's capital, well removed from the tourist trail, is Dong Ngac, a 1,000-year-
old 'village of scholars' that has barely changed for centuries. Despite being tiny - the village is
less than one square meter in size and home to 1,000 or so people, Dong Ngac has birthed an
unusually large number of academic high (6. achieve) __________, including dozens of
doctors of literature. These include Pham Gia Khiem, Vietnam's deputy prime minister from
1997 to 2011, and Hoang Tang Bi, a (7. reverence) __________ writer and social activist in the
early 1900s. To this day, families here compete fiercely against each other to rear the most (8.
stand) __________ pupils. So valued is Dong Ngac for producing influential scholars that it
has been presented with dozens of royal awards, dating as far back as Vietnam's Le Dynasty
period (1428-1788).
The village's (9. literature) __________ tradition is even celebrated via its architecture. (10.
Symbolize) __________ of books are carved into the old gates that sit at each end of its four
hamlets. (11. Scatter) __________ throughout these hamlets are almost 100 homes, the oldest of
which date back to the early 1600s. The complexity of their stonemasonry and woodwork (12.
captivating) __________ as one walks through its nest of narrow lanes, being greeted by smiles
and waves from residents, who share treats like (13. pipe) __________ hot lotus tea and Banh
Gio rice dumplings, two of local specialties.
READING
Part 1. Read the passage carefully and then fill ONE suitable word in each gap.
Until now, the word 'travel' has always conjured up visions of packed bags, the open road, and
generally the idea of escape. However, an exciting new concept is changing all that. Not (14)
__________ may you now travel to another country, but to another 'world', (15) [
__________even leaving home. 'Second Life' is an interactive simulated world on the Internet
which offers visitors the ultimate form of escapism; the ability to reinvent (16) __________ and
live their dream life. First, you create your persona, called an avatar, and (17) __________ it a
name. Then you start exploring the virtual environment, and choose to interact, or not, (18)
__________ the other avatars you meet. For many, the beauty of the avatars is that they can fly.
You can travel great distances, gaining a bird's eye view of various 'environments',
(19)__________ the effect is rather surreal. The world changes while you watch, as other more
experienced 'gamers' buy and sell property, using for currency the Linden dollar, which has an
(20)_________ rate against the US dollar. Never before (21) __________ a game offered
players so many options! You can shop, work, travel and form relationships, go to parties, or
hold your own parties if you want to. 'Second Life' is expanding rapidly, as more and more
people are (22) __________ attracted to it, so don't miss out! Simply allow your PC to transport
you there and let your imagination do the (23) __________!
Part 2. Read the following passage and answer the questions.
Islands remote areas are showing the world how sustainable it can be to use renewables
for energy needs.
A. There are many reasons why countries around the world are trying to switch to renewable
sources of power such as wind, wave and solar power. First and foremost is the amount of
environmental damage caused by fossil fuel use. CO2 emissions from oil and coal powered
electricity plants along with oil used in industry, and for transport, are all contributing to climate
change, a threat that could have enormous consequences for the majority of populations on the
planet. Another issue for remote areas in the world is the cost of transporting fossil fuels in order
for them to be able to generate power. Transporting oil to remote areas and islands is costly and
potentially dangerous. For these reasons, renewable sources of energy are very attractive to areas,
such as small remote islands, where shipping in fossil fuels is prohibitively expensive and can be
affected by the difficulties of getting supplies in by ship because of storms at sea.
B. A number of islands around the world have switched to using renewables to meet the
majority of their electricity needs and have also committed to becoming 100% self-sufficient in
renewable energy in the near future. Islands make good test grounds for the use of renewable
sources because they tend to be small, so it is relatively easy to produce enough energy to power
an area of that size. Geography also contributes to making these experiments more productive as
islands are more likely to be exposed to high winds, have access to waves as well as more days of
sunshine. Islands can therefore pioneer the use of a variety of renewables together and give a
picture, in microcosm, on how bigger states can follow their lead. The lessons that these smaller
areas learn from the use of renewables can be passed on to help to make the transition to greater
use of renewable sources smoother in larger areas, such as urban conurbations.
C. One problem with renewables is whether they can be depended on to give unlimited energy.
If there's no wind or the sun is behind a cloud then power can't be generated and energy storage
technology is still in its infancy This means that renewables can't always be depended on. And
that isn't the only problem with renewable energy sustainability. Power companies need to make
money in order to operate and provide energy to populations and businesses. One way renewables
have affected power company viability is because governments have given households subsidies
for installing solar panels. The subsidies are repaid by selling back excess energy to the power
companies, which means that, in effect, those with solar power make no financial contribution to
using the electricity grid's infrastructure and this leaves the other customers to pick up the slack
with higher prices. But power companies are looking to reduce the price they pay for excess solar
power from households so that other customers pay a fairer price and the power company remains
competitive as a business.
D. One island showing that renewable energy can be viable is the Spanish Canary Island of El
Hierro. It is the smallest and most isolated of the Canary Islands and because of the geography of
the local seabed, electricity cannot be supplied by underwater cables and shipping in fuel isn't
always possible either when Atlantic storm systems are en route. So, to combat the problems
caused by depending on one renewable for electricity, Al Heirro has developed a mixed renewable
energy system using both wind and hydro to produce most of the island's energy. The system
consists of five wind turbines and two water reservoirs. One of the reservoirs is situated at 700
meters above sea level and the second is down on the coast. The reservoirs are connected by two
3 km long pipes that contain hydraulic turbines that also produce energy. Due to the island having
a lot of wind, the wind turbines normally produce plenty of electricity to supply the grid and the
excess electricity is used to pump water from the lower reservoir to the upper one. When the wind
drops, the water from the upper reservoir can be released through the piping so the hydraulic
turbines can create energy on windless days. This use of mixed renewables combats the problem
caused when one source becomes inoperable because of unfavourable weather.
E. Many other islands are developing renewable systems that can be regarded as test runs for
similar projects in larger areas. Samso, in Demark is showing how switching to renewables not
only means more environmentally friendly, cleaner energy, it can also help with economic
regeneration by creating jobs related to the energy industry. Kodiak Island in Alaska is another
place using a mixed renewable system. It has a back-up battery storage system that cuts in when
the grid switches from wind to hydro and this prevents sudden power losses that cause flickering
lights or cut-offs. The island of Tilos in Greece is not only working towards one hundred percent
self-sufficiency with renewables but looking at using
that energy to power motorbikes and vehicles on the islands as well. In short, small islands are
showing the path the rest of the world can take to a sustainable energy future.
Questions 24-27.
Label the diagram below. Choose ONE WORD ONLY from the text for each answer.
Questions 28-31. Complete the sentences below.
Choose NO MORE THAN TWO WORDS from the text for each answer.
28. Carbon dioxide__________ are a major factor in the creation of climate change.
29. Because of their small size, islands are ideal__________ for the use of renewable sources.
30. Households with solar power may not make a__________ to power companies although
they still use existing structures.
31. Samso has shown how renewable energy can aid__________ as well as help the
environment.
WRITING
Part 1. Rewrite the following sentences without changing their meaning using the words in the
brackets. Do not change the words given. You must use between 2 and 5 words, including the
word given.
Question 32 The jumper you knitted for her daughter no longer fits her. (grown)
⤷ My daughter __________ the jumper you knitted for her.
Question 33 Minh was very pleased to be selected for the team. (delight)
⤷ Much __________, he was selected for the team.
Question 34 I am dying to visit Santorini. (bucket)
⤷ Santorini is __________.
Question 35 [ Diane finds that creating things stops her from thinking about her work. (mind)
⤷ Diane finds that creating things __________ her work.
Question 36 The project has been given funding for another year, but there is still difficulty.
(woods)
⤷ The project has been given funding for another year, but it is not __________ yet.
Question 37 He hasn't announced anything, but my take on the situation is that the Prime
Minister's going to resign soon. (lines)
⤷ He hasn't announced anything, but__________, I'd say that the Prime Minister's going to
resign soon.
Question 38 Fans were anxious waiting for the final whistle. (breath)
⤷ Fans __________ waiting for the final whistle.
Question 39 I promise that your money is safe with me. (word)
⤷ I give __________ that your money is safe with me.
Part 2. Rewrite each of the following sentences beginning the word given in bold so that it has
the same meaning as the original one.
Question 40 The light went out the moment they stepped into the room.
⮲ Hardly _________________________________________________________________
Question 41I didn't visit you because I didn't know you were in hospital.
⮲ If I _______________________________________________________
Question 42 It seems that no one predicted the correct result.
⮲ No one ________________________________________________________________
Question 43 The electrician has mended my broken light.
⮲ I have ___________________________________________________________
Question 44 We weren't surprised by his success.
⮲ It came ________________________________________________________________
Question 45 Despite his anger, he apologized to his friend for shouting.
⮲ No matter how________________________________________________________
Question 46 : The house was too expensive for them to buy.
⮲ It was
_______________________________________________________________________
Part 3. Write an academic essay of about 250 words on the following topic. “It is said that
self-study is very important for secondary school students”. Do you agree or disagree with the
statement? Use specific reasons and examples to support your answer.
TEST 5

PHẦN I: LUYỆN NGHE


Part 1: For each question, write the correct answer in the gap. Write one or two words or a
number or a date or a time. You will hear a teacher giving his students information about a
school trip to a farm. (Taken from PET trainer)
School trip to farm
Meeting place: 8 a.m. next to the (1) __________
Need to bring: a (2) __________
Morning activity: feeding the (3) __________
Afternoon activity:
(4) __________
Return to school at:
(5) __________
For more information:
(6) www. __________.farm.com
Part 2: You will hear five short extracts in which people are talking about their visit to a city.
For questions 7 – 11, choose from the list (A – H) what each speaker liked most about the city
they visited. Use the letters only once. There are three extra letters which you do not need to
use. (Taken from FCE Test)
A. the efficiency of the public transport system
B. the natural beauty of the scenery
C. the variety of goods in the markets
D. the style of the architecture
E. the well-designed plan of the city
F. the helpfulness of the people
G. the range of leisure opportunities
H. the standard of the accommodation
Question 7. Speaker 1 __________
Question 8. Speaker 2 __________
Question 9. Speaker 3 __________
Question 10. Speaker 4 __________
Question 11. Speaker 5 __________
Part 3: Questions 12 – 21 (Taken from IELTS Cambridge 11)
New staff at theatre
Questions 12 and 13: Choose TWO letters, A-E.
Which TWO changes have been made so far during the refurbishment of the theatre?
A. Some rooms now have a different use.
B. A different type of seating has been installed.
C. An elevator has been installed.
D. The outside of the building has been repaired.
E. Extra seats have been added.
Questions 14 and 15: Choose TWO letters, A-E.
Which TWO facilities does the theatre currently offer to the public?
A. rooms for hire
B. backstage tours
C. hire of costumes
D. a bookshop
E. a café
Questions 16 and 17: Choose TWO letters, A-E.
Which TWO workshops does the theatre currently offer?
A. sound
B. acting
C. making puppets
D. make-up
E. lighting
Questions 18-21: Label the plan below.
Write the correct letter, A-G, next to Questions 17-20.
Ground floor plan of theatre
18. box office __________
19. theatre manager’s office __________
20. lighting box __________
21. artistic director’s offices __________
PHẦN II: (TRẮC NGHIỆM)
PART A: PHONETICS
Questions 1 - 3. Choose the word whose underlined part is pronounced differently from that
of the others in the same line and write A, B, C or D on your answer sheet.
Question 1 A. cooked B. crashed C. crossed D. crooked
Question 2 A. hesitate B. basic C. reserve D. resumé
Question 3 A. hasty B. nasty C. wastage D. taste
Questions 4 - 5. Choose the word whose stress pattern is different from that of the others in
the same line and write A, B, C or D on your answer sheet.
Question 4 A. literature B. approximate C. immediate D. apparently
Question 5 A. interfere B. innovate C. sacrifice D. penalize
PART B: GRAMMAR AND VOCABULARY
Questions 6 - 15. Choose the word/phrase that best fits the gap in each of the following
sentences and write A, B, C or D on your answer sheet.
Question 6 Tung and Hoa are talking about time management skills.
- Tung: "I reckon that time management skill is very essential to our study."
- Hoa: “__________. We should spend our time wisely to study well."
A. It's time to go B. I couldn't agree with you more
C. That's wrong D. I can't think about it
Question 7Your rental agreement __________ states that no pets are allowed in the building.
A. credibly B. explicitly C. mildly D. decently
Question 8 All of my friends are using TikTok, so I decided to jump on the __________ and
join it, too.
A. media B. platform C. bandwagon D. departure
Question 9 The thick fog __________ out any possibility of our plane taking off before
morning.
A. ruled B. struck C. stamped D. crossed
Question 10 There are __________ words in English having more than one meaning. Pay
close attention to this fact.
A. a large many B. many a
C. quite a lot D. a great many
Question 11 The __________ prices of property in big cities may deter people on low
incomes from owning a house there.
A. competitive B. forbidding C. prohibitive D. inflatable
Question 12: Only a few companies were found to be in __________ with the new law.
A. submission B. obedience C. compliance D. fulfilment
Question 13 The mass media are __________ of communication, such as books,
newspapers, recordings, radio, movies, television, mobile phones and the Internet.
A. models B. means C. parts D. types
Question 14 This course __________ no previous knowledge of the subject.
A. assumes B. assigns C. assures D. assembles
Question 15 Spain has little good farmland and lacks many __________ raw materials.
A. important industry B. industrious and important
C. important industrial D. important industries
PART C: READING
Questions 16-20. Read the passage and choose the best answer to each question that follows.
Write A, B, C, or D on your answer sheet.
Twenty-three million years ago, a giant carnivore larger than any modern-day lion or polar
bear stalked sub-Saharan Africa, according to the fossils of a previously undiscovered species
that spent decades in a museum drawer at the National Museum of Kenya. The fossils were
originally found in Kenya decades ago, but at that time researchers were searching for ancient
apes, so the fossils were put aside.
The skull of the fossil is comparable to that of a rhinoceros, and given its size and giant
sharp teeth, the carnivore was at the head of its food chain. Researchers estimate that it weighed
1.6 tons and could prey on large herbivores akin to today's elephant and hippopotamus. Its
canine teeth and molars would have been able to tear flesh and crack bones easily.
The researchers dubbed the newly discovered species Simbakubwa kutokaafrika, which is
Swahili for "big lion coming from Africa." But it's not a close relative of lions, other big cats or
even mammalian carnivores that roam the Earth today. Instead, Simbakubwa was part of the
hyaenodonts, an extinct group of mammalian carnivores that lived in Africa.
After the extinction of dinosaurs, they enjoyed a 45 million-year reign as the key predators.
Additionally, tectonic plates shifted, allowing Simbakubwa to cross a landbridge into Eurasia
and flourish on other continents as well. And even though they lived for millions of years, they
went extinct between 15 million and 18 million years ago.
"We don't know exactly what drove hyaenodonts to extinction, but ecosystems were
changing quickly as the global climate became drier. The gigantic relatives of Simbakubwa were
among the last hyaenodonts on the planet," said Matthew Borths, study author and curator of the
Division of Fossil Primates at Duke University. Nancy Stevens, study co-author and professor
in the Heritage College of Osteopathic Medicine at Ohio University, noted in a statement that
"This is a pivotal fossil, demonstrating the significance of museum collections for understanding
evolutionary history. Simbakubwa is a window into a bygone era. As ecosystems shifted, a key
predator disappeared, heralding Cenozoic faunal transitions that eventually led to the evolution
of the modern African fauna."
(Adapted from https://edition.cnn.com)
Question 16 Which of the following does the passage primarily concern?
A. The characteristics of an ancient species called hyaenodonts.
B. The study of fossils and ancient dinosaurs in Kenya.
C. The discovery of an ancient carnivore's fossils in Africa.
D. The largest and oldest living carnivore.
Question 17 What does the word "akin"in the second paragraph most likely mean?
A. hostile B. similar C. dangerous D. independent
Question 18 What does the word "it" in the second paragraph refer to?
A. a rhinoceros B. the skull C. the food chain D. the carnivore
Question 19 Which of the following is TRUE about Simbakubwa?
A. They bear little relation to modern-day big cats or mammalian carnivores.
B. They reigned over many species including dinosaurs for 45 million years.
C. They originated in areas outside the continent of Africa.
D. They suffered from negative effects caused by the movement of tectonic plates.
Question 20 What can be inferred from Matthew Borths' view in the last paragraph?
A. Drier global climate had almost no effect on ecosystems.
B. Shifting ecosystems probably contributed to the extinction of hyaenodonts.
C. Researchers have determined the exact reason why hyaenodonts went extinct.
D. The gigantic relatives of Simbakubwa were last seen in Africa.
Questions 21 - 30. Read the passage and choose the most suitable word from the ones given
below to fill in each gap. Write A, B, C, or D on your answer sheet. There can be no question
that online shopping is (21) __________ huge benefit to the customer. Far from becoming (22)
__________, online shoppers are very demanding. Overpriced merchants with poor services
should beware. Gone are the days when stores could charge what they liked for goods and get
away with it. The same, too, for shady manufacturers: smarter consumers know which
products have a good (23) __________ and which do not, because online they now read not
only the sales (24) __________ but also reviews from previous purchasers. And if customers
are disappointed, a few clicks of the mouse will take them to places where they can let the
world know. Nowadays there is nothing more damning than a flood of negative comments on
the Internet. (25) __________, the big boys, as always, are ahead of the game. Some
companies are already adjusting their business models to take account of these trends. The
stores run by Sony and Apple, for instance, are more like brand showrooms than shops. They
are there for people to try out (26) __________ and to ask questions to knowledgeable staff.
Whether the products are ultimately bought online or offline is of secondary importance.
Online traders must also adjust. Amazon, for one, is (27) __________ turning from being
primarily a bookseller to becoming a (28) __________ retailer by letting other companies sell
products on its site, rather like a marketplace. During America's Thanksgiving weekend last
November, Amazon's sales of consumer electronics in the United States (29) __________ its
book sales for the first time in its history. Other transformations in the retail business are (30)
__________ to follow.
Question 21 A. for B. of C. with D. in
Question 22 A. compassionate B. dissatisfied C. competent D. complacent
Question 23 A. distinction B. reputation C. opinion D. resolution
Question 24 A. bubble B. message C. blare D. blurb
Question 25 A. Moreover B. However C. Though D. Also
Question 26 A. schemes B. tools C. emblems D. devices
Question 27 A. mistakenly B. unreasonably C. rapidly D. secretly
Question 28 A. mass B. block C. lump D. chunk
Question 29 A. receded B. exceeded C. repressed D. excluded
Question 30 A. tied B. secured C. bound D. fastened
PHẦN II: ĐỀ THI CHUYÊN THANH HÓA (TỰ LUẬN)
PART B: GRAMMAR AND VOCABULARY
Questions 1 - 7. Supply the correct form of the verbs in brackets. Write the answers on your
answer sheet.
Question 1 He (GO) __________ to the last meeting instead of staying home because it was
important for his promotion later on.
Question 2 She objected to his parents (NOT GIVE) __________ him a clear explanation the
previous day.
Question 3 Vietnamese athletes are reported (AMASS) __________ 29 more gold medals
last Sunday.
Question 4 It would have been a much more serious accident she (DRIVE) __________ fast
then.
Question 5 Peter didn't study for the test. He (COPY) __________ the answers from someone
else.
Question 6 He rubbed his eyes and yawned as though he just (WAKE) __________ up after a
long sleep.
Question 7 I would make a suggestion that your mother (SPEAK) __________ to by her
doctor before going on a diet.
Questions 8 - 15. Supply the correct form of the words in brackets. Write the answers on
your answer sheet.
Question 8 Eating that meat could be dangerous. It looks (COOK) __________ to me.
Question 9 Since 2020, the Supreme Court has held arguments via (CONFER) __________,
allowing the public to listen to live audio.
Question 10 I'm sorry to be so (DECIDE)__________ I'd like to think things over for another
day or two.
Question 11 The new version of the program comes with a much better user (FACE)
__________.
Question 12 Ann's father was in hospital due to a severe heart (ORDER) __________.
Question 13 In shadow puppetry, the screen is the medium through which the audience
experiences the performance, so selecting the best screen is among the great (ESSENCE)
__________.
Question 14 The woman detected her (CANCER) __________ conditions soon after she felt
ill in her body.
Question 15 The theft of so much money proved both (PROBLEM)__________ and
embarrassing for the company.
Questions 16 - 20. There are six mistakes in the passage. Find out and correct them. Write
the answers on your answer sheet. The first one has been done for you as an example.
Example: Line 1: is -> are
Line It is often said that books is always good friends and reading is an active mental
process. Unlike TV, books make you use your brain. By reading, you think more and
1
become smarter. Reading improves concentration and focus. Reading books takes
brain power. They requires you to focus on what you are reading for long periods.
Unlike magazines, Internet posts or e mails that might contain small pieces of
information. "Books tell the whole story". Because you must concentrate in order to
read, you will get better off concentration. Many studies show if you do not use your
memory, you lose it. Reading helps you stretch your memory muscles. Reading
5
requires remembering details, facts and figures and in literature, plot lines, themes
and characters.
Books give you knowledge of other cultures and places. The more information you
have got, the richer your knowledge is. Books can expanse your horizons by letting
you see which other cities and countries have to offer before you visit them.

10

Question 16 __________
Question 17 __________
Question 18 __________
Question 19 __________
Question 20 __________
PART C: READING
Questions 21-25. Read the following passage and choose the most suitable phrase from A- F
to fill in each gap. There is one extra phrase that you don't need to use. Write the answers on
your answer sheet.
A. the Sun will eventually explode
B. the force that pulls all things together
C. the outward push of the heat will be stronger than the Sun's gravity
D. the Sun is not getting bigger or smaller
E. it will use its fuel even faster
F. the dust and gas turned into a huge ball of fire

The Birth and Death of the Sun


About five billion years ago, there was no sun and no planets. The solar system was just a
spinning cloud of gas and dust. Over millions of years, gravity, (21) __________, brought the
dust cloud together in a ball. As the gas and dust was pressed together, it became hotter and
hotter. Eventually, it became so hot that (22) __________, and the Sun was born.
The Sun uses a gas called hydrogen as its fuel. It is extremely hot, over 16 million degrees
at the center of the Sun. Because the Sun is so hot, the gas is pushed outward by the heat. The
outward push of the gas is exactly as strong as the inward pull of gravity, so (23) __________.
This will be true as long as the Sun has hydrogen for fuel. But in another four or five billion
years, the Sun will use up most of its hydrogen. Then, (24) __________, and the Sun will start
to grow. At this time, Mercury and Venus will be destroyed and the Earth will become much too
hot for life. Because the Sun will be bigger, (25) __________. Eventually, when the Sun's fuel
is used, its fire will go out and it will start to cool. Gravity will finally pull the Sun into a small,
cold ball called a white dwarf. There will be no heat and very little light, and the solar system
will be a cold, dead place.
Questions 26-35. Read the following passage then fill in each gap with ONE suitable word.
Write the answers on your answer sheet.
The first groups of people to discover New Zealand came from Polynesia. Exactly when
these explorers arrived has often been a (26) __________ for debate, but today the general
understanding is that it was during the 13th century (27) __________ their canoes eventually
landed on New Zealand's shores.
In some ways the new country must have seemed (28) __________ an ideal place to settle:
the land was fertile, and thick forests provided fire wood, shelter, and building materials. Still,
life would have been challenging for the different Polynesian tribes, who had to (29)
__________ to a new environment. The tribes only began to refer to themselves as Maori, (30)
__________ "ordinary people”, when Europeans in (31) __________ of new opportunities
began arriving in the 18th century. To the Maori, of course, the European settlers and sailors
were not "ordinary" (32) __________ very strange.
It was not only the knowledge of canoe-building and navigation that the Polynesians
brought to New Zealand. They were also skilled craftsmen. There is archaeological evidence
that the tools they produced were (33) ] __________ high quality and would have enabled tribes
to plant and harvest crops. Craftsmen were also occupied with making weapons such as knives
and some crafts that had once been popular in Polynesian islands were no (34) __________
done in New Zealand although researchers are unsure why. Pottery is an (35) __________ of
this despite the fact that the clay needed to make pots and bowls could easily be found in the
new country.
PART D: WRITING
Questions 36 - 40. Rewrite each of the following sentences beginning as shown, so that the
meaning stays the same.
Question 36 They will not announce the findings until next week.
=> No announcement
____________________________________________________________
Question 37 Many creatures still survive and thrive in the harsh conditions of the deserts.
=> Harsh
_________________________________________________________________________
Question 38 We can't start the meeting until Tom comes.
=> Not until
__________________________________________________________________________
Question 39 We live far from our relatives. We miss them very much.
=> The farther ______________________________________________________________
Question 40 The minister gave no precise figures about casualties.
=> The minister didn't go
________________________________________________________
Questions 41 - 45. Rewrite each of the following sentences using the given words so that it
keeps the same meaning. Do not change the form of the words given.
Question 41 My friend took no notice of my advice. (DEAF)
=> My_____________________________________________________________________
Question 42 If you hadn't changed our original agreement, everything would have been fine.
(STUCK)
=> If_______________________________________________________________
Question 43 John found it difficult to get used to the fact that he was fired. (TERMS)
=> John _____________________________________________________________________
Question 44 Women outnumber men by two to one in Greece. (TWICE)
=> There _____________________________________________________
Question 45 We can't possibly imagine how we are going to afford a new car. (FAINTEST)
=> We don't__________________________________________________
Paragraph writing
Write a paragraph of about 150 words concerning the following topic: Nowadays violence is on
the increase among teenagers in many schools. Suggest some solutions to this problem.
TEST 6

PHẦN I: LUYỆN NGHE


Part 1: You will hear an interview with a girl called Jasmine, talking about her experiences
of flying a plane.
1. Why did Jasmine decide to try a flying experience day?
A. Someone recommended it.
B. She wants to become a pilot.
C. To see her area from high up.
2. How did Jasmine feel at the beginning of the flying experience day?
A. nervous about making mistakes
B. worried about how small the plane was
C. disappointed with the arrangements
3. What did Jasmine think about the training she did before the flight?
A. It was badly presented.
B. It was done too quickly.
C. Some of it wasn’t useful.
4. Jasmine says that during the flight her instructor
A. said very little.
B. stayed very calm.
C. joked with her a lot.
5. Jasmine says that the flight
A. made her feel tired.
B. seemed to last a long time.
C. was better than she had hoped.
6. Which experience day would Jasmine like to try most?
A. horse riding
B. deep-sea fishing
C. sports car driving
Part 2: You will hear five different people talking about why they have applied to go on a
space journey to the planet Mars. For questions 7–11, choose from the list (A–H) each
speaker’s reason for applying to go on the trip to Mars. Use the letters only once. There are
three extra letters which you do not need to use.
A. to discover new natural resources
B. to learn new skills
C. to take advantage of a rare opportunity
D. to be involved in advancing scientific knowledge
E. to become a famous personality
F. to face an extreme challenge
G. to provide others with inspiration
H. to be among the first to have the experience
Question 7. Speaker 1 __________
Question 8. Speaker 2 __________
Question 9. Speaker 3 __________
Question 10. Speaker 4 __________
Question 11. Speaker 5 __________
Part 3: Complete the notes below. Write ONE WORD ONLY for each
answer. Health benefits of dance
Recent findings:
- All forms of dance produce various hormones associated with feelings of happiness.
- Dancing with others has a more positive impact than dancing alone.
- An experiment on university students suggested that dance increases 12
__________. - For those with mental illness, dance could be used as a form of 13
__________. Benefits of dance for older people:
- accessible for people with low levels of 14 __________.
- reduces the risk of heart disease
- better 15 __________ reduces the risk of accidents
- improves 16 __________ function by making it work faster
- improves participants’ general well-being
- gives people more 17 __________ to take exercise
- can lessen the feeling of 18 __________ very common in older people
Benefits of Zumba:
- A study at The University of Wisconsin showed that doing Zumba for 40 minutes uses up as
many 19 __________ as other quite intense forms of exercise.
- The American Journal of Health Behavior study showed that:
+ women suffering from 20 __________ benefited from doing Zumba.
+ Zumba became a 21 __________ for the participants.
PHẦN II: (TRẮC NGHIỆM)
PART B. PHONETICS
I. Choose the letter A, B, C, or D to indicate the word whose underlined part differs from the
other three in pronunciation in each of the following questions.
Question 1 A. choreograph B. Christianity C. archaeology D. chivalry
Question 2 A. inaudible B. draughty C. astronaut D. launch
II. Choose the letter A, B, C, or D to indicate the word that differs from the other three in
the position of primary stress in each of the following questions.
Question 3 A. comprise B. design C. outbreak D. before
Question 4 A. fabulous B. cathedral C. practical D. hyperlink
Question 5 A. memorialize B. conservative C. pedestrian D. astrophysics
PART C. VOCABULARY AND GRAMMAR
I. Choose the best answer (A, B, C, or D) to each of the following questions.
Question 6 The school authorities __________ the child's unruly behavior on his parents' lack
of discipline.
A. attribute B. accuse C. blame D. ascribe
Question 7 He may be shy now, but he'll soon come out of his __________ when he meets
the right girl.
A. shoe B. shell C. shed D. hole
Question 8 Sales of the toy __________ sharply when the TV news reported that a child had
swallowed part of the toy and almost died.
A. dropped off B. dropped over C. took away D. turned out
Question 9 I don't think it would be wise to try to make Max change his mind about
divorcing Narnara. Well, in his place I __________ her at all.
A. would never have married B. needn't have married
C. would never marry D. must never have married
Question 10 The new Garden City is well worth __________ if you're in the area.
A. being seen B. a visit C. to visit D. the sight
Question 11 Her outgoing character contrasts __________ with that of her sister.
A. sharply B. thoroughly C. fully D. coolly
Question 12 My father __________ when he found out that I'd damaged the car.
A. hit the roof B. saw pink elephants
C.made my blood boil D. brought the house down
Question 13 You can imagine how upset I was after the closure of the magazine. Since the
first day on its staff I __________ it was my best job ever.
A. have considered B. considered C. had considered D. was considered
Question 14 I'm afraid we haven't got a spare bed. Can you __________ with a mattress on
the floor?
A. make do B. make by C. make over D. make up
Question 15 I know him by __________ but I have no idea what his name is.
A. sight B. myself C. heart D. chance
Question 16 Would you please leave us details of your address __________ forwarding any of
your mail to come?
A. as a consequence of B. for the purpose of
C. for the sake of D. by means of
Question 17 M.D. played much better than their opponents; they almost __________ the
match.
A. dominated B. eliminated C. activated D. terminated
Question 18 If you don't repay the money, we will, as a last __________, take you to court.
A. Measure B. attempt C. act D. resort
Question 19 In the middle of the blue river __________.
A. does a tiny violet flower emerge B. did a tiny violet flower emerge
C. emerged a tiny violet flower D. emerge a tiny violet flower
Question 20 She said that she would be punctual for the opening speech, __________ she
were late?
A. but what if B. how about C. and what about D. so if
Question 21 There has been a recommendation from the union leaders that the offer of 5%
__________.
A. is rejected B. has been rejected C. be rejected D. rejects
Question 22 In all __________, he failed to understand the consequences of his actions.
A. odds B. probability C. certainty D. ossibilities
Question 23 We failed to lay eyes on a tiger during our expedition, __________ film one.
A. let alone B. not to mention C. apart from D. but for
Question 24 Is it far to Stamford? - Not at all. It's only a __________ from here.
A. stone's throw B. bird's eye view
C. short cut D. step in the right direction
Question 25 The genus Equue became extinct in North America during the glacial period,
and it was not reintroduced until __________ by the Spaniniards.
A. was brought there B. brought there
C. bringing it there D. it brought there
PART D. READING
I. Read the text below and choose the word that best fits each of the blanks in the following
passage.
CROCODILES
Crocodiles see well, their eyes are equipped with three eyelids, each having a different
function. Their eyeballs slide back out of harm's way during an attack. Should they lose one of
their eight-centimeter-long teeth, a replacement is always ready. A crocodile may go through
several thousand teeth during a lifetime of over seventy years. Crocodiles cannot chew, as their
teeth are (26) __________only to penetrate and hold. These animals can attack at any (27)
__________ of the year, but they are more active in the warmer months and when in (28)
__________ of mates. Underwater, crocodiles (29) __________ their victims at the water's (30)
__________ by sensing any movement in the water. Once they have a hold on their victim, they
drag it deep into the water to (31) __________. They then crush and swallow it. Many battles
occur (32) __________ mates. About six weeks after mating, the female makes a nest, often on
a river bank, and (33) __________ about fifty eggs. She then seals the nest for protection and
also as a way of (34) __________ the temperature. After ten to twelve weeks the baby crocodiles
come out of the eggs: only about one percent of these (35) __________ it to adulthood, as
thousands die in flooding or are eaten by fish or bigger crocodiles. In an attempt to ensure a
source of healthy animals, crocodile farms have been set up, and a vast industry now exists in
crocodile skin and meat.
Question 26 A. composed B. constituted C. designed D. styled
Question 27 A. time B. phase C. month D. interval
Question 28 A. discovery B. hunt C. exploration D. search
Question 29 A. prefer B. accept C. propose D. choose
Question 30 A. border B. rim C. edge D. shore
Question 31 A. sink B. drown C. capsize D. soak
Question 32 A. against B. over C. between D. for
Question 33 A. lays B. sets C. drops D. puts
Question 34 A. dominating B. ruling C. imposing D. controlling
Question 35 A. reach B. make C. get D. arrive

II. Read the following passage and choose the letter A, B, C, or D to indicate the correct
answer to each of the following questions.
About 200 million years ago, as the Triassic Period came to a close, many species of animals
disappeared from the face of the Earth. Scientists previously believed that the series of
extinctions happened over a period of 15 to 20 million years. Recent discoveries in Nova Scotia
suggest, however, that the extinctions may have happened over a much shorter period of time,
perhaps less than 850,000 years.
Evidence for a rapid extinction of species at the end of the Triassic Period is found in the
McCoy Brook Formation along the Bay of Fundy in Nova Scotia. Fossils found in this formation
indicate a rapid disappearance of species rather than a slow and gradual change over time. One
explanation for a relatively sudden extinction at the end of the Triassic may be that a large
meteorite struck the earth at the time and is responsible for a 70 - kilometer hole nearby. If
geologists and other researchers can find evidence, such as shocked quartz in the rock
formations, that a meteorite did strike the earth, it would give more credence to the theory of
rapid Triassic extinctions. It is possible, however, that even if a rapid extinction happened in and
around Nova Scotia, it did not necessarily occur in the rest of the world.
Question 36 What is the main topic of this passage?
A. the disappearance of animal species at the end of the Triassic Period
B. evidence of a relatively sudden extinction of species
C. the possibility of an extinction happening simultaneously throughout the world
D. a meteorite hole in the Bay of Fundy in Nova Scotia
Question 37 The author uses the phrase "the face of the Earth" in paragraph 1 in order to
__________.
A. emphasize the disappearance B. focus on one part of the Earth
C. focus on one period of time D. point out the reference to land, not water
Question 38 All of the following were mentioned in the passage EXCEPT __________.
A. the extinction of late Triassic animals
B. the duration of time for the extinction
C. a large meteorite hitting the Earth 10 million years ago
D. the use of types of rock in scientific research
Question 39 Where in the passage does the author give evidence for the argument?
A. Lines 1-2 B. Lines 6-10 C. Lines 11-12 D. Lines 13-14
Question 40 According to the passage, what would give evidence that a meteorite struck the
earth?
A. a gradual change in species over time B. a change in the quartz
C. deposits in the veins of rocks D. a change in the waters of the Bay of Fundy
Question 41 Which of the following could best replace the word "struck" as used in the
passage?
A. affected B. discovered C. devastated D. hit
Question 42 Which of the following is most probably the meaning of "shocked quartz" in
the passage?
A. narrow chasms B. tiny lines C. hardened ores D. cracked minerals
Question 43 The word "it" in paragraph 2 refers to __________.
A. evidence B. an extinction C. the Earth D. a meteorite
Question 44 The word "credence” in the passage could be best replaced by __________.
A. demonstration B. elevation C. suitability D. credibility
Question 45 Which of the following best describes the author's tone?
A. aggressive B. cynical C. apologetic D. explanatory
PHẦN II: ĐỀ THI CHUYÊN BẮC NINH (TỰ LUẬN)
PART C. VOCABULARY AND GRAMMAR
I. Give the correct tense/form of the verb in each bracket.
- The huge damage (1. not report) [269584] __________ (2. cause) __________ by the recent
violent earthquake.
- The uneaten breakfast was still on the table. She (3. must/call) __________ away in a
hurry.
- The US postal service policy for check approval includes a requirement that two pieces of
identification (4. present) __________.
- Mary was sitting on the ground. She was out of breath. She (5. run) __________ all the way
here for half an hour.
- The car broke down. But for that we (6.be) __________ in time. - The man (7. sit)
__________ on the park bench said hello to the jogging woman in a friendly way. - "Eric is
really upset about losing his job."
"Well, (8. fire) __________ once myself, I can understand."
- I (9. not do) [ __________ the test for the time being, I would be playing football with you.
- I'm sure you (10. forget) __________ me by the time I am back in five years. II.
Give the correct form of the word in each bracket.
People are often put off meditation by what they see as its many mystical associations. Yet
meditation is a (11. STRAIGHT) __________ technique which merely involves sitting and
resting the mind. In addition to its (12. SIMPLE) __________, meditation offers powerful help
in the battle against stress. Hundreds of studies have shown that meditation, when (13. TAKE)
__________ in a principled way, can reduce hypertension which is related to stress in the body.
Research has (14. PROOF) __________ that certain types of meditation can substantially
decrease key stress symptoms such as anxiety and (15. IRRITABLE) __________. In fact,
those who practice meditation with any regularity see their doctors less and spend, on average,
seventy percent fewer days in hospital. They are said to have more stamina, a happier (16.
DISPOSE) __________ and even enjoy better relationships.
When you learn to meditate, your teacher will give you a (17. PERSON) __________
‘mantra' or word which you use every time you practice the technique and which is (18.
SUPPOSE) __________ chosen according to your needs. (19. INITIATE) __________ classes
are taught individually but subsequent classes usually consist of a group of students and take
place over a period of about four days. The aim is to learn how to slip into a deeper state of (20.
CONSCIOUS) __________ for twenty minutes a day. The rewards speak for themselves.
III. Each of the following sentences has one word/phrase that is not correct. Identify and
correct it.
Question 21 The vacuum tube did an important contribution to the early growth of radio and
television.
Question 22 He hopes to pursue an advance degree that may aid him in getting a job with
excellent promotion prospects.
Question 23 Since rats are destroyable and may carry diseases, many cities try to exterminate
them.
Question 24 Made up of more than 150 member countries, the organization known as the
United Nations were established after World War II to preserve international peace and security.
Question 25 Lack of animal protein in the human diet is a serious cause of the malnutrition.
PART D. READING
I. Read the text below and fill each blank with ONE suitable word.
Always a sure source of affection, my grandparents (26) __________ hugely important
figures in my life. They would shower my sisters and me with sweets, indulgences and stories,
(27) __________ tales about my parents as naughty children. When the last of (28)
[__________ died, we all wondered who would hold the family together.
People have relied on grandparents in Britain since the Industrial Revolution, (29)
__________ whole families moved into cities from the country to get work in the new factories,
taking grandmother along to look after the children. (30) __________ the fact that more
grandmothers are working now, grandparents are still the backbone of childcare in Britain. They
provide 44% of full-time care for pre-school children, which (31)__________ you wonder how
the country would manage without them.
The traditional image of a grandparent is a smiling old person surrounded by a cohort of
happy children, but this doesn't match the facts. (32) __________ we have now is the so-called
“beanpole family”, thinly stretched over several generations, with fewer family members in each
and with growing (33) __________ of single-parent families. Grandparents are getting younger
– more than 50% of grandparents have already had their first grandchild by the age 54.
For many of them, grandparenthood means juggling a job, involvement with grandchildren
and, sometimes, the care of their own parents. It is up to us to balance the demands we make on
them if we don't want to wear them (34) __________. Grandparents are (35) __________ a
valuable part of the family that we just cannot do without them.
PART E. WRITING
I. Complete the second sentence so that it has a similar meaning to the first one. Question
36 You could be arrested for not giving a breath sample to the police.
=> Refusal ____________________________________________________
Question 37 Absolute secrecy was crucial to the success of the mission.
=> ithout___________________________________________________________________
Question 38 It is very kind of you to give me a lift.
=> I appreciate___________________________________________________________
Question 39 It's sad, but the crime rate is unlikely to go down this year.
=> Sad as ________________________________________________________
Question 40 "Please don't run so fast!" Suzy begged her friend.
=> Suzy pleaded
_______________________________________________________________
II. Complete the second sentence so that it has a similar meaning to the first sentence,
using the word given. Do not change the word given.
Question 41 His irresponsible attitude is endangering his career as a doctor. (jeopardy)
=> __________________________________________________________________
Question 42 Only final-year students are allowed to use the main college car park.
(restricted)
=>___________________________________________________________________
Question 43 He could not explain why he was always late for work. (account)
=> _________________________________________________________
Question 44 She doesn't see or hear from her childhood friends any more. (lost)
=> ___________________________________________________________
Question 45 A government official leaked the story to the world press. (wind)
=> _________________________________________________________________
III. Essay Writing
"The responsibility of bringing up children should be shared equally between mother and
father." To what extent do you agree or disagree?
Write an essay of about 250 words to express your opinion, using specific details and
examples to support your answer.
TEST 7

PHẦN I: LUYỆN NGHE


Part 1: For each question, choose the correct answer. (Taken from PET trainer)
1. You will hear a boy telling his friend about a snowboarding trip. What problem did the boy
have on the trip?
A. He damaged some equipment.
B. He injured himself.
C. He became ill.
2. You will hear two friends talking about the new library at their school. The girl thinks that
A. the staff are helpful.
B. there should be more books.
C. it’s a good place to do homework.
3. You will hear two friends talking about a new clothes shop. They agree the shop would be
better if
A. the assistants were more friendly.
B. there was more choice of clothes.
C. it was in the town centre.
4. You will hear two friends talking about a new classmate. The boy thinks the new classmate
A. is very clever.
B. likes playing sports.
C. talks too much.
5. You will hear a girl talking about her big brother going away to college. How does she feel
about it?
A. pleased there’s less noise
B. surprised than she’s so sad
C. upset he’s gone so far away
6. You will hear two friends talking about playing tennis. The boy wants the girl to
A. practice with him regularly.
B. recommend a tennis coach.
C. teach him some new techniques.
Part 2: You will hear a young woman called Amanda Murillo talking to a group of college
students about Taekwondo, the Korean martial art. For questions 7–16, complete the
sentences with a word or short phrase. (Taken from FCE Test) Taking up Taekwondo
Amanda says she was getting bored with going to (7) __________ every
week. Amanda’s (8) __________ advised her to take up Taekwondo.
At the Taekwondo club, Amanda was told that size and strength was less important than (9)
__________.
Amanda was helped by the fact that she can keep her (10) __________ quite well. Amanda
says she was more (11) __________ than some of the male students. Amanda explains that the
students do some (12) __________ exercises when they have finished running.
Amanda was surprised to find how high she could (13) __________ after the initial
exercises. Amanda always wears safety equipment on her (14) __________, as well as on
her hands. The only serious injury that Amanda has had was to her (15) __________. The
next color belt that Amanda wants to get is (16) __________.

Part 3: Questions 17 – 21 (Taken from Listening for IELTS)


Complete the form below. Write NO MORE THAN TWO WORDS OR A NUMBER for
each answer.
Wright's Employment Agency
Registration form
Name: Helen SHEPARD
Address: 18 Henley Street, Mill Town
Postcode: 17 __________
Telephone: 07945 76674
Looking for 18 __________ work.
Experience:
- 19 __________ in residential children's home
- waitressing
- cleaning in hotel
Own transport? 20 __________
Availability: not night shifts, can start 21 __________
PHẦN II: (TRẮC NGHIỆM)
SECTION A. PHONETICS
I. Choose the letter A, B, C or D to indicate the word whose underlined part differs from the
other three in pronunciation in each of the following questions.
Question 1 A. informed B. impressed C. installed D. admired
Question 2 A. comment B. compose C. command D. complain
Question 3 A. reason B. threaten C. release D. season
Question 4 A. collections B. hamburgers C. benefits D. religions
Question 5 A. gather B. clothes C. author D. bathe
II. Choose the letter A, B, C or D to indicate the word that differs from the other three in the
position of main stress in each of the following questions.
Question 6 A. happy B. hobby C. region D. agree
Question 7 A. village B. summer C. decide D. mountain
Question 8 A. medicine B. addition C. endanger D. survival
Question 9 A. imagine B. horizon C. properly D. computer
Question 10 A. eliminate B. compulsory C. technology D. academic
SECTION B. VOCABULARY AND GRAMMAR
I. Choose the letter A, B, C or D to indicate the best answer to complete each of the
following sentences.
Question 11 It is going to rain. You'd better __________ an umbrella.
A. take B. taking C. took D. taken
Question 12 It's high time she __________ behaving like a child.
A. stops B. to stop C. stopped D. stop
Question 13 The bad weather caused seriously to their crop. They wish the weather
__________ good.
A. had been B. were C. would be D. will be
Question 14 [269679 I had my teeth __________ some days ago.
A. Checked B. checking C. to check D. check
Question 15 Do you remember Ha Long Bay, __________ has many beautiful caves?
A. where B. that C. which D. who
Question 16 He is always busy, so he has __________ time to relax.
A. much B. little C. few D. plenty of
Question 17 Lucy has studied English for 4 years, __________?
A. isn't she B. doesn't she C. hasn't she D. wasn't she
Question 18 She quickly became accustomed __________ his messy ways.
A. for B. to C. on D. at
Question 19 House prices in our city have __________, so it is quite hard for low income
people to buy one.
A. taken on B. gone up C. called for D. filled in
Question 20 Students are encouraged to raise questions __________ accepting opinions
without questioning.
A. in addition B. for instance C. instead of D. because of
Question 21 The little boy is asked to __________ goodbye to his grandparents before going
to school.
A. talk B. say C. tell D. speak
Question 22 We tried to make a __________ of our situation, but it wasn't really funny.
A. joke B. fun C. tease D. humor
Question 23 We are trying to get the baby into a __________ for feeding and sleeping.
A. way B. network C. routine D. path
Question 24 The economy is regarded as the key __________ of a country.
A. ingredient B. component C. feature D. factor
Question 25 I'm not sure what flavor I want - I'm still __________ up my mind.
A. turning B. making C. staying D. taking
Question 26 __________ candidates there were, the more competitive the exam was.
A. The most B. The less C. The fewest D. The
more
Question 27 - “Shall I help you with your suitcase?"
- "__________”
A. Not a chance. B. That's very kind of you.
C. I can't agree more. D. What a pity!
Question 28 - "Guess what? My first novel has just been published.” - “__________”
A. It's my pleasure. B. Congratulations!
C. Better luck next time! D. It's very kind of you.
Question 29 There is a __________ bag at the corner of the room.
A. small black plastic B. black small plastic
C. plastic small black D. plastic black small
Question 30 He couldn't perform well in his recent test __________ his anxiety.
A. though B. due to C. because D. in spite of
SECTION C. READING COMPREHENSION
I. Complete the following passage by choosing the best option A, B, C or D.
CATS
Cats of all kinds are present in the legends, religion, mythology, and history of many
different cultures. Cave paintings created by early humans display different types of wild cats
(31) __________ are now extinct, or no longer around. Cats (32) __________ to the ones kept
as pets today started showing up in artwork thousands of years ago. For example, the ancient
Egyptians believed cats were the sacred, or special animal of a goddess named Bast. They
believed that Bast often appeared as a cat, so many ancient Egyptians respected and honored cats
and kittens. (33) __________, other cultures feared cats or thought that they brought illnesses
and bad luck. Today, with millions kept as pets in homes around the world, cats have become
important members of (34) __________ families. No one knows for sure when or how cats
became very popular household pets. It's possible that people noticed how cats hunted mice and
rats, so they set food and milk out to keep the cats near their homes. This helped to prevent too
many of these rodents from coming into homes and eating people's food or (35) __________
sickness.
Question 31 A. why B. which C. who D. where
Question 32 A. close B. same C. like D. similar
Question 33 A. However B. Therefore C. For instance D. Furthermore
Question 34 A. a lot B. much C. many D. every
Question 35 A. suffering B. spreading C. treating D. experiencing
II. Read the passage carefully and choose A, B, C or D to indicate the best answer to each of
the following questions.
Social media marketing has increased due to the growing active user rates on social media
sites. For example, Facebook currently has 2.2 billion users, Twitter has 330 million active users
and Instagram has 800 million users.
One of the main uses is to interact with audiences to create awareness of the brand or
service, with the main idea of creating a two-way communication system where the audience
and/or customers can interact back; providing feedback as just one example. Social media can
be used to advertise; placing an advert on Facebook's Newsfeed, for example, can allow a vast
number of people to see it or targeting specific audiences from their usage to encourage
awareness of the product or brand. Users of social media are then able to like, share and comment
on the advert, becoming message senders as they can keep passing the advert's message on to
their friends and onwards. The use of new media put consumers on the position of spreading
opinions, sharing experience, and has shift power from organization to consumers for it allows
transparency and different opinions to be heard.
Media marketing has to keep up with all the different platforms. They also have to keep
up with the ongoing trends that are set by big influencers and draw many people’s attention.
The type of audience a business is going for will determine the social media site they use.
Question 36 Which best serves as the title for the passage?
A. Advertisements on social media B. New types of social media
C. Let listen to our customers D. Two-way communication
Question 37 The word "vast" in the second paragraph is CLOSEST in meaning to
__________.
A. enormous B. definite C. small D. certain
Question 38 According to the second paragraph, users can do the followings with the adverts
on social media EXCEPT __________.
A. like B. share C. pass D. adjust
Question 39 What is the benefit of creating a two-way communication system?
A. It allows customers to interact back B. It provides examples for users
C. Users can create their own adverts D. More and more people visit the sites
Question 40 According to the third paragraph, what do media marketing have to keep up with?
A. The trends for a few groups of people
B. The new trends set by every normal user
C. All the various platforms
D. Other types of media
III. Read the passage carefully and choose A, B, C or D to indicate the best answer to each
of the following questions.
Right now, the biggest source of energy in the world is fossil fuel. Fossil fuels are oil, gas,
and coal. More than 80 percent of the world's energy comes from fossil fuel. There are many
problems with fossil fuel. One problem is that when fossil fuel is burned, it pollutes the air. Also,
when we take fossil fuel from the Earth, we often cause a lot of damage. Another problem is that
we are running out of it. That is why we need new sources of energy. A big source of energy for
many countries is nuclear power. Thirty-one countries use nuclear power.
Nuclear power has some advantages. First of all, we can't run out of nuclear power. Nuclear
power does not make the air dirty. Also, if a country has nuclear power, it doesn't need to buy
as much as oil from other countries.
However, there are also a lot of problems that come with nuclear power. For example,
nuclear accidents are very serious. In 1986, there was a nuclear accident in Ukraine and about
4,000 people got sick and died. In 2011, there was another very serious nuclear accident in Japan.
Japan is now trying to clean up the nuclear waste from the accident.
Many people don't want nuclear power in their countries. They say that it is not safe. A lot
of people want their countries to use safer and cleaner ways to get electricity. Although many
people hate nuclear energy, more and more countries are using it. One reason for this is that the
world is using more and more energy. We just don't have enough fossil fuel. However, if we use
nuclear power, then we may have more serious problems in the future.
Question 41 Which of the following is NOT true about fossil fuel?
A. It can pollute the air.
B. We don't use much of it.
C. Its sources are limited.
D. Exploiting fossil fuel damages the environment.
Question 42 What does the word "it" in paragraph I refer to?
A. damage B. fossil fuel C. air pollution D. nuclear power
Question 43 All of the following are true about nuclear accidents EXCEPT that __________.
A. they have been very serious
B. their effects can last many years
C. it takes short time to clean up the nuclear waste from the accident
D. there were serious nuclear accidents in Ukraine in 1986 and in Japan in 2011
Question 44 The phrase "clean up" in paragraph 3 is closest in meaning to __________.
A. block B. trap C. disappear D. remove
Question 45 Which of the following statements can be inferred from the passage?
A. Some governments are wrong when they are using nuclear energy.
B. We can continue using nuclear energy until there is an accident.
C. Nuclear power has both advantages and disadvantages.
D. Many countries stop using nuclear energy because many people hate it.
PHẦN II: (TỰ LUẬN)
SECTION B. VOCABULARY AND GRAMMAR
I. Complete each of the following sentences with the correct form or tense of the verbs in
brackets.
Question 1 My brother and sister __________ (talk) about something when I walked into the
room.
Question 2 She is the first student __________ (accept) to that famous university.
Question 3 If nobody had reminded him, he __________ (forget) the date of the meeting.
Question 4 They always get someone __________ (look) after their house when they go away
for a few days.
Question 5 His English has improved a lot since he __________ (take) an English course by
Mr. John.
Question 6 By next month, he __________ (work) as an assistant in this company for one
year.
Question 7 She dreams of __________ (choose) to represent England in the Olympics.
Question 8 I am extremely tired because I __________ (drive) for over eight hours.
Question 9 Doctors advise that people __________ (have) enough vitamin C.
Question 10 There is no point __________ (feel) sorry for what happened.
II. Use the correct form of the words in capital to complete each of the following sentences.
Question 11 We can depend on him as he is a very __________ person. RELY
Question 12 These workers got a bonus because they did their jobs __________. EFFECT
Question 13 Our electricity bills are high, so we must start to __________. ECONOMY
Question 14 The weather in Britain is __________. so we don't know whether it's sunny or
rainy after five minutes. PREDICT
Question 15 When Jack came home, everything had been __________ done. NICE
Question 16 [269726 You are __________ to choose how to spend the evening.FREEDOM
Question 17 Our house is __________ by several burglar alarms.PROTECT
Question 18 He is one of the most famous __________ in the world. NVIRONMENT
Question 19 We'll __________ our room with roses. BEAUTY
Question 20 There is no __________ to this problem. SOLVE
SECTION C. READING COMPREHENSION
I. Complete the passage by filling in each of the following with ONE suitable word.
DICTIONARIES
Samuel Dictionaries are among the most important tools of self-education. (21)
__________Samuel Johnson wrote his influential English dictionary in the eighteenth century,
the work kept him busy for seven years. At the end of that period he (22) __________ written
the meanings of over forty thousand words. Most modern dictionaries require a (23) __________
deal less time and effort to write because writers often use earlier dictionaries (24) __________
a source of reference.
Nowadays, most dictionaries are put together by teams of writers, or lexicographers.
Sometimes they need to work together in meetings; at other times they work independently of
(25) __________ other, on different parts of the dictionary.
(26) __________ one time, the starting point for deciding on which words to include used
to be the lexicographer's own knowledge. These days, teams (27) __________ use of a large
collection of examples of not only writing but also everyday speech, which is known as a corpus.
Teams also refer books and articles about language as (28) __________ as asking experts
in particular subjects about the more specialized words. Finally, ordinary people are asked to say
what they think about the (29) __________ the words are defined and (30) __________ they
find the examples provided helpful or not.
SECTION D. WRITING
I. Each of the following sentences has ONE mistake. Identify the incorrect word/phrase then
write the correct answer on the answer sheet.
Question 31 My French friend finds driving on the left difficultly.
Question 32 It was so a sunny day that none of us wanted to do any work.
Question 33 I'd rather you don't make any noise because I am tired.
Question 34 Neither his explanation nor the examples is clear.
Question 35 He was very proud in making a great discovery.
II. Complete the second sentence so that it has a similar meaning to the first.
Question 36 "It certainly wasn't me who broke the glass." said Lan.
=> Lan
denied____________________________________________________________________
Question 37 Although she was very tired, she agreed to help her child with his homework.
=> Tired
______________________________________________________________________
Question 38 We couldn't cook dinner until all the guests had left.
=>
Only______________________________________________________________________
Question 39 The last time I saw Rose was three years ago.
=> I haven't____________________________________________
Question 40 They couldn't go on their trip because of the thick fog.
=> The thick fog prevented ___________________________________________
Question 41 It was wrong of you not to book tickets for the film in advance.
=> You should _______________________________________________________________.
Question 42 If they translated this story into English, it would be read more widely.
=> Were they
____________________________________________________________________________
___________
Question 43 I have a good relationship with my neighbors.
=> I get on_________________________________________________________________
Question 44 "Who did you have lunch with yesterday, John?" asked Tom.
=> Tom
asked___________________________________________________________________
Question 45 They believe that Mike will get promotion for his dedication to the company.
=> Mike_____________________________________________________________________
III. Use the suggested words to write complete sentences. Make changes if necessary.
Question 46 You/ always/ be/ careful/ when/ you/ cross/road/.
Question 47 It/ essential/ that/ every child/ have/ same/ educational opportunities.
Question 49 Hung/ only/ play/ guitar/ well/ also/ sing/ beautifully.
Question 50 I/ very impressed/ all the work/ you/ done/ your house.
IV. Complete the second sentence so that it has a similar meaning to the first sentence,
using the word given. DO NOT CHANGE THE GIVEN WORD. You must use between two
and five words including the word given.
Question 51 It will be nice to see you again at the party. (FORWARD)
=> I’m ______________________________________________ you again at the party.
Question 52 My views on this subject are exactly the same as yours. (DIFFERENCE)
=> There is no _____________________________________ my views on this subject and
yours.
Question 53 I haven't got enough money to go on holiday this year. (AFFORD)
=> I wish __________________________________________________ go on holiday this
year.
Question 54 We went swimming despite the heavy rain. (EVEN)
=> We went swimming
_____________________________________________ heavily.
Question 55 Let's me try to finish this exercise. (CHANCE)
=> Give me ______________________________________________________ this exercise.
TEST 8
PHẦN I: LUYỆN NGHE
Part 1. For questions 1-9, listen to a conversation in a tourist information office in Guernsey
and decide whether these statements are True (T) or False (F). Write your answers in the
corresponding numbered boxes provided.
1. There is a concert at the old castle on Saturday night.
2. St James concert hall is an ideal place to learn about the history of the island.
3. The woman says that Hauteville House was the home of a famous writer. 4.
The guided walk in St Peter Port starts at 5.00 and lasts 1.5 hours.
5. To join the guided walk, visitors must buy a ticket from the information office.
6. The man plans to take part in the Autumn Walking Festival.
7. The name Tennerfest is related to the price offered.
8. Tennerfest will start in six weeks’ time.
9. The man is not interested in visiting the island of Jersey.
Part 2. For questions 10-15, listen to a radio interview with the comedian Brain Conley, who
does a considerable amount of work in panto, a type of family-friendly show which is popular
at Christmas and choose the correct answer A, B or C which fits best according to what you
hear. Write your answers in the corresponding numbered boxes provided.
10. Brian likes Birmingham because
A. it has provided him with a steady income.
B. it is where he grew up.
C. it was where he first became famous
11. When they discuss children’s participation in panto, Brian says that
A. he prefers children not to come up on stage.
B. it’s important to get the parents’ permission if a child wants to participate.
C. he thinks that children’s participation adds a certain quality to the show.
12. According to Brian, the advantage of panto is that
A. all the actors can change their lines to reflect current events.
B. the qualities required in panto match his talents.
C. it gives him the chance to play a comedy character.
13. Brian became involved in comedy because
A. he had wanted to do it since he was a child.
B. his friends at school encouraged him to do it.
C. he found he could earn more by doing comedy.
14. What does Brian say about providing comedy for corporate events?
A. It is easy because the audience has had a lot to drink.
B. He has learnt how to respond to comments from the audience.
C. It’s the only way for many comedians to find work.
15. What does Brian feel with regards to nerves?
A. He agrees with a comment someone made early in his career
B. He no longer feels nervous because he is more experienced
C. The extent of his nerves has changed over the years.
Part 3. For questions 16-25, listen to a talk about the history of the electric guitar and supply
the blanks with the missing information. Write NO MORE THAN TWO WORDS taken from
the recording for each answer in the space provided.
DATE MAKER NAME OF FEATURES
GUITAR

1890s Orville Gibson no name similar in shape to a


16.__________

1925 Jogn Dopyera The National made of metal, good for


Guitar playing 17.__________

1930s C.F. Martin The Dreadnought strings made of 18.__________


Company

1931 George The used two 20.__________shaped like


Beauchamp 19.__________ horseshoes to increase sound

1935 Adolph The Rickenbacker made from 21.__________


Rickenbacker Electro Spanish

1941 Les Paul The Log the first to be completely


22.__________

1950 Leo Fender The Fender its simplicity made it ideal for
Broadcaster 23.__________

1951 Leo Fender The easy to carry around


24.__________

1952 Ted McCarty The Gibson Les 25.__________in colour


Paul

1954 Leo Fender The Fender double cutaway design


Stratocaster

PHẦN II: PHẦN TRẮC NGHIỆM


Mark the letter A, B, C or D on your answer sheet to indicate the correct answer to each of
the following questions.
Question 11 She came back with an answer as quick as a/an __________.
A. twinkling B. lightning C. flash D. express
Question 12 The temperature __________ takes place varies widely from material to
material.
A. which melting B. at which melting C. which they melt D. at which they melt
Question 13 David has not seen Linda for 15 years and has __________ of her address.
A. no idea B. nothing C. no thought D. no mind
Question 14 The only means of __________ to the station is through a dark subway.
A. arrival B. admission C. access D. approaching
Question 15 __________ I’d like to help you out. I’m afraid I just haven’t got any spare money
at the moment.
A. Even B. Despite C. Much as D. Try as
Question 16 As a citizen, it is normal to __________ to the laws and rules made by the
society.
A. resist B. conform C. hinder D. obey
Question 17 Everyone is hoping and praying that __________ peace will eventually come to
the area.
A. irrevocable B. lasting C. durable D. ongoing
Question 18 Shortly after receiving her gold medal, the Olympic champion answered questions
at a press __________.
A. conference B. assembly C. discussion D. seminar
Question 19 I __________ doubt whether he will actually carry out his threat.
A. seriously B. absolutely C. highly D. deeply
Question 20 If Harold __________ with his piano playing, he can eventually reach concert
standard.
A. sustains B. perseveres C. maintains D. survives
Question 21 By then I __________ my driving test. I hope.
A. pass B. will have passed C. have passed D. will be passed
Question 22 My father didn’t like coffee and __________ did my mother.
A. either B. none C. so D. neither
Question 23 Is it true that this country produces more oil than __________?
A. any countries else B. any other country
C. any other countries D. any country else
Question 24 He is __________ by many people for the money he has helped raise for
charity.
A. admired B. regarded C. considered D. approved
Question 25 Language is so __________ woven into human experience that it is scarcely
possible to imagine life without it.
A. rigidly B. tautly C. tightly D. stiffly
Mark the letter A, B, C or D on your answer sheet to indicate the correct preposition or
adverb particle to complete each of the following questions.
Question 26 If you keep passing notes in class, you will get into trouble __________ the
teacher.
A. about B. for C. from D. with
Question 27 __________ second thoughts, I’d rather not go out tonight.
A. On B. Under C. With D. In
Question 28 Someone __________ the audience had a heart attack during the play.
A. at B. in C. among D. inside
Question 29 In Turkey, most of the iron ore comes from the central Anatolia and is
__________ good quality.
A. of B. about C. in D. out of
Question 30 Brian was __________ pressure from his boss to finish the project by the
deadline.
A. by B. with C. over D. under
Question 31 The fishing grounds __________ the coasts of Norway are among some of the
best managed in Europe.
A. at B. on C. off D. over
Question 32 Were there many people __________ board of the train when it derailed?
A. on B. in C. at D. off
Question 33 She has a lot of experience __________ dealing with difficult situations.
A. with B. in C. on D. from
Question 34 Society tries to deal with young offenders __________ a variety of ways.
A. to B. in C. from D. by
Question 35 The difference between a dollar bill and a counterfeit one can be difficult to detect
__________ the naked eye.
A. with B. by C. under D. beyond
Mark the letter A, B, C or D on your answer sheet to indicate the underlined part that needs
correction in each of the following questions.
Question 36 Although this car appears to be manufactured by a different
A B C
company, it has the same body style, size, and perform as that one.
D
Question 37 In the human body, blood flows from a heart through the arteries,
A B C
and it returns through the veins.
D
Question 38 In a determined drive to pare its debt, Time Warner is launching a
A B
stock offering plan that would potentially raise $2.8 billion.
C D
Question 39 Some of the people were standing in the street watched the firework
A B
display while others were singing a song.
C D
Question 40The first of two such investigation requires the students to read
A B
continuously over a period of four hours.
C D
Question 41 [269479]: The closer it gets to December 21st, the first day of winter, the short
A B C
the days become.
D
Question 42 Nuclear energy, despite its early promise as a source of electrical power,
A B C
is still insignificant in compared with older and safer energy sources.
D
Question 43 Most doctors agree that it is not good for patience to lie in bed without
A B C D
exercising.
Question 44 Beaten by Holyfield, his position was not highly appreciated in
A B C D
world boxing.
Question 45 There are many different ways of comparing the economy of one
A B
nation with those of another.
C D
Mark the letter A, B, C or D on your answer sheet to indicate the correct phrasal verb to
complete each of the following sentences.
Question 46 I need more time to consider the offer before I __________ my mind.
A. give up B. make up C. go up D. take up

Question 47 You look really tired. You should __________ a week’s holiday, I think.

A. do with B. make for C. make with D. pass for

Question 48 I __________ this old book while I was tidying my room.

A. came up B. came across C. came out D. came over


Question 49 Having seen a sharp bend ahead, Roger pressed hand on the brake pedal in order
to __________.
A. slow down B. speed up C. come by D. pass over
Question 50 I think you should __________ a rain coat in case it rains.
A. bring up B. bring about C. bring along D. bring forward
Question 51 Ivy was really surprised when Gary __________ at her front door because she
hadn’t seen him for 10 years.
A. showed up B. pulled in C. turned in D. jumped on
Question 52 I couldn’t hear what Andy was trying to tell me over the phone, so I told him to
__________.
A. get up B. do up C. speak up D. call up
Question 53 Peter and Mary had to __________ their wedding until the following year due to
the outbreak of the pandemic.
A.put off B. put on C. put away D. put aside
Read the following passage and mark the letter A, B, C or D on your answer sheet to
indicate the correct word that best fits each of the numbered blanks from 54 to 63. NOISE
POLLUTION
Noise is more than a mere nuisance. At certain levels and durations of exposure, it can cause
physical damage to the eardrum, and (54)__________ in temporary or permanent hearing loss.
In addition to causing hearing loss, (55)__________ noise exposure can also raise blood pressure
and pulse (56)__________, cause irritability, anxiety, and mental fatigue, and interfere with
sleep, recreation, and personal communication. Noise pollution control is, therefore,
(57)__________ importance in the workplace and in the community.
Noise effects can be (58)__________ by a number of techniques, for example, increasing
the distance or blocking the path between the noise source and the recipient, reducing noise
levels at the source, and (59)__________ recipients with earplugs or earmuffs. Increasing path
distance is very effective because, as a sound wave spreads outward from the source, the fixed
(60)__________ of energy in the wave is dissipated over an ever-expanding wave front. Path
barriers (61)__________ walls, ceilings, and floors can be effective by absorbing as well as
reflecting sound energy. Special earmuffs are (62)__________ to protect industrial and
construction workers.
The best way to prevent noise pollution is to reduce the sound levels at the source, for instance,
by improving design, muffling machinery and engines, and probably (63)__________ and
lubricating machinery to reduce vibrations.
Question 54 A. effect B. bring C. result D. create
Question 55 A. excess B. excessively C. excessive D. exceedingly
Question 56 A. ranks B. points C. levels D. rates
Question 57 A. at B. within C. for D. of
Question 58 A. done B. minimized C. brought D. created
Question 59 A. protects B. protect C. protected D. protecting
Question 60 A. kind B. number C. type D. amount
Question 61 A. as B. or C. such as D. like
Question 62 A. ready B. accessible C. enough D. available
Question 63 A. remaining B. obtaining C. achieving D. maintaining
Read the following passage and mark the letter A, B, C or D on your answer sheet to
indicate the correct answer to each of the questions from 64 to 70.
HOW I FOUND MY TRUE VOICE
As an interpreter, Suzanne Glass could speak only for others – but the work provided
terrific material for her first novel.
“No, no, no! You’ve got to get away from this or you’re going to lose it.” The voice
reverberating in my head was my own. I was at an international conference. My throat was killing
me and my headphones were pinching. I had just been interpreting a speaker whose last words
had been: “We must take very seriously the standardization of the length of cucumbers and the
size of tomatoes.” You can’t afford to have your own thoughts when you’re interpreting
simultaneously, so, of course, I missed the speaker’s next sentence and lost his train of thought.
Sitting in a darkened booth at the back of a huge conference hall, I was thrown. Fortunately, my
colleague grabbed my microphone and took over.
This high-pressure, high-output work was not quite the dream profession I had hoped for.
Although I had fun with it in the beginning – occasionally being among the first to hear of
medical and political breakthroughs would be exciting for any 25-year-old – I realized that this
was a job in which I would never be able to find my own voice. I had always known that words
would be my life in one form or another. My mother thought she’d given birth to an alien when
I began to talk at the age of seven months. That momentous day, she had placed my playpen in
the hallway and gone into the bedroom. In imitation of the words she had repeated to me again
and again, I apparently called out towards the bedroom door: “I see you. I see you.” I was already
in training for a career as a professional parrot.
But how mistaken I was to think that international interpreting would be glamorous. The
speaker rarely stops to think that there’s someone at the back of the room, listening to his words,
absorbing their meaning, and converting them into another language at the same time. Often I
was confronted with a droner, a whisperer or a mumbler through my headphones. The mumblers
were the worst. Most of the time, an interpreter is thought of as a machine – a funnel, a conduit,
which, I suppose, is precisely what we are. Sometimes, when those we are translating for hear
us cough or sneeze, or turn round and look at us behind the smoky glass of the booth, I think
they are surprised to see that we are actually alive. Ironically, part of the secret of interpreting is
non-verbal communication. You have to sense when your partner is tired, and offer to take over.
At the same time, you have to be careful not to cut him short or hog the microphone. Interpreters
can be a bit like actors: they like to show off. You do develop friendships when you are working
in such close proximity, but there’s a huge amount of competitiveness among interpreters. They
check on each other and sometimes even count each other’s mistranslations.
Translating other people’s ideas prevented me from feeling involved and creative as an
interpreter. Actually, you can’t be a creative interpreter. It’s a contradiction in terms. Sometimes,
when I disagreed with a speaker, I wanted to rip off my headphones, jump up and run out of the
booth, shouting: “Rubbish. Rubbish. You’re talking a lot of nonsense, and this is what I think
about it.” Instead, I had to sit there and regurgitate opinions in violent contradiction with my
own. Sometimes, I’d get my revenge by playing games with the speaker’s tone of voice. If he
was being serious, I’d make him sound jocular. If he was being light-hearted, I’d make him
sound earnest.
Eventually, I wanted to find a career where my own words would matter and where my own
voice would be heard. So, to redress the balance, I decided to write a novel. While I was writing
it, I did go back and interpret at a few conferences to get inside the head of Dominique, my main
character. At first, I was a little rusty and a couple of the delegates turned round to glare at me,
but after twenty minutes, I was back into it, playing that old game of mental gymnastics.
Interpreting is like learning to turn somersaults: you never forget how to do it. But for me, sitting
in the booth had a ghost-like quality to it – as though I had gone back into a past life – a life that
belonged to the time before I found my own voice.
Question 64 In the first paragraph, the writer says she discovered that __________.
A. there were some subjects she had no interest in dealing with
B. the standard of her work as an interpreter was getting worse
C. she would no longer understand subjects she had previously covered
D. her mind was wondering when she should have been doing her job
Question 65 The word “simultaneously” in paragraph 1 is closest in meaning to __________.
A. once and for all B. all at once C. all gain D. once too often
Question 66 What does the writer say about being an interpreter in the second paragraph?
A. It was what she had wanted to be ever since she was a small child.
B. It turned out to be more challenging than she had anticipated.
C. It was the kind of job her parents had always expected her to do.
D. It gave her access to important information before other people.
Question 67 What does the writer say about speakers she interpreted for?
A. Some of them had a tendency to get irritated with interpreters.
B. Some of them made little attempt to use their own language correctly.
C. She particularly disliked those she struggled to hear properly.
D. They usually had the wrong idea about the function on interpreters.
Question 68 The writer says that when she returned to interpreting, __________.
A. she changed her ideas about the main character in her novel
B. she did not start off very well
C. she thought that two of the delegates recognized her
D. she briefly wished she had not given it up
Question 69 The word “to glare” in paragraph 6 is closest in meaning to__________.
A. despise B. wonder C. to caress D. to glower
Question 70 What is the writer’s main point in the article as a whole?
A. Most interpreters eventually become disillusioned with the work.
B. It is not always a good idea to go into a profession because it looks glamorous.
C. Being an interpreter did not allow her to satisfy her need to be creative.
D. Most interpreters would actually like to do something more creative.
PHẦN TỰ LUẬN
I. Use the word given in capitals at the end of each line to form a word that fits in the space.
Question 1 Sales forecasts indicate a poor __________ for the clothing industry. (LOOK)
Question 2 Tom spoke __________ because he was so tired. (BREATH)
Question 3 I did everything I could to make their stay enjoyable, but they weren’t at all
__________. (APPRECIATE)
Question 4 The best way to solve this dispute is to find a neutral, __________ third party and
follow his or her suggestions. (INTEREST)
Question 5 His performance in the match today __________ his reputation as a great player.
(LIE)
II. Read the following passage and fill in each of the blanks with ONE suitable word.
Experts say that if you feel drowsy during the day, even during boring activities, you haven’t
had (6) __________ sleep. If you routinely fall asleep within five minutes of lying down, you
probably have severe sleep deprivation, possibly even a sleep disorder. Microsleeps, or very
brief episodes of sleep in an otherwise awake person, are another mark of sleep deprivation. In
many cases, people are not aware that they are experiencing (7) __________. The widespread
practice of “burning the candle at both ends” in Western industrialized societies has (8)
_________ so much sleep deprivation that what is really abnormal sleepiness is now almost the
norm.
Many studies (9) __________ it clear that sleep deprivation is dangerous. Sleep deprived
people who are tested by using a driving simulator or by performing a hand-eye coordination
task perform as badly as or worse than those (10) __________ are intoxicated. Sleep deprivation
also magnifies alcohol’s effects (11) __________ the body, so as fatigued person who drinks
will become (12) __________ more impaired than someone who is well rested. Driver fatigue
is responsible for an estimated 100,000 motor vehicle accidents and 1,500 deaths each year,
according to the National Highway Traffic Safety Administration. Since (13) __________ is the
brain’s last step before falling asleep, driving while drowsy can – and often does – lead to
disaster. Caffeine and other stimulants cannot overcome the (14) __________ of severe sleep
deprivation. The National Sleep Foundation says that if you have trouble keeping your eyes
focused, if you cannot stop yawning, or if you can’t remember (15) __________ the last few
miles, you are probably too drowsy to drive safely.
III. Choose the most suitable heading for each section from the list of headings (A-I) below.
Write the appropriate letters (A-I) in the space provided after questions 16-20. N.B. There are
more headings than sections, so you will not use all of them.
List of Headings

A. Species protected by tracking


B. Researchers go deeper with innovation
C. Unravel the dwindling of species
D. Mapping ocean highway
E. Functions of satellites in tracking
F. Tagging for tracking
G. New technique facilitating fishery
H. Black box of marine biology
I. Stratified ocean highway
Example: Section 1: B
16. Section 2:__________ 17. Section 3:__________ 18. Section 4:__________

19. Section 5:__________ 20. Section 6:__________

From Black Box to Blue Box


Section 1
The American Association for the Advancement of Science (AAAS) has just held its
annual meeting. One highlight was a session on new techniques for tracking marine animals.
Making a living as a fisherman has never been easy. With the continual decline in fish stocks
currently under way, it is becoming an even harder way to grind out a living. And it is not only
fish that are disappearing, but marine fauna generally. In the past 20 years, for example, 90%
of leatherback turtles and large predatory fish, such as sharks, have disappeared.
Section 2
Where and how this is happening has been difficult to say, since the ocean is something of
a black box. Things go in, and things come out, but what happens in between is hard to unravel.
According to researchers presenting their work at the AAAS meeting in Seattle, Washington,
this is now changing. Today, when many marine biologists swig their morning coffee and
download their messages, they receive special e-mails from their research subjects. These
messages, relayed by a satellite, tell them exactly where their animals have been. This has been
made possible thanks to advances in underwater electronic tagging, and it is causing a revolution
in marine biology.
One of the leading researchers in oceanic tagging is Barbara Block of Hopkins Marine
Station in Pacific Grove, California. She tags bluefin tuna, which are commercially valuable
animals that can reach 680 kg (1,500lb) in weight, and swim at speeds of up to 80kph (50mph).
So far, her group has tagged around 700 bluefin. Many of the tags are surgically implanted, a
tricky thing to do while on board a moving boat. These tags archive their data in memory chips,
and are eventually recovered when a fish is caught and butchered. (The tags carry a healthy
reward). Other tags, though, are fastened to the outside of a fish, and pop off at a pre-programmed
time and date. They then broadcast their results to a satellite. Dr. Block’s work has shown that
blue-fin can migrate thousands of kilometres across the Atlantic, ignoring boundaries that have
been set to protect stocks in the western Atlantic.
Section 3
Tagging is also helping David Welch, head of the Canadian government’s salmon
programme, to find out where and why large numbers of the fish are vanishing. He uses small
acoustic tags (the size of a large multivitamin capsule) that are sewn into the body cavities of
salmon. These tags broadcast their signals to microphones on the seabed.
Dr. Welch can now track where an individual salmon spends its life and watch trends in an
entire population. He was surprised to find that most salmon do not die as they leave the river
and enter the sea, as previously believed. And he is finding that climatic fluctuations play an
important role in determining population.
Dr. Welch and his colleagues are planning to install a system of microphones stretching
from the coast of Washington State to southeastern Alaska. This could follow the movements of
some 250,000 fish – collecting data on their direction of travel, speed, depth and position. If that
works, the plan is to extend the system from Baja California in Mexico to the Bering Sea – a
project that would involve about 1,000 underwater tracking stations.
Section 4
Meanwhile, Andrew Read, a marine biologist at Duke University in North Carolina, is
following 45 tagged loggerhead turtles. These animals must come to the surface to breathe.
When they do so, the tags (which are glued to their shells) talk to the nearest convenient satellite.
Dr. Read told the meeting that the tracking data he collects are now available online, to
allow fishermen to follow the movements of turtles and, if they wish, to modify the deployment
of their nets accordingly. Bill Foster, a fisherman from Hatteras, North Carolina, and Dr. Read,
proposed the project because the Pamlico Sound near Hatteras was closed to large-mesh gill nets
(which are dragged behind a boat like a curtain) for four months a year because too many turtles
were being caught by accident. Now, the fishermen are helping the researchers, and attaching
tags to healthy turtles that are accidentally caught in their nets.
Section 5
Together, all this work is beginning to fill in the map of marine ‘high ways’ used by
particular species, and their preferred habitats. It is also showing where particular animals prefer
to stay close to the surface, and where they prefer deeper waters. As in the case of Dr. Read’s
turtles, this is helping scientists to devise ways of protecting rare species in an efficient manner,
without interfering too much with the exploitation of common ones.
Larry Crowder, also at Duke University, has overlaid maps of marine highways for
loggerhead and leatherback turtles in the Pacific onto those of ‘longline’ fisheries, in which
people catch prey on fishing lines that are several kilometres long. Turtles often take the bait on
the hooks that these lines carry. Dr. Crowder wants to identify the places of greatest danger to
these turtles, in the hope that such places will be considered for protection. This need not, he
says, mean a ban on fishing, but rather the use of different hooks, and other sorts of gear that are
less damaging to turtles. It also turns out that turtles spend 90% of their time within 40 metres
of the surface, so setting hooks deeper than this would reduce the chance of catching them
accidentally.
Section 6
Conservationists are now pushing the notion of ‘ocean zoning’. Like the land, parts of the
sea – such as turtle highways – would be defined as sensitive, and subject to restrictions on how
extractive industries operate. If this idea is ever to work, tagging data will be crucial. And
because tagging data come in continually, this could mean that sensitive areas in the ocean could
be flexible, changing in both time and space. Enforcing such zones might be difficult. But it
would help fish, and other marine fauna, breathe a bit easier. And careful management might
leave the fishermen on top as well.
Question 16 Question 17 Question 18 Question 19 Question 20 IV. Finish the second
sentence in such a way that its meaning is similar to that of the original one.
Question 21 The tourists were unharmed after the train crash.
↳ The tourists were none _____________________________________________________.
Question 22 Fancy you and I meeting in the middle of Africa like this!
↳ It’s really odd ____________________________________________________________!
Question 23 You must remember to lock the drawer whatever you do.
↳ On
__________________________________________________________________________.
Question 24 We can start at either 7 a.m or 8 a.m.
↳ It makes _____________________________________________________.
Question 25 Could you possibly get here before lunch?
↳ Is there ___________________________________________________________?
V. Rewrite each of the sentences below in such a way that its meaning is similar to that of the
original one, using the word given in brackets. Do not change this word in any way.
Question 26 She wasn’t very big but she was very strong. (LACKED)
↳ What __________________________________________________________.
Question 27 Peter made absolutely sure nothing would go wrong when he planned the
expedition. (CHANCE)
↳ Peter left __________________________________________________________.
Question 28 The audience didn’t enjoy his performance. (DOWN)
↳ His performance didn’t _____________________________________________________.
Question 29 As far as I am concerned, the project is nearly completed. (NEARING)
↳ To the ______________________________________________________________.
Question 30 David cannot possibly borrow the car tonight. (QUESTION)
↳ It’s __________________________________________________________.
TEST 9

PHẦN I: LUYỆN NGHE


Part 1. For questions 1-9, listen to a radio interview with a ghost hunter called Carlene
Belfort and decide whether these statements are True (T) or False (F). Write your answers in
the corresponding numbered boxes provided.
1. Carlene’s parents encouraged her to become a professional ghost hunter. 2.
She believes a ghost hunter needs to possess a special power.
3. She mostly works for people who want reassurance.
4. Whenever ghosts are present, she feels them touching her hair.
5. There was one time when evils made her camera stop working.
6. She is most afraid when things move on their own accord.
7. She doesn’t understand why some scientists do not believe her.
8. She wants skeptics to experience a ghost hunt for themselves.
9. She feels she is providing a real service.
Part 2. For questions 10-15, listen to a recording about someone giving advice on how to ask
their boss for a pay rise and choose the correct answer A, B or C which fits best according to
what you hear. Write your answers in the corresponding numbered boxes provided.
10. The key factor when asking for a pay rise is
A. voicing your demands in a convincing way.
B. making it clear you feel undervalued.
C. proving you are an asset in the business.
D. comparing yourself to the rest of the staff.
11. If you have any failings, you should
A. check that no one knows about them.
B. put them right gradually so that it is not too obvious.
C. accentuate your strengths, such as punctuality.
D. make sure your boss likes you as a person.
12. When preparing what to say in your salary negotiation
A. put yourself in your superior's shoes.
B. do not forget that you really need that extra money.
C. make a list of all the points in your favor.
D. focus on what you can do for the company in the future.
13. What should you do if your boss raises objections to your pay rise?
A. pre-empt them by raising them yourself and giving a counterargument
B. make sure you can quote company rules to him or her
C. appreciate that your boss is only doing his or her job
D. accept any offer as it is better than nothing
14. During salary negotiations, it is important to
A. mention that the company is very successful.
B. ensure your boss is aware that you are taking these negotiations seriously.
C. arrange to see your boss early in the day when he or she is fresh.
D. try not to put your boss in an awkward position.
15. What should you do if you do not get a pay rise or as much as you wanted?
A. be prepared for a long drawn-out conflict
B. know that you might have to resign as a matter of principle
C. either have an alternative or ask for constructive criticism
D. either get a colleague to back you up or talk to your boss again soon
Part 3. For questions 16-25, listen to a lecture about dust storms and supply the blanks with
the missing information. Write NO MORE THAN THREE WORDS taken from the recording for
each answer in the space provided.
Main focus of lecture: the impact of 16__________ on the occurrence of dust storms. •
Two main types of impact:
A) break up the ground surface, e.g. off-road vehicle use
B) remove protective plants, e.g. 17__________ and__________
Name of area Details

USA ‘dust bowl Caused by mismanagement of farmland


Decade renamed the 18__________

West Africa Steady rise in dust storms over a 20-year period

Arizona Worst dust clouds arise from 19__________


Dust deposits are hazardous to 20__________

Sahara Increased wind erosion has occurred along with long-term 21__________

Intensive 22__________ in Central Asian Republics


Drop in water in major tributaries
Total volume of water in lake reduced by 23__________
Increase in wind-blown material
Lake has become more 24__________
Serious effects on 25__________nearby
PHẦN II: ĐỀ THI (TRẮC NGHIỆM)
SECTION I. GRAMMAR AND LEXICAL ITEMS
Part 1. Choose the correct word or phrase to complete each sentence. Write your answer A, B,
C or D in the numbered boxes.
Question 1 If only he __________ me a chance, I could have told him the truth.
A. could give B. has given C. gave D. had given
Question 2 She insisted that the report __________ as his source of information.
A. not to mention B. not mentioning C. not be mentioned D. not mention
Question 3 Hardly __________ can find the answer to this complex situation.
A. no one B. someone C. anyone D. everyone
Question 4 Stop fighting you two, shake hands and __________ your peace with each other!
A.set B. do C. bring D. make
Question 5 She seems to be angry with the whole world. She's got a chip __________.
A.on her shoulder B. in her bonnet C. under her hat D. between the ears
Question 6 __________ little we may like it, old age comes to most of us.
A. In spite of B. So C. As D. However
Question 7 A number of films were not shown to the public because of government
__________.
A.inhibition B. censorship C. omission D. compensation
Question 8 Is this __________ difficult exercise of the two that the teacher assigned
yesterday?
A.more B. the more C. most D. the most
Question 9 John and Mary are talking about burning trash.
- John: "Burning trash may have negative effects on the environment.”
- Mary: “__________. It causes air pollution.”
A. Thanks, that's fine B. You're wrong
C. There's no doubt about it D. I don't quite agree
Question 10 She is so absent-minded that she __________ her cellphone three times.
A. has lost B. loses C. was losing D. had lost
Question 11 Visitors to the local museum are mostly attracted by __________ vase.
A. an old European beautiful B. a beautiful old European
C. an old beautiful European D. a European old beautiful
Question 12 To be the eldest child, you have to __________ a lot of responsibility.
A. get B. make C. bear D. convey
Question 13 We should participate in the movements __________ the natural environment.
A. organizing to conserve B. organized conserving
C. which organize to conserve D. organized to conserve
Question 14 I can't believe how cheap this T-shirt was. It was a real __________.
A. bargain B. offer C. discount D. sale
Question 15 An __________ child is forever asking questions. He's incredibly curious.
A.acquisitive B. acquitted C. inquisitive D. exquisite
Question 16 The project was a dead __________ from the start due to a lack of funding.
A.chicken B. duck C. cow D. cat
Question 17 Tom: "What is your greatest phobia?” - Julia: “__________.”
A. I'm afraid not. B. Worms, definitely!
C. I love puppies. D. Exactly what I think!
Question 18 This school needs __________. It looks so old.
A.repainted B. to repaint C. repainting D. to be repainting
Question 19 It's not a pleasant feeling to discover you've been taken for a __________ by a
close friend.
A.cheat B. trick C. lift D. ride
Question 20 A good way of __________ food is keeping it in a fridge.
A.preserving B. extending C. enduring D. prolonging
SECTION II. READING
Part 2. Read the following passage and decide which answer (A, B, C, or D) best fits each gap.
Write your answers in the numbered boxes.
A few decades ago, entry to university meant studying a strictly academic subject - a science,
or perhaps literature, a foreign language or the classics. These days, (21) __________ education
establishments offer (22) __________ conceivable subjects from Chinese medicine to crime scene
science. In Britain, this trend coincided with a dramatic increase in the number of universities and
more relaxed entry requirements, (23) __________ more students to experience tertiary education.
Many feared these measures would result in a (24) __________ down of degree-level courses and
the press seized every opportunity to ridicule such courses. The creation in 1999 of a bachelor's
degree in surfing studies was taken as the ultimate proof that universities were prepared to give
degrees in anything (25) __________ an attempt to attract young people, thereby (26) __________
Britain's academic reputation. In reality, the course, run by Plymouth University in South-West
England, is highly scientific, and the curriculum is demanding (27) __________ oceanography,
meteorology, materials technology and business management. Students on this course and other
seemingly more practical courses have (28) __________ performed better in tests than those doing
straight science degrees. Far from having an adverse effect on higher education, these new
disciplines (29) __________ students and aid learning by offering the opportunity to (30)
__________ theoretical knowledge directly. Many of today's graduates have a broad base of
knowledge and are well-placed to enter the job market.
Question 21 A. higher B. primary C. special D. greater
Question 22 A. many B. some C. all D. every
Question 23 A. enhancing B. benefiting C. letting D. allowing
Question 24 A. lowering B. dumbing C. detracting D. clamping
Question 25 A. on B. of C. in D. at
Question 26 A. messing B. dragging C. ruining D. cramming
Question 27 A. consisting B. compromising C. including D. insisting
Question 28 A. virtually B. actually C. ideally D. merely
Question 29 A. lighten B. motivate C. hinder D. grasp
Question 30 A. apply B. wrestle C. consume D. indulge
Part 3. Read the passage and do the tasks that follow. Write your answers in the numbered
boxes.
Population ecology is the science that measures changes in population size and composition
and identifies the causes of these fluctuations. Population ecology is not concerned solely with the
human population. In ecological terms, a population consists of the individuals of one species that
simultaneously occupy the same general area, rely on the same resources, and are affected by
similar environmental factors. The characteristics of a population are shaped by its size and by the
interactions among individuals and between individuals and their environment.
Population size is a balance between factors that increase numbers and factors that decrease
numbers. Some factors that increase populations are favorable light and temperature, adequate food
supply, suitable habitat, ability to compete for resources, and ability to adapt to environmental
change. Factors that decrease populations are insufficient or excessive light and temperature,
inadequate food supply, unsuitable or destroyed habitat, too many competitors for resources, and
inability to adapt to environmental change.
Another important characteristic of any population is its density. Population density is the
number of individuals per unit, such as the number of maple trees per square kilometer in a county.
Ecologists can rarely determine population size by actually counting all individuals within
geographical boundaries. Instead, they often use a variety of sampling techniques to estimate
densities and total population sizes. For example, they might estimate the number of black bears in
a national park by counting individuals in a few sample plots representative of the whole park. In
some cases, they estimate population size through indirect indicators, such as the number of nests
or burrows, or signs such as tracks or droppings.
Another important population characteristic, dispersion, is the pattern of spaciousness among
individuals within the population's geographical boundaries. Various species are distributed in their
habitats in different ways to take better advantage of food supplies and shelter, and to avoid
predators or find prey. Within a population's range, densities may vary greatly because not all areas
provide equally suitable habitat, and also because individuals space themselves in relation to other
members of the population.
Three possible patterns of dispersion are clumped, uniform, and random. A clumped dispersion
pattern means that individuals are gathered in patches throughout their habitat. Clumping often
results from the irregular distribution of resources needed for survival and reproduction. For
example, fallen trees keep the forest floor moist, and many forest insects are clumped under logs
where the humidity is to their liking. Clumping may also be associated with mating, safety, or other
social behavior. Crane flies, for example, swarm in great numbers, a behavior that increases mating
chances, and some fish swim in large schools so they are less likely to be eaten by predators.
A uniform or evenly spaced distribution results from direct interactions among individuals in
the population. For example, regular spacing of plants may result from shading and competition for
water. In animal populations, uniform distribution is usually caused by competition for some
resources or by social interactions that set up individual territories for feeding, breeding, or testing.
Random spacing occurs in the absence of strong attraction or repulsion among individuals in
a population. Overall, random patterns are rare in nature, with most populations showing a tendency
toward either clumped or uniform distribution.
Populations change in size, structure, and distribution as they respond to changes in
environmental conditions. Four main variables – births, deaths, immigration and emigration -
determine the rate of change in the size of the population over time. A change in the birth rate or
death is the major way that most populations respond to changes in resource availability. Members
of some animal species can avoid or reduce the effects of another with more favorable
environmental conditions, thus altering the population's dispersion. *For questions 31-36, select
the best option to answer each of the following questions.
Question 31 The passage mainly discusses the __________.
A. Population ecology versus human population
B. The distribution of populations
C. The characteristics of population ecology
D. The density and dispersion of populations
Question 32 Which of the following is an indirect indicator of a population's density?
A. The distribution of food in a given area.
B. The number of nests in a given area.
C. The number of births in a given period of time.
D. The number of individuals counted in a given area.
Question 33 According to the passage, all of the following factors cause the population of a
species to increase in size EXCEPT __________.
A. A favorable amount of light and temperature
B. A sufficient food supply
C. A large number of other species competing for food
D. An ability to adjust to environmental change
Question 34 The word "they" in paragraph 3 refers to __________.
A. ecologists B. trees C. individuals D. boundaries
Question 35 Which of the following situations would be most likely to result in a uniform
dispersion pattern?
A. Whales develop strong bonds among relatives.
B. Birds compete for a place to build their nests.
C. Elephants form a circle to protect their young.
D. Fish swim in large schools to avoid predators.
Question 36 The word "patches” in paragraph 5 is closest in meaning to __________.
A. dark places B. family groups
C. warm spots D. small areas
PHẦN II: (TỰ LUẬN)
SECTION I. GRAMMAR AND LEXICAL ITEMS
Part 2. Complete the sentences with suitable prepositions or particles. Write your answers in
the numbered boxes.
Question 1 I'm so tired of Sophie showing __________ her engagement ring all the time.
Question 2 We will have to mark __________ these old books if we want to sell them quickly.
Question 3 You can't keep your children __________ your thumb all their lives. Question 4 He
had to rub __________ his French to help his son when he started to learn it at school.
Question 5 They spent a week in the capital city and then made __________ the country.
Question 6 This book gives fascinating insights __________ wildlife in the jungle. Question 7
My younger brother is a thorn __________ my flesh. He keeps annoying me.
Question 8 Remember to go __________ your essay checking for grammar and spelling
mistakes before you hand it into me.
Question 9 Your hair and eyes remind me __________ your grandmother.
Question 10 The police arrived immediately after the call and caught the burglar __________ the
spot.
Part 3. There are TEN mistakes in the passage below. Read the passage carefully, underline the
mistakes and write your corrections on the corresponding lines in the box for Answers.
What happen if you don't get enough sleep? Randy Gardner, a high school Answers
student on the United States, wanted to find out. He designed 1. ___happens___
an experiment on the effects of sleeplessness for a school science 2. __________
project. With doctors watching him carefully, Gardner stayed awake for 3. __________
264 hours and 12 minutes. That's eleven days and nights with sleep!
4. __________
What effects did sleeplessness have on Gardner? After 24 hours
without sleep, Gardner started having trouble reading and watching 5. __________
television. The words and pictures was too blurry. By the third day, 6. __________
Gardner had trouble doing things with his hands. By the fourth day, 7. __________
Gardner was hallucinating. For examples, when he saw a street sign, he
thought it was a person. He also imagined he was a famous football 8. __________
player. Over the next few days, Gardner's speech became such slurred 9. __________
that people couldn't understand him. He also had trouble remembering 10. __________
things. By the eleventh day, Gardner couldn't pass a counted test. In the
11. __________
middle of the test, he simple stopped counting. He couldn't remember
what he was doing. 12. __________
When Gardner finally went to bed, he slept for fourteen hours and forty 13. __________
five minutes. The second night, he slept for twelve hours, the third 14. __________
night, he slept for ten and one-half hours, and by the four night, he
15. __________
had returned to his normal sleep schedule. Despite Gardner
recovered quickly, scientists believe that going without sleep can be 16. __________
endangered. They say that people should not repeat Randy's experiment. 17. __________
18. __________
19. __________
Part 4. Complete the sentences with the correct form of the words in capitals. Write your
answers in the numbered boxes.
The concept of "rhetoric", or effective public speaking, dates back thousands of years. The
underlying (1. ASSUME) __________ behind the rhetoric is that how you present an argument can
(2. GREAT) __________ influence whether people are persuaded by you or not. There is
undoubtedly plenty of evidence to support this idea - it's practically unthinkable, for example, for
a (3. SUCCESS) __________ politician to be a poor communicator – but it is just a question of
style winning over substance. Certainly, it is often said of politicians that they talk complete (4.
SENSE)[ __________ but what they say with such (5. CONVINCE) __________ that we tend to
believe them, at least when they're in opposition. On the other hand, (6. WISE) __________ and
knowledge are of little value if you can not communicate them (7. EFFECTIVE) __________ to
your peers or to the next generation. It is the combination of clear reasoning, sound (8.
JUDGE)__________ and effective presentation and communication skills that define true rhetoric.
A true (9. RHETORIC) __________ should always come across as knowledgeable, and never as
(10. OPINION) __________ or ignorant.
SECTION II. READING
Part 1. Read the text below and think of the word which best fits each space. Use only one word
for each space. Write your answers in the numbered boxes. Early photography
In the early days of photography, a stand or some other firm support for the camera was essential.
This was (1)__________ photographic materials were so insensitive to light that a typical
exposure lasted several seconds. The camera would have to be held still for this time in
(2)__________ to obtain a sharp picture. The subjects also had to be still if their images
(3)_________ to register properly on the film. Some early street scenes include blurred,
transparent, ghostlike images of people (4) __________
wandered past when the scene was in the process of being photographed. Studio portraits from
the late 1800s showed people posed rigidly, often leaning (5) __________ furniture, which
helped them to remain motionless. As it was important to keep the head still, support was often
provided (6) __________ the neck. Bright studio lights, sometimes produced by setting fire to a
strip of magnesium or a small pile of magnesium powder, helped in reducing the required
exposure time. These burned with an intensely blue flame that gave the necessary (7)
__________ of light, although the smoke was unpleasant and there was also a risk of fire.
The problems associated (8) __________ long exposure were overcome by the introduction of
faster, more sensitive photographic plates, and later, roll films. The development of smaller
cameras (9) __________ photography becoming a popular hobby. Nowadays, digital cameras
have further revolutionized photography, enabling even the (10) __________ inexperienced
photographers to produce professional-looking pictures.
Part 3. Read the passage and do the tasks that follow. Write your answers in the numbered
boxes.
Population ecology is the science that measures changes in population size and composition
and identifies the causes of these fluctuations. Population ecology is not concerned solely with the
human population. In ecological terms, a population consists of the individuals of one species that
simultaneously occupy the same general area, rely on the same resources, and are affected by
similar environmental factors. The characteristics of a population are shaped by its size and by the
interactions among individuals and between individuals and their environment.
Population size is a balance between factors that increase numbers and factors that decrease
numbers. Some factors that increase populations are favorable light and temperature, adequate food
supply, suitable habitat, ability to compete for resources, and ability to adapt to environmental
change. Factors that decrease populations are insufficient or excessive light and temperature,
inadequate food supply, unsuitable or destroyed habitat, too many competitors for resources, and
inability to adapt to environmental change.
Another important characteristic of any population is its density. Population density is the
number of individuals per unit, such as the number of maple trees per square kilometer in a
county. Ecologists can rarely determine population size by actually counting all individuals
within geographical boundaries. Instead, they often use a variety of sampling techniques to
estimate densities and total population sizes. For example, they might estimate the number of
black bears in a national park by counting individuals in a few sample plots representative of the
whole park. In some cases, they estimate population size through indirect indicators, such as the
number of nests or burrows, or signs such as tracks or droppings.
Another important population characteristic, dispersion, is the pattern of spaciousness among
individuals within the population's geographical boundaries. Various species are distributed in their
habitats in different ways to take better advantage of food supplies and shelter, and to avoid
predators or find prey. Within a population's range, densities may vary greatly because not all areas
provide equally suitable habitat, and also because individuals space themselves in relation to other
members of the population.
Three possible patterns of dispersion are clumped, uniform, and random. A clumped dispersion
pattern means that individuals are gathered in patches throughout their habitat. Clumping often
results from the irregular distribution of resources needed for survival and reproduction. For
example, fallen trees keep the forest floor moist, and many forest insects are clumped under logs
where the humidity is to their liking. Clumping may also be associated with mating, safety, or other
social behavior. Crane flies, for example, swarm in great numbers, a behavior that increases mating
chances, and some fish swim in large schools so they are less likely to be eaten by predators.
A uniform or evenly spaced distribution results from direct interactions among individuals in
the population. For example, regular spacing of plants may result from shading and competition for
water. In animal populations, uniform distribution is usually caused by competition for some
resources or by social interactions that set up individual territories for feeding, breeding, or testing.
Random spacing occurs in the absence of strong attraction or repulsion among individuals in
a population. Overall, random patterns are rare in nature, with most populations showing a tendency
toward either clumped or uniform distribution.
Populations change in size, structure, and distribution as they respond to changes in
environmental conditions. Four main variables – births, deaths, immigration and emigration -
determine the rate of change in the size of the population over time. A change in the birth rate or
death is the major way that most populations respond to changes in resource availability. Members
of some animal species can avoid or reduce the effects of another with more favorable
environmental conditions, thus altering the population's dispersion.
* For questions 1-4, write (T) if the statement is true according to the passage; (F) if the
statement is not true and (NG) if the information is not mentioned.
Question 1 __________ Counting how many individuals within geographical boundaries is a
common way to help ecologists determine population size.
Question 2 __________ Population dispersion is the distribution pattern of individuals within a
population's geographical boundaries.
Question 3 __________ The author mentions immigration and emigration in the last paragraph
to identify factors affecting population dispersion.
Question 4 __________ Densities will remain unchanged within a population's territory due to
the balance of birth and death rate.
Part 4. You are going to read an article about polyglots, people who speak many languages. For
questions 1-10, choose from the people (A-D). The people may be chosen more than once.
Questions:
1. a lack of concern about their identity?
2. advice about when to fit an important aspect of language acquisition?
3. a reason for concentrating on one language at a time?
4. a means of coping with the most challenging phase of language acquisition?
5. a description of the stages in a person's language learning method?
6. an early appreciation of the advantage of being multilingual?
7. a long-term view of their own language learning?
8. an emotion brought on by the sound of a language?
9. appreciation for an experience arising from being a polyglot?
10. the prospect of changing their language-learning goal?
Natural born linguists
What drives multi-language speakers? Martin Williams finds out.
A. Lyudmila Orlova
Being multilingual is fundamental to who I am because I think in different languages. My
mind starts a thought in one language, then finds a particular word in another language that fits
exactly what I am thinking. Each language resonates with me in a distinct way. Russian makes me
more melancholic because of its minor tone, for example. There's a downside though: when I'm in
a monolingual environment for too long, I yearn to switch to a different language. I have to think
"will this person understand me if I say something in language X"? I had an early start at learning
languages. I moved to the US from the former Soviet Union when I was three and learnt English
quickly. For some reasons though, my primary teacher didn't think I was bright enough to study
languages at secondary school which goes to show you that general academic achievement isn't
always a good indicator of one's ability to learn a language. If I had just studied languages the
formal way in school, I would never have become a hyper polyglot.
B. Simon Richards
I was always fascinated by languages and accents - I tried to mimic them all the time when I
was a child. At school, I was always drawn to the kids who had some link to abroad and I wanted
to find out more. I got to study languages at school and university eventually, and it grew from
there. Today, my daily life is multilingual. I often get mistaken for other nationalities and I
honestly no longer regard nationality as important. It seems almost odd for me to talk about being
just British now. Learning languages is an endless and ongoing process for me, which I intend to
continue for as long as my body will allow. When I am really in the learning zone, my focus is
narrow and I try to forget about my other languages. If I did, I'd go mad I simply start a new
language and don't think about the others, unless I see obvious links to help the learning process
and to understand grammar.
C. Anthony Fields
I used to spend long summers in Greece and Japan as a child, trying to play with the other kids,
but none of them spoke any English. It struck me how nice would it be to be able to talk to anybody
in the world, regardless of what language they spoke. Pronunciation is the most important thing for
me. So I start off really basic, focus on that. Once I feel more confident, I move on to music from
the language to tune in further. The dead time when you're on a bus or doing any mundane task is
a deal for squeezing in crucial subconscious language learning, which will all pay off in the long
run. Learning new languages never stops: there is always more to learn. But my student days are
almost over and the prospect of searching for a career is looming closer - so I am thinking about
taking a couple of languages to a much higher level. Speaking other languages at the native level
is an entirely different task with different rewards than just being a polyglot.
D. Liam Clarke
I did poorly in languages in school. I barely passed German and, until I was 21, I only spoke
English. I moved to Spain after graduating; after six months, I still hadn't learned any Spanish. I
kept telling myself that I didn't have the language gene. Eventually, I decided to put my excuses
aside and dive in. It took a few weeks, but as soon as I had a basic conversation in Spanish I got
hooked. That was 10 years ago, and since then I've traveled the world, learning many languages. I
don't think visiting the country where the language is spoken is really that necessary nowadays,
because of the internet. But if you do travel, it opens a lot of interesting doors. I recently had a
birthday lunch with four generations of Italians, for example. No way would that have happened If
I'd only spoken English. The initial stage is the hardest, but the only real way to get through this is
to grin and bear it. I've learned to ignore the fact that I'm making mistakes. That confidence allows
me to strive for the next level.
SECTION III. WRITING
Part 1. Rewrite each of the following sentences using the word(s) given so that its meaning
stays the same.
Question 1 He was so enthusiastic that he apparently ignored any warning signs.
→ Such _____________________________________________________
Question 2 Carol works more effectively when she's pressed for time.
→ The less time ______________________________________________________________
Question 3 We must think about ways of improving the transport system.
→ Thought must __________________________________________________________
Question 4 It's a widespread assumption that Christ was wrongly accused.
→ Christ is____________________________________________________________________
Question 5 My parents find Mary's behaviours the most irritating about her.
→ What most ________________________________________________________
Part 2. Complete the second sentence so that it has a similar meaning to the first one, using the
word given. Do not change the word given in any way. You must use between THREE and
FIVE words including the word given.
Question 1 He is said to be a good pianist. (REPUTATION)
→ He has __________ a good pianist.
Question 2 It's unusual for Linda to get angry with her kids. (TEMPER)
→ Linda hardly ever __________ her kids.
Question 3 The boy does whatever his father wants in an obedient way. (ATTENDANCE)
→ The boy __________ his father.
Question 4 You must accept the fact that she has left you. (RESIGN)
→ You must __________ the fact that she has left you.
Question 5 He thought very hard in order to remember the answer. (RACKED)
→ He __________ remember the answer.
Part 3. ESSAY WRITING: A number of teenagers today spend much of their summer vacation
doing part-time jobs. Do you that its benefits outweigh its drawbacks? In about 250 words, write
an essay to express your idea. Use specific details and examples to support your composition.
TEST 10
- PHẦN I: LUYỆN NGHE
Part 1: For each question, choose the correct answer. (Taken from PET trainer)
1. You will hear two friends talking about a film they’ve just seen.
Why didn’t the boy enjoy the film?
A. It was very frightening.
B. It lasted too long.
C. It had terrible acting.
2. You will hear two friends talking about some biology homework.
The girl suggests that the boy should
A. ask his teacher for help.
B. get information from the internet.
C. look in the biology textbook.
3. You will hear two friends talking about an interview with a singer they’ve seen on TV.
They agree that
A. the singer’s answers were interesting.
B. the interviewer was quite rude.
C. the questions were confusing.
4. You will hear a girl telling her friend about a diving trip.
How did the girl feel about it?
A. sure she will go again
B. glad she went with a relative
C. pleased with her diving skills
5. You will hear a girl talking to a friend about basketball.
The girl is trying to
A. explain the rules of the game.
B. describe a game she took part in.
C. encourage the boy to start playing.
6. You will hear a boy talking about a trip to a city with his family.
Why did the boy’s family get lost?
A. They couldn’t understand their map.
B. Someone gave them the wrong directions.
C. The guidebook contained incorrect information.
Part 2: You will hear five short extracts in which people are talking about collecting things
as a hobby. For questions 7–11, choose from the list (A–H) why each speaker collects the
things. Use the letters only once. There are three extra letters which you do not need to use.
(Taken from FCE Test)
A. I enjoy the challenge.
B. It means I spend time with my family.
C. It’s a way of meeting interesting people.
D. I want to help the local community.
E. I use my collection to teach other people.
F. It’s a financial investment.
G. It connects me to the past.
H. I like to have beautiful things around me.

Question 7. Speaker 1 __________


Question 8. Speaker 2 __________
Question 9. Speaker 3 __________
Question 10. Speaker 4 __________
Question 11. Speaker 5 __________
Part 3: Complete the table below. Write NO MORE THAN TWO WORDS OR A NUMBER for
each answer. (Taken from Listening for IELTS)
Job Location Hours Hourly rate

Hylands Hotel Near (12) Shift work No £6.75 plus (13)


(cleaning, __________ later than 10 pm __________
waitressing,
kitchen work)

The Cedars Hamilton (14) __________ £6.10


(home for elderly) Terrace Some weekends
and evenings

Looking after (15) Poplar Street 24 hours each (16) __________


__________ morning plus transport

PHẦN II: ĐỀ THI CHUYÊN SỞ GD&ĐT TP HỒ CHÍ MINH


A. TRẮC NGHIỆM
I. Use of English
Part A: Choose the correct answer to fill in the blank
1. A week into their camping holiday, the kids _______ in the open air.
A. used to cook B. would cook C. used for cooking D. were used to
cooking
2. Mike'd rather his classmate _______ games while he's studying. It's very distracting.
A. aren't playing B. didn't C. play D. hadn’t play
3. _______ for her hard work and determination, she would not have been promoted to CEO.
A. If it was not B. Were it not C. Had it not been D. Should it not have
been
4. They can’t possibly play in this heavy rain. I think they ______ as well cancel he match.
A. should B. can C. may D. will
5. She suggested _______ a charity performance on Sunday evening to raise money for the
research.
A. us to do B. us that we do C. to us doing D. to us that we
should do
6. He decided to leave one hour early, ______ caught in the traffic jam.
A. lest he be B. less he be C. least he should be D. as he be
7. “There is nothing I would like more than ______ to this highly prestigious school,” said
Nam’s mother.
A. for you to be admitted B. that you are admitted
C. if you are admitted D. only you being admitted
8. He had a reason for changing his job, but precisely _______ will probably never be known.
A. what reason was it B. what that reason was
C. which was that D. which the reason was
9. When the second half was coming to an end, Manchester United scored their third goal,
______ Leeds gave up completely.
A. until when B. at that point C. at which point D. from then
10. In spite of her physical disability, she has aways tried to lead ______ possible.
A. as normal a life as B. as normal life as
C. her life as norrnal as D. such normal life as
11. When his brother left home for university, Lam finally had a bedroom ______.
A. for him only B. for only himself C. all to him D. all to himself
12. Alice is such an impulsive girl that she often makes important decisions _____ without
consulting anyone.
A. in the nick of time B. for the time being
C. not for a moment D. on the spur of the moment
13. When our football team won the gold medal, we were ______.
A. up in the air B. on nine cloud C. over the moon D. in the seventh
heaven
14. - Jim: I’m going to use energy efficient appliances in order to lower costs and reduce
environmental impact.
- Peter: It sounds like a great dea, Jim. You should go _____ it.
A. for B. on C. over D. with
15. Linh wanted to sound confident at the interview, but I'm afraid she came _______ as
dogmatic.
A. out B. off C. through D. over
16. Through her efforts An is beginning _______ in her career.
A. pull off B. break through C. get ahead D. blow away
17. Julia is ______ of her mother. They look exactly alike.
A. living picture B. vivid picture C. visual image D. spitting
image
18. I don't want to decide until I know all the facts. Until then, I intend to ______.
A. keep a mind B. keep an open mind
C. make up my mind D. be in two minds

19. - Alex: I hear they are going to build a shopping center near this pubbic park.
- Ben: Really? ______
A. No big deal. B. It’s beyond me.
C. That's news to me. D. What it comes down to it.
20. - Tam: Do you think that the teacher would let us leave early to watch the football match?
- Minh: ______
A. Fat chance. B. Not at all. C. You'll never guess. D. Not on your life.

Part B: Choose the word or phrase that best fits each space in the following passage
PASSAGE 1
Shaped like an elongated 'S', Vietnam lies on the eastern part of Indochinese peninsula in
Southeast Asia. Vietnamesecuisine differs (1)______ radically in the north, centre and south of the
country but two key features stand out.
Firstly, rice plays an essential role in the nation's diet and so do noodles. In fact, the
Vietnamese are crazyabout noodles! Made from wheat, rice or mung beans, noodles are eaten at
any time of the day. They (2)_____ in different shapes and thicknesses and are eaten in soups, as
(3)_____ dishes or simply plain.
Secondly, fresh vegetables andherbs are a major focus of every meal. (4)______, most
Vietnamese shops for food are at least twice a day which (5)_______ why their cuisine is famous
for its use of fresh ingredients. Dishes are (6)_______ with vegetablessuch as bean sprouts, carrots,
water spinach, eggplants, cucumbers and bok choy, and seasoned with herbs like lemongrass,
coriander, basil, chilli and mint.
The characteristic flavours of Vietnamese food are sweet, spicy and sour. Asin many Asian
countries, the Vietnamese do not eat their meals in (7)______. Instead, all the food is served atonce
and people (8)_______ from the dishes in the centre of the table. Vietnamese cuisine has been
growing in (9)______ worldwide because it is not only delicious but also extremely healthy. Food
is rarely fried. Instead, it is steamed or simmered. Salads are oil free. It is thus very light and
(10)_______ its focus on vegetables, incredibly nutritious.
1. A. slightly B. fully C. mainly D. quite
2. A. form B. come C. hold D. keep
3. A. side B. accompanied C. combined D. alongside
4. A. True B. Although C. However D. Indeed
5. A. explains B. justifies C. accounts D. reasons
6. A. handed B. given C. introduced D. presented
7. A. courses B. proportions C. parts D. starters
8. A. divide B. share C. split D. portion
9. A. esteem B. recognition C. popularity D. attraction
10. A. in favour of B. for a reason C. because of D. on condition that
PASSAGE 2
Why is it that parents are so often egocentric when it comes to matters concerning their
children? Although their (1)_____ are undoubtedly altruistic, they assume that the choices they
make for their offspring are the right ones. Take the (2)_____ of colour, for example. Who decides
what colour of trousers to buy for young Jimmy? Who chooses the colours for his bedroom or
bedclothes? (3)_____, parents condition their child's perception of colour from a very early age.
Choosing a pair of pink trousers for their six-year-old son runs counter to most parents' idea of
how to dress a boy, and they would be (4)______ to decorate their daughter's bedroom in brown.
In a similar way, a small child seen drawing a red tree may be quietly told that trees should be
green. Yet the underlying criticism (5)_____ in that can be detrimental to the child's (6)______ of
the world around them.
Psychologists believe that allowing children to choose their own colours increases their
self-confidence and their ability to express themselves. They use colour as a(n) (7)______ of
helping children to identify their feelings and discuss them. For instance, (8)_____ have shown
that after listening to a sad story, children tend to draw in dark brown, black or grey, whereas one
with a happy ending will (9)______ a response in yellow or orange. So, a mother should be
delighted to see her four-year-old drawing an orange tree or a yellow house, and perhaps be
concerned if the child only uses grey. (10)______ children free rein to choose colours for
themselves may help parents to understand them better.
1. A. desires B. intentions C. incentives D. concerns
2. A. question B. type C. theme D. view
3. A. Unerringly B. Unwillingly C. Unenthusiastically D. Unwittingly
4. A. likely B. incapable C. loath D. inept
5. A. inferred B. implied C. inlaid D. imbued
6. A. thought B. sense C. feeling D. instinct
7. A. means B. tool C. method D. aid
8. A. findings B. reports C. studies D. research
9. A. lead B. evoke C. invoke D. envisage
10. A. Giving B. Letting C. Entrusting D. Entitling

II. Reading
Part A: Read the passage and choose the best answers to the questions.
Victorian style is a broad term that refers to the characteristics of design from the latter
period of Queen Victoria's rule, from 1837 until her death in 1901. At the time, the style was used
as a signal of affluence among the upper class of British society due to its omate, showy interiors.
Today, it is still popular across the globe; particularly in Asia, where it is used to create opulent
spaces that convey a high position in society.
One of the principles of Victorian design is that unused space in a room is a sign of poor
taste and lower economic status. With that in mind, European designers sought to fill every space.
Victorian design has sometimes been viewed as indulging in a grand excess of omament. Every
room was decorated with objects that reflected the owner's influences and preferences. Extravagant
decorations, lace tablecloths, stained glass, vases, busts, framed paintings or prints, multi-layered
window treatments, richly pattemed fabrics, and accessories were used throughout the house. The
Victorian Hotel in Vancouver, British Columbia, exemplifies the classic Victorian Style.
Chandeliers hang from the ceiling, vases with fresh flowers dot every table, and the dining room
features elaborately carved furniture and decorative teapots made of china. It is elegantly
appointed, true to the Victorian style. This hotel often hosts the royal family when they visit
Canada.
Emblematic of a Victorian home was the use of vibrant. brightly colored fabrics. Complex
patterns covered every surface. Oriental rugs and heavy draperies were thought of as the height of
good taste. Windows were rarely left uncovered: rather. on them hung thick, decorative drapes.
The lack of light that could enter rooms through these heavy draperies was compensated for with
eye-catching appointments of velvet, silk, and satin. Linen pattems ranged from flowers and
animals to geometric shapes, stripes, and woven Arabic symbols: squares and circles, typically
repeated, overlapped and interlaced to form intricate patterns. Victorians loved these patterns and
used them liberally.
The decoration of walls and ceilings was yet another element of Victorian design. Ceilings
were covered in embossed paper or painted a light color so as not to detract from the wall
decorations. Wallpaper became enormously popular and is an enduring feature of Victorian design.
At first, this was because of the development of mass production as well as the repeal of the 1712
Wallpaper Tax. Wallpaper was decorated with beautiful flowers in primary colors of red. blue. and
yellow, printed on beige paper. This was followed by the latter half of the Victorian era when
wallpaper was inspired by Gothic art of earth tones and stylized leaf and floral patterns. William
Morris was one of the most renowned designers of wallpaper and fabrics during this period. He
was influenced by medieval art. which he saw as the noblest art form, and used Gothic tapestries
from that era in his work.

1. The word "affluence" is closest in meaning to _______.


A. influence B. impact C. wealth D. interest
2. According to Paragraph 1, the Victorian style is used in Asia _______.
A. to conform to cultural norms B. to signal an elevated status
C. to attract visitors D. to create a comfortable and welcoming
atmosphere
3. According to Paragraph 2, why did Europeans fill up every space?
A. To show visitors their collections of objects
B. To demonstrate that they appreciated a grand excess of ornamentation
C. To showcase their personal interests by adoming any open spaces with pieces of art that
were aesthetically pleasing
D. To show that they had wealth and sophisticated taste
4. What can be inferred about the Victorian Hotel in British Columbia?
A. It was meticulously designed to meet the standards of the Victorian style.
B. It was one of the most expensive hotels to design in Vancouver.
C. It is more elegant than the home of an affluent family during the Victorian era.
D. It was designed to accommodate visits from the royal family.
5. The phrase “indulging in” is closest in meaning to _______.
A. living in B. deriving pleasure from C. obsessing about D. showing an interest
in
6. In Paragraph 2, why does the author mention the objects found in a Victorian style horne?
A. To illustrate the objects that could be purchased only by the rich
B. To point out that Victorians indulged in many objects that did not serve a useful
purpose
C. To give examples of the items that were popular in the Victorian era
D. To suggest that design has changed a great deal since Victorian times
7. What is the purpose of Paragraph 3?
A To describe another feature of Victorian style
B. To account for the use of Arabic symbols during the Victorian era
C. To describe the patterns used in Victorian design
D. To explain how homeowners compensated for a lack of natural light in Victorian style
homes
8. The word “intricate” is closest in meaning to _________.
A interesting B. colorful C. attractive D. complicated
9. All of the following are patterns that could probably be seen in Victorian linens EXCEPT
______.
A. roses and tulips B. bows and arrows C. squares and circles D. horses and sheep
10. According to Paragraph 4, why did wallpaper first become popular?
A. It became more widely available and more affordable.
B. It was a way to cover up plain walls and make them more interesting.
C. It was a reflection of how popular the Gothic style had beconw.
D. It was not as heavily taxed by the government as paint.

PHẦN II: ĐỀ THI CHUYÊN SỞ GD&ĐT TP HỒ CHÍ MINH


B. TỰ LUẬN
II. Reading
Part B: Five paragraphs have been removed from the article. Read the article and then
choose from the list A-F given below the best paragraph to fill in each of the blanks (1-5).
There is one paragraph you do not need to use
Research has shown over the years that birth order can have a fundamental effect on numerous
areas of an individual's life. Listed below are some of the main factors that scientists believe may
vary according to a child's position in the family.
Personality is one of the favourite areas of research, and most studies agree that last-borns grow
up to be the most sociable, extrovert, and creative of the siblings.
1
A second question that interests scientists is whether birth order affects brain power. Research
carried out at the Vrije Universiteit in Amsterdam showed that, in general, the IQ of a first-born
child is higher than that of a second-born child.
2
Moving on to the relationship between the siblings, studies have shown that older siblings invest
more time in younger ones than vice versa. Researchers at Newcastle University found that first-
borns had significantly more frequent face-to-face contact with their siblings than middle-borns or
last-borns.
3
On a completely different note, a study in Italy has shown that the number of children in a family
can actually affect their health. It seems that the fewer children there are in a family, the greater
the chance that they suffer from asthma or eczema.
4
Continuing on the theme of health, a Japanese study has shown that later-borns are less likely to
be overweight. Researchers found that boys from three-child families had a significantly lower
risk than only children.
5

A- Experts have no explanation for this lack of interest except that younger siblings may require
more personal space after having shared the family home throughout their childhood with their
older brothers and sisters.
B- The explanation for this difference in size is not clear, but experts maintain that mothers of
small families are often more concerned with persuading their children to eat, which can lead to
overfeeding and obesity.
C- The general reason given for these traits is that the last-born tends to be the parents' most
favoured child. A warm atmosphere makes a child feel secure, encouraging them to grow up as
self-confident individuals.
D- One possible explanation is that later-borns are associated with greater risk-taking, so they
are more prone to accidents and hospitalization, which means they take more time off work.
E- Experts use a theory called the dilution hypothesis to explain diminishing intelligence. This
suggests that the levels of parental attention and stimulation will drop as more babies come along
because family resources have to be shared among more children.
F- One theory offered to explain the prevalence of these afflictions is that younger children are
exposed to a wider range of infections by their older siblings, causing their immune system to
develop further and offer them more protection.

Part C: Supply each blank with one suitable word to make a meaningful passage.
PASSAGE 1
They are the generation born approximately between 1980 and 2000. What (1)______ them
from their parents (known as Baby Boomers, born between 1946 and 1964, and Generation X,
born between (2)______ 1960s and 1980s) is that they have been (3)______ up surrounded by
digital technology and (4)______ media.
Not only are they (5)______ with apps, iPhones and social networking, but many of them
have been responsible for bringing (6)_______ the communications revolution. (7)_______
successful young entrepreneurs, like Mark Zuckerberg founder of Facebook, they have not been
afraid to aim (8)_______ in their quest to realize their ambitions. Along the way these
entrepreneurs have accrued riches beyond their wildest (9)_______. Well-educated and
industrious, they see no limits to what they can do and trust (10)_______ technology to get them
there.
PASSAGE 2
Dairy farms have long had an overabundance of waste, particularly manure, produced by
their cows. Some farmers, however, have found a way to (1)______ it to good use: they have been
converting it into electricity. First, bacteria are added to the cows' manure to break it (2)______.
This process, known as manure digestion, produces biogas, which can then in (3)_______ be used
to generate electricity.
Recently, one dairy farmer who became (4)_______ in manure digestion 30 years ago was
finally able to start using the System. He was able not only to produce ample (5)_______ to use
on his farm, (6)______ he was also able to sell his surplus electricity to the local power company.
(7)_______ to a senior agricultural engineer. environmental concerns, the country's
growing need for renewable (8)______ and rising electricity costs render manure digestion an
increasingly attractive option. Many farmers say that they would like to reduce or even eliminate
their electricity (9)______. Doing this by transforming farm waste (10)_______ electricity would
help the environment at the same time.

III. Error correction: Identify the five (05) mistakes in the following passage and correct
them.
Line

1 Languages are not static. They are dynamic and they will continue to evolve in
2 the future for as long as people who use them also change. The advent of
online chatting, emails and text messaging has brought with it new vocabulary and
3 host of abbreviations that, in the opinion of many, will enter the most
4 respected dictionaries one day. By the time you are old and grey, perhaps 'u' and 'c'
5 will replace the pronoun 'you' and the verb 'see'. Such changes will happen
whether people like it or not-in fact, they are already happening; the terms LOL. FYI,
6
IMHO, BFF and OMG have already made it into the Oxford English Dictionary
7 online edition. No doubt, language purists will be lookied closely at future
language developments.

Question 26
Question 27
Question 28
Question 29
Question 30
IV. Word forms: Supply the appropriate forms of the words in brackets.
1. Kait said it was the _______ custom of the villagers to have a feast after the harvesting.
[memory]
2. The team recently introduced a/an _______ kitchen robot that can serve tea and other drinks.
[human]
3. Henry hopes he will soon _______ to the humid tropical conditions in Vietnam. [climate]
4. Though many people doubt its accuracy, handwriting analysis, or ______, is used by
employment agencies. [graph]
5. The _______ of Charles as Prince of Wales took place on July 1st 1969 and was well received
by many Welsh people. [invest]
6. The archive was a/an _______ important addition to the school library’s collections. [estimate]
7. The channel continued airing _______ after a classic movie every night. [shop]
8. The type vehicle you are in, certain disorders and sensory mismatch can all make you feel more
_______ on your journey. [travel]
9. Banking continues to be one of the most _______ career choices among students. [seek]
10. There are unsung heroes who individually and _______, quietly work for social change and
never receive public attention. [assume]
V. Writing: Complete the second sentence so that it means the same as the first one, using
the word given. you must use from three to eight words, including the word given. (20 pts)
1. Passengers are absolutely forbidden to cross the railway track. (ALLOWED)
→ On _________________________________________________ the railway track.
2. We should have had someone repair the roof in the dry season rather than leaving it until the rainy one.
(BETTER)
→ It would ______________________________ repaired in the dry season rather than leaving it
until the rainy one.
3. The company avoids employing unqualified staff unless there is no alternative. (RESORT)
→ Only ________________________________________________________ unqualified staff.
4. Williams suggested a guest list should be written. (DRAWN)
→ “Why ______________________________________________________________,” said
Williams.
5. Bill was concentrating hard on his work, so he didn't notice when I came in. (WRAPPED)
→ Bill was so _____________________________________________________ notice of my
coming in.
6. Ben is far superior to Dylan in terms of technical knowledge. (MATCH)
→ When it _____________________________________________________________________
for Ben.
7. She always enjoys this programme although she watches it very often. (TIRES)
→However __________________________________________________________ of it.
8. The organizers planned everything as carefully as they could possibly have done. (UTMOST)
→ Everything _________________________________________________ the organizers.
9. "The situation won't be repeated in the future, I promise," said Mike to his mother. (WORD)
→ Mike ________________________________________________ repetition of the situation in
the future.
10. Jack admitted that he was responsible for the broken window. (OWNED)
→ It was _________________________________________________________ the window.
PHẦN II: ĐỀ TỰ LUYỆN CÓ GIẢI CHI TIẾT
TEST 1

PART ONE: LISTENING

Exercise 1. For questions 1-5, choose the answer (A, B, C or D) which fits best according to
what you hear. (5 pts)

LATIN AMERICAN STUDIES

1. Paul decided to get work experience in South America because he wanted_______.

A. to teach English there B. to improve his Spanish

C. to learn about Latin American life D. to improve his English

2. What project work did Paul originally intend to get involved in?

A. construction B. agriculture

C. tourism D. architecture

3. Why did Paul change from one project to another?

A. his first job was not well organized B. he found doing the routine work very
boring

C. the work was too physically demanding D. his first job was boring

4. In the village community, he learnt how important it was to________.

A. respect family life B. develop trust

C. use money wisely D. spend time with neighbors

5. What does Paul say about his project manager?

A. he let Paul do most of the work B. his plans were too ambitious

C. he was very supportive of Paul D. he was too ambitious


Exercise 2. You will hear a man called Stephen Mills talking to a group of people about a trip
to India to see tigers. Listen and fill in a blank with ONE word you hear from the recording.
Write your answers in the table below. (10 pts)

Hello. I'm Stephen Mills. I will travel with you on the Tiger Tour to India, but I'm
not your tour guide. I work as an (1)__________ and I'm going to take some photographs of
tigers so that I can use them when I paint some pictures later. An (2) _____________guide will
join the group when we arrive in India, but I'm leading the group until then.

We leave London on the (3)______________ of November, arriving in India the


following day. It's a good time of year to visit the wildlife park where the tigers live. The rainy
season finishes in October. And later in the year, the park gets more (4)_______________ and
the tigers become shy.

We'll spend ten days in the wildlife park. There are twenty other types of animal and
three hundred types of bird to see as well as tigers. There are eighteen of us altogether and
everything is organized for our (5)______________ by the tour company. For example, although
we're in the jungle, we won't have to sleep in (6)______________ ! The hotels where we'll stay
are all very comfortable.

To be sure of seeing tigers, we'll stay in two different parts of the wildlife park. We'll
spend three days in the north, where we'll travel around in an open truck, and the rest of the time
in the south, where we'll travel around on (7)_____________ That should be fun!

On the way back to London, we have dinner and one night's (8) ______________and
breakfast in the Indian capital, Delhi. There you can either go sightseeing or go shopping,
whichever you (9)______________. But please note that lunch is not provided on our day of
(10)____________ , as the plane leaves at two in the afternoon. Now, if there are any questions...

PART TWO: VOCABULARY AND GRAMMAR

Exercise 1. Mark the letter A, B, C or D to indicate the correct word or phrase to complete
each of the following sentences. (10 pts)

1. Marie Curie,................................, was awarded a Nobel Prize for her work.


A. the scientist discovered radium B. whose scientific discovery of radium

C. was the scientist who discovered radium D. the scientist who discovered radium

2. ………............... had worsened so quickly took the doctor by surprise.

A. Which the patient’s condition B. The patient’s condition

C. That the patient’s condition D. As the patient’s condition

3. She resented …………………………. .

A. being made work overtime B. to be made to work overtime

C. being made to work overtime D. to be made working overtime

4. The school staff needs new …………….. in order to bring in novel ideas.

A. thing B. blood C. employees D. offices

5. She …………… fainted when she heard that her child died.

A. rather than B. nothing but C. all but D. near

6. - I think that Mick will leave his new job before the year is out.

- Yes, I agree. I don’t think he’ll……………………..either.

A. stay the course B. relieved of his duties

C. get the sack D. take industrial action

7. He is very ………………….. about the kitchen - everything has to be perfectly clean and in its
place.

A. creditable B. particular C. concerned D. delectable

8. They gave us a ……………………………. dinner.

A. beautifully-prepared lavish five-course Italian

B. five-course lavish Italian beautifully-prepared

C. lavish beautifully-prepared Italian five-course


D. beautifully-prepared five-course lavish Italian

9. It is imperative that your facebook password…………………….confidential.

A. need to keep B. needs to be kept C. need keeping D. needs keeping

10. - “ Last I heard, she was working in South Africa.” - “................................?”

A. Why did she B. How come C. How did she D. Why come

11. Timmy had a lucky escape. He……………………..killed.

A. could have been B. must have been

C. should have been D. shouldn’t have been

12.The kids’ performance was admirable…………………………….. .

A. but for their informal training

B. inasmuch as they had received no formal training

C. although their training was formal

D. regardless of the informal training they received

13. John: ………………what he has said be true?

Mary: Yes, it………………be true.

A. May/may B. Can/can C. May/must D. Can/must

14. Disappointment ………………..on Richard’s face when his parents didn’t let him go skiing.

A. enacted B. registered C. extended D. assigned

15. Down into the cave ……………………..

A. did the rescue party go B. the rescue party went

C. went the rescue party D. did go the rescue party

16. ………………………., the diners settled the bill and left the restaurant.

A. Having hunger satisfied B. Their hunger satisfied


B. Hunger been satisfied D. Satisfying their hunger

17. She is reported to be……………………… a spectator.

A. an athlete more than B. more an athlete than

C. an athlete of more than D. more of an athlete than

18. In total, the repairs will cost somewhere in the…………………..…….. of £3000.

A. region B. field C. area D. zone

19. Inspector Moro was determined to get to the……………….. of the mystery.

A. ground B. tune C. end D. bottom

20. We couldn’t help laughing when he…………………. his teacher so well.

A. took over B. took off C. took up D. took out

Exercise 2. Give the correct form of the words in brackets to complete each of the following
sentences. (10 pts)

1. As the fog……………………….., we couldn’t continue our journey. (THICK)

2. She’s very efficient, and …………………………polite to the customers. (FAIL)

3. With its decorative evergreen leaves, this plant will …………………….up shaded areas.
(LIFE)

4. His rude comments left her…………………..…. and in tears. (SPEAK)

5. Japan’s …………………………. economy was an excellent example of an economic


recovery. (WAR)

6. This drink contains no artificial flavorings or ……………………. (PRESERVE)

7. The reason for Sue’s death was sleeping pill……………………….(DOSAGE)

8. The terrible scenes were indelibly ………………………… on his mind (PRINT)

9. The scheme looked good on paper but in practice, it proved to


be………………………(WORK)
10. My friends started going out late to nightclubs so I decided to ………………………myself
from the group. (SOCIAL)

Exercise 3. The following passage contains 10 mistakes. Identify the mistakes and write the
corrections in the corresponding numbered boxes. (0) has been done as an example. (10 pts)

0. Line 1: include → includes

Line

1 Whirlwind, any rotating air mass, include the tornado and the large
cyclonic and anticyclonic storm. In meteorology, the term whirlwind is more
2
strictly application to the smaller swirling atmospheric phenomenon commonly
3 known as dust devil or dust whirl, which occurs mostly over deserts and

4 semiarid plains during hot, calm days. The principle cause of whirlwind is
intense insulation, or incoming solar radiation received from the earth, which
5
produces an overheated air mass just above the ground. This air mass arises,
6 usually in the form of a cylindrical column, sucks up loose surface material, just
as dust, sand, and leaves. Whirlwinds vary with height from 30 to 152 m, but
7
exceptionally vigorous dust devils may exceed 1,524 m in height. The vortices
8 of whirlwinds range in size from a few meters to several hundred meters and,
9 depend on their force and size, dust devils may disappear in seconds and last
several hours. Brief whirlwinds are erratic in motions, but the longer-lasting
10
ones move slowly with the prevailing winds.
11

12

13
PART THREE: READING COMPREHENSION

Exercise 1. Read the text below and think of the word which best fits each space. Use only
ONE word in each space.

The city of Melbourne, Australia has always had a (1)__________ for unusual
weather. Melbourne people enjoy telling this joke to visitors: if you don’t like the weather in
Melbourne, don’t worry, just wait five minutes, because it’s sure to change.

At the beginning of 1992, Melbourne had its (2)___________ January for over 100
years. It rained for nine days on (3) ____________. As well as raining all day, the weather was
also cold. At night, people in some Melbourne suburbs were switching on their heating as if it
was winter. It was (4)____________ cold many people could (5)____________ believe that it
was summer at all.

The best weather in Melbourne, however, is not usually in the summer: it is in the
autumn. The autumn usually has more pleasant days than the summer. The weather in autumn is
usually

(6)___________ of warm days and cool, comfortable (7)____________.

The (8)____________ of the city at this time of year is beautiful, too. Melbourne has
many lovely gardens and parklands with beautiful trees. In the autumn, the trees change their
leaves to red, gold and brown. As they are blown in the air by the wind of a (9)_____________
autumn day, the leaves add life to the city. So even if the summer is cold and wet, people in
Melbourne can still look forward to the (10)_____________ of a warm sunny autumn.

Exercise 2. Read the following passage and mark letter A, B, C or D to indicate the correct
answer to each of the questions from 26 to 35 (10 pts)

It has become an American tradition that those who attain great wealth return some of
it to the public through philanthropy. An early example of this was the generosity of Amos
Lawrence of Massachusetts, a wealthy merchant, who in the 1830's and afterwards contributed
much money for famine relief in Ireland. He also donated generously to educational and other
humanitarian causes.

In the early years of the twentieth century several men who had amassed vast fortunes
likewise became great philanthropists. Andrew Carnegie, an exceptionally energetic man, who
had begun working twelve hours a day when he was only fourteen years old, became one of the
world's richest men by pioneering in the steel industry. After his retirement in 1990 he devoted
his time and his wealth to the establishment of free public libraries. He also set up foundations
for medical research and for world peace. Carnegie's belief, as he expressed it in an essay, was
that the wealthy person must 'consider all surplus revenues which come to him simply as trust
funds' which he 'is strictly bound as a matter of duty to administer in the manner...best calculated
to produce the most beneficial results for the community - the man of wealth thus becoming the
mere trustee and agent for his poorer brethren.

John D. Rockefeller, who also began as a poor boy, became fabulously rich through oil
refineries and other enterprises. In his old age, in the early 1900's, he began to donate millions
for beneficial undertakings. The various Rockefeller foundations support research as well as
humanitarian causes in the United States and in other parts of the world. Rockefeller funds are
now fighting hunger through the so-called 'green revolution,' whereby new agricultural
techniques have greatly multiplied the yield of food-crops in Mexico, India, Pakistan, and part of
Africa.

26. How to become a philanthropist?

A. You must be very rich and know how to keep it.

B. You are very rich and donate your money to humanitarian causes.

C. You are a very rich and well-known merchant.

D. You are a very rich and well-known person in the steel industry.

27. Who contributed much money for famine relief in Ireland?

A. Amos Lawrence B. Massachusetts C. Andrew Carnegie D. Rockefeller


28. Who is very successful in the steel industry?

A. Amos Lawrence B. Massachusetts C. Andrew Carnegie D. Rockefeller

29. The word “enterprises” in the last paragraph means:

A. projects B. companies C. organizations D. industries

30. The word “revenues” in the second paragraph means:

A. best things B. incomes C. fund D. trust fund

31. The word "brethren" in the second paragraph means:

A. men B. people to C. brother D. bother

32. The word "yield” in the last paragraph means:

A. total areas B. total techniques C. total amount D. shout out

33. The word "green revolution" in the last paragraph are about

A. a terrible fighting B. a new technique

C. an agricultural project D. a major revolution

34. Which activity is not mentioned in the passage?

A. Fighting in the United States and in other parts of the world.

B. Setting up free public libraries.

C. Supporting medical research.

D. Donating to educational and other humanitarian causes.

35. Which of the following best sums up Carnegie's opinion?

A. A wealthy person should appreciate the fortune he has, earn more and more for his family and
his business.

B. A wealthy person is a person who knows how to make benefits from his money for his family
and his business.
C. A wealthy person should be an honest person that people trust.

D. A wealthy person has to keep and administer his money to produce most benefits for the
community.

Exercise 3. Read the following passage and mark letter A, B, C, or D to indicate the correct
word or phrase to complete each of the numbered blanks from 36 to 45. (5 pts)

PROFESSIONAL SPORT

When I talk about (36)___________, I mean something individual. If you look at


professional golfers or tennis players, you will observe that practice (37) ___________ the basis
of their preparation for tournaments, of course it does, (38) ___________ these are individual
sports. Training is simply doing the physical work necessary to be sufficiently fit, but practice
entails making an analysis of one’s game, locating its weak points, and working to (39)
___________them. If your opponent keeps (40) ___________ points by serving high to your
backhand, there’s no place to hide; you either have to (41) ___________ the weakness, or you’ll
keep losing to him. He’ll exploit your weaknesses mercilessly.

(42) __________ of what is fascinating, and slightly repellent, about sport lies in that
curious inversion of values (43) __________ much that we admire in ordinary life – humility,
compassion, unwillingness to take advantage of the weakness of others - is (44) ___________on
the field of play. Professional sport is all about winning. And, within (45) ___________ the more
you practice (and the more you train), the better your chances of doing so.

36. A. improvement B. practice C. training D. rehearsal

37. A. lays B. composes C. enacts D. forms

38. A. albeit B. for C. as with D. let alone

39. A. eradicate B. rid C. wipe D. extinguish

40. A. getting B. making C. winning D. scoring


41. A. alter B. square C. reform D. rectify

42. A. kind B. sort C. part D. most

43. A. insofar B. whereby C. thereof D. hence

44. A. reversed B. changed C. contrasted D. contradicted

45. A. due B. sense C. reason D. merit

Exercise 4. You are going to read an article about dreaming. Six sentences have been removed
from the article. Choose from the sentences A-G the one which fits each gap (37-42). There is
one extra sentence which you do not need to use.

Can we control our dreams?

Strange as it seems, the answer is yes – and it could help us solve our problems.
Do we have any influence over the often strange, wandering, night-time journeys in our mind?
Could we learn to dream differently, getting rid of repeated nightmares or finding answers to the
problems that we cannot solve in daylight hours? Strange though it may seem, the answer is yes.
Research suggests that, using practical and psychological techniques, we can influence our
dreams and use them to draw on the vast, largely unused resource of our unconscious mind.

Deirdre Barrett, an assistant clinical professor of psychology at Harvard Medical School, is


convinced we all have the power to manage our dreams. ‘If you want to dream about a particular
subject,’ she says, ‘focus on it once you are in bed. 37 . You can also place an object
or photo that represents the desired dream on your bedside table,’ Barrett says.
Another key factor in using one’s dreams creatively is to avoid jumping out of bed the moment
you wake up. 38 . ‘If you don’t recall a dream immediately, lie still and see if a
thought or image comes to mind,’ Barrett says. ‘Sometimes a whole dream will come flooding
back.’
The point of this second strategy is to make use of the information presented by our unconscious
as we sleep. It’s hard to put an exact figure on the ratio of our unconscious to conscious mind,
but psychologists estimate it to be nine to one. We may believe that thinking is our best problem-
solving strategy, but the power of our conscious mind is relatively tiny. 39 . So letting
the unconscious mind work on it may be healthier and more productive.
Barrett put this to the test in a week-long study with college students. She asked them to use
dreaming as a way of finding ways of dealing with a particular problem. 40 . ‘If we’re
stuck on a problem, it’s our waking, logical way of thinking that’s stuck,’ Barrett says. ‘The
dream’s power lies in the fact that it’s a different manner of thought – it adds to and develops
what we’ve already done while awake.’
Most of us enjoy the rich, pleasantly strange experience of dreaming (and we all dream – some
people just don’t remember it). But no one enjoys nightmares that keep coming back, or the kind
of unpleasant dreams from which you wake sweating. 41 . ‘It’s very common for them
to have nightmares about being chased by a monster,’ says Delphi Ellis, a counsellor and dream
expert. ‘This often happens as they get older and become aware of their place in the huge world.’
As an adult, troubling or frightening dreams are often an indication of difficult issues from the
past,’ Ellis says. 42 . They and all other kinds of dream are an incredibly valuable
resource, which most of us simply ignore. So learn to listen to them, even the horrible ones –
they’re always trying to tell you something.’
A. It’s one in which you know you’re dreaming as the dream is occurring – the kind of ‘dream
within a dream’ that film characters sometimes have.
B. Even more anxiety-causing, if you’re a parent, are the scary ones that have such an effect on
your kids.
C. Doing so means you’ll lose half of what your dream contained as the day’s distractions take
over your thoughts.
D. About half of them dreamt about it and one-quarter of them solved it
E. Since dreaming is so visual, form a picture in your mind of something related to that topic as
you fall asleep.
F. The more you ignore dreams like those, the more your unconscious turns up the volume – so a
nightmare is that message on full volume.
G. Also, when this consists of going over and over negative or worrying issues in our minds, it is
strongly linked with stress, depression and anxiety.

Exercise 5. You are going to read a magazine article about bicycles. For questions 43 – 52,
choose from the sections (A – D). The people may be chosen more than once.
Which person…

43. bought a second-hand bike?


44. says their new bike is good value for money?
45. found it difficult to slow down at one point?
46. had to take their bike in for repair?
47. needed to put the bike together before they could ride it?
48. bought a new bicycle to replace one that had been stolen?
49. says that riding their bike up hills is tiring?
50. says they wish they had checked the size of the bike sooner?
51. had to get off their bike when they were riding to fix it? 52. compares cycling with
another way of keeping fit?
My new bike

Four people share their experience of owning a bicycle


A Jonas Hagen
I bought my new mountain bike online and as soon as it was delivered to my home in kit form I
set to work. Once it actually looked like a bike and I’d checked that everything seemed to be
working properly, I set off down the road. All went well at first, but later on I had a brief
moment of panic when the brakes suddenly failed and I narrowly avoided crashing into a hedge.
I adjusted them when I got home, and since then they’ve been fine. The only other adjustment
I’ve had to make is to raise the seat to the maximum because it turns out this bike is for riders
whose inside leg measures considerably less than mine. I should really have noticed that before I
bought it.
В Lili Huang
I originally bought my bike just for occasional use, but now I go everywhere on it. It’s great
exercise, every bit as good as going to the gym. It feels just the right size for me and somehow I
always feel full of energy when I’m on it, even when my friends and I ride into the mountains at
weekends. I’ve only ever had one breakdown, which was when the chain broke. Fortunately
there was a garage nearby, where a very kind car mechanic quickly got me back on the road. I
don’t know what I’d do without my bike, which is slightly worrying because a lot of people
round here have had their bikes stolen. That’s why I keep it in the hall downstairs, rather than in
the street.
C James Thompson
This is only the second bicycle I’ve ever bought. It was on offer at the local cycle shop and I
think I got something of a bargain because on the whole I’ve been pleased with it. At first I had
some difficulties with the gears, but I managed to sort those out while I was riding. It’s a very
solid bike, though that does mean it’s rather heavy and I wouldn’t want to have to push it far if I
had a breakdown. It also makes pedalling up steep slopes hard work, although fortunately most
of the routes round here are reasonably flat. I don’t think it’s the kind of bike anyone would want
to steal, but I always secure it with a good strong lock just in case. Recently I’ve also fitted more
powerful front and rear lights so that drivers can see me better after dark.
D Mia Doherty
I actually chose this bike in something of a hurry. I’d left my old one outside the sports centre
and when I came back it had just disappeared. I reported it, of course, but that was the last I ever
saw of it, and I needed a new one to get to work every day. I probably paid more than I should
have done for it, though I know the previous owner had looked after it well and I haven’t had any
trouble with it. Apart, that is, from a flat tyre which meant I had to stop and mend it on the way
home in the pouring rain. Riding it certainly helps keep me fit, and even in weather like that I
wouldn’t change it for a car. The only incident I’ve had was when a dog ran out into the road in
front of me, but the brakes did their job superbly.

PART FOUR: WRITING


Exercise 1. Finish the second sentence in such a way that its meaning is similar to that of the
original one. (10 pts)

1. It wasn’t Sarah that you saw in the department store, for sure.

→ It couldn’t ……………………………………………………………………….…

2. I don’t really like her, even though I admire her achievements.

→ Much …………………………………………………………………………...her.

3. The boy was about to cry when he was reprimanded by his mother.

→ The boy was on ……………………………………………………………………

4. If she fails to accomplish her mission, her job will be put at risk.

→Should………………………………………………………………………… line.

5. The northwest of Britain has more rain each year than the southeast.

→ The annual………………………………………………………………………….

6. It’s not certain that Jones will get the job.

→ It’s open…………………………………………………………………………….

7. The bus came round the corner.

→ Round ……………………………………………………………………………..

8. “Please don’t run so fast!” Suri begged his friend.

→ Suri pleaded………………………………………………………………………

9. It wasn’t Melanie’s fault that she ended up breaking the law.

→ Through no ………………………………………………………………………

10. His recent behaviors are outrageous.

→ The way…………………………………………………………………ordinary.
Exercise 2. Rewrite each of the sentences below in such a way that its meaning is similar to
that of the original one, using the word given in brackets. Do not change this word in any way.
(5 pts)

1. I expected the film to be good, but it wasn’t at all. (LIVE)

→ The film …………………………………………………………………………

2. Will you please stay with me for a while? (COMPANY)

→Will you ……………………………………………………………………….?

3. The new plans for the school have been approved by the authorities. (MET)

→ The new plans…………………………………………………………………..

4. He is so ambitious - he's determined that he'll be successful in the company. (MARK)

→ He is determined ……………………………………………….in the company.

5. Russ’s opinions on the new management policies were very different from those of his

fellow workers. (ODDS)

→ Russ …………………………………………….the new management policies.


TEST 2

PART ONE: LISTENING

Exercise 1. You will hear part of an interview with a man called Tony Elliott who founded a
magazine called Time Out. For questions 1-5, choose the answer (А, В, C or D) which fits best
according to what you hear. (5 pts)

1. Tony says that Time Out was unlike other publications in 1968 because

A. it was written by one person. В. information was more accurate.

C. it had a comprehensive list of events. D. it was in the form of a magazine.

2. What experience did Tony have of publishing?

A. He had worked for What’s On. В. He had written numerous articles.

C. He had transformed an existing magazine. D. He had started a student magazine.

3. Why did Tony leave university?

A. He wanted to go to France. В. He didn’t have time to study.

C. He had failed his French examinations. D. He had found an alternative career.

4. According to Tony, what led to the magazine becoming a weekly?

A. some market research В. the quantity of information

C. technical improvements D. external pressure

5. Tony says that, compared to 1968, people who buy Time Out today are

A. more intelligent and active. В. more likely to be parents.

C. more or less the same age. D. more mature and professional


Exercise 2. Listen and fill in a blank with ONE word you hear from the recording. Write your
answers in the table below. (10 pts)

DESIGNING A PUBLIC BUILDING: THE TAYLOR CONCERT HALL

Introduction

The designer of a public building may need to consider the building’s

● Function
● Physical and (6)__________________ context
● Symbolic meaning
Location and concept of the Concert Hall

● On the site of a discussed (7)__________________


● Beside a (8)___________________
● The design is based on the concept of a mystery
Building design

● It’s approached by a (9)___________________ for pedestrians


● The building is the shape of a (10)____________________
● One exterior wall acts as a large (11)___________________
In the auditorium:

● The floor is built on huge pads made of (12)_________________


● The walls are made of local wood and are (13)_________________ in shape
● Ceiling panels and (14)__________________on walls allow adjustment of
acoustics
Evaluation

● Some critics say the (15)_________________ style of building is inappropriate


PART TWO: VOCABULARY AND GRAMMAR

Exercise 1. Mark the letter A, B, C or D to indicate the correct word or phrase to complete
each of the following sentences. (10 pts)

1. In filmmaking, ………………. the segments of a film are presented can be varied to create
drastically different dramatic effects.

A. the order in which B. the order which in

C. in the order which D. in which the order

2. ………………… he was kidnapped by the Iraqi Guerrillas yesterday has been confirmed.

A. What B. If C. That D. O

3. It pays………………….. some professional advice before you make a decision.

A. get B. getting C. to getting D. to get

4. He decided to ……………………… and tell his wife exactly what had happened.

A. bark up the wrong tree B. come as a bolt from the blue

C. ring a bell D. make a clean breast of it

5. He is……………….influenced by his father and grandfather. His behaviors and decisions are
exactly the same.

A. mighty B. strongly C. terribly D. weakly

6. Richard is asking Ann about quantum physics, but Ann doesn’t know the answer to his
question. What would Ann be most likely to say?

– Richard: “So, Ann, do the electrons in a pair spin in two opposite directions or not?”

– Ann: “...............................................!”

A. Mind your own business B. Whatever floats your boats

C. This is way below me D. Search me


7. People expect their representatives on the council to be ready and willing to………………..
the important local issues.

A. address B. target C. hit D. criticize

8. The nature of agricultural production on the prairies is distinct from………………….. of the


rest of Canada.

A. those B. what is C. that D. it is

9. All the evidence suggests that he……………………… the money before he left here.

A, steal B. steals C. stole D. had stolen

10. – “We’ll have to hurry if we want to finish this project on time.”

– “.................. ............”

A. OK. But I’ll call you later B. I’ll say we will

C. I’m tired. I’ll go home early D. What a wonderful idea!

11. Luckily I…………………….. a new pair of sunglasses as I found mine at the bottom of a
bag.

A. needn’t have bought B. needed not to buy

C. hadn’t to buy D. didn’t need to buy

12. He works until nine o’clock every evening, and that’s quite ………………… the work he
does over the weekend.

A. except for B. apart from C. without D. not for

13. Nguyen Hue made ……………….. surprise attack against……………….. Chinese during
Tet, …………………… Vietnamese and Chinese Lunar New Year holiday.

A. a/O/O B. the/O/O C. a/the/the D. a/the/O

14. After a thorough search, the police………………… most of the missing jewels.

A. recovered B. refreshed C. retreated D. recorded


15. Fish are the most ancient form of vertebrate life, and…………………. all other vertebrates.

A. from them evolved B. evolved them

C. to evolve D. they are evolved

16. To celebrate the 100th anniversary of the college, ............................ .

A. honoring those alumni who had graduated fifty years ago

B. ceremonies for graduates of fifty years ago were held in honor of these alumni

C. alumni graduating fifty years ago received honors

D. alumni who had graduated fifty years ago were honored

17. Every delicacy Miss Cook produces is done………………………….

A. to a turn B. at will C. there and then D. sooner or later

18. In all…………………., he’s already left.

A. odds B. probability C. certainty D. possibilities

19. Son Tung MTP is a singer who has taken audiences ............................

A. by heart B. by night C. by wind D. by storm

20. Since he spoke about the subject so indirectly, it was difficult to see what he
was………………….

A. getting on B. getting at C. getting in D. getting up n

Exercise 2. Give the correct form of the words in brackets to complete each of the following
sentences. (10 pts)

1. Why are you so late? Did you ……………………again? (SLEEP) oversleep

2. We're a ……………………….wealthy county, but our resources are getting exhausted.


(COMPARE) comparatively
3. Mitchell joined the Navy at 16 by …………………..his birth certificate. (FALSE) falsifying

4. I love this painting of an old man. He has such a beautiful…………………… smile.


(CHILDREN) childlike

5. There are very few…………………… places left on the earth. Man has been nearly
everywhere. (EXPLORE) unexplored

6. They stopped at the top of the hill to admire the ………………………. .(SCENE) scenery

7. The Manufacturing Technology Show in Chicago attracted nearly 90,000 …………………….


and 1,200 exhibitors. (ATTEND) attendees

8. ……………………… nights without sleep make any new parent feel ready to quit.
(SUCCESS) successive

9. The new regulations will not make a/an ………………….. difference to most people.
(APPRECIATE) appreciable

10. The newspaper denies any intention to ……………………the senator’s reputation.


(FAMOUS) defame

Exercise 3. The following passage contains 10 mistakes. Identify the mistakes and write the
corrections in the corresponding numbered boxes. (0) has been done as an example. (10 pts)

0. Line 1: have → has

Line OAK
1 Oak wood have a density of about 0.75 g/cm3, great strong and
hardness, and is very resistant to insect and fungal attack because of its high
2
tannin content. It also has very appeal grain markings, particularly when
3 quarter-sawn. Oak planking was common on high status Viking long ships in

4 the 9th and 10th centuries. The wood was hewn from green logs, with axe and
wedge, to produce radial planks, similar to quarter-sawn timber. Wide, quarter-
5
sawn boards of oak have been prized since the Middle Ages for use in interior
6 paneling off prestigious buildings such as the debating chamber of the House of
Commons in London, and in the construction of fine furniture. Oak wood, from
7
Quercus robur and Quercus petraea, was used in Europe for the construction of
8
ships, especial naval men of war, until the 19th century, and was the principal
9 timber used in the construction of European timber-framed build.

10 Today oak wood is still commonly used for furniture making and
flooring, timber frame buildings, and for veneer production. Barrels in which
11
wines, sherry, and spirits such as brandy, Scotch whisky and Bourbon whiskey
12 are age are made from European and American oak. The use of oak in wine can

13 add many different dimensions to wine based on the type and style of the oak.
Oak barrels, which may be charred before use, contribute to the colour, taste,
14
and aroma of the contents, imparting a desirable oaky vanillin flavour to these
15 drinks. The great dilemma for wine product is to choose between French and
American oakwoods. French oaks give the wine greater refinement and are
16
chosen for the best wines since they increase the price compared to those aged
17 in American oak wood. American oak contributes greater texture and resist to

18 ageing, but produces more violent wine bouquets. Oak wood chips are used for
smoking fish, meat, cheeses and another foods.
19

20

21

22
23

PART THREE: READING COMPREHENSION

Exercise 1. Read the text below and think of the word which best fits each space. Use only
ONE word in each space.

In many countries of Europe, there has been a steady drift of people away from
villages to large cities. These people, many of whom have grown in great poverty and
deprivation, hope to improve their standard of (1) _____living________and see the metropolis as
the solution to all their problems. In many ways, they find what they are looking for. Large cities
do offer a huge number of facilities, (2) _____among_______ which better education, better
health care and improved housing are perhaps the most important. Large companies and factories
, the vast (3) ____majority_________ of which pride themselves on looking after the interests of
their employees, also open up any (4) ______number_____ of career opportunities for those
willing to work hard. Inevitably, however, (5) ______there_____ comes a time when people
begin to long for the simplicity of the village or small town.
Traffic problems and pollution, both of which affect most large cities today, cause the
most unhappiness. For people to whom fresh air, unpolluted water and beautiful countryside are
distant-(6) _____but_______ nonetheless painful clear-memories of a previous, peaceful life in a
village, the situation must at (7) ______times_______ be unbearable. The pressure of
overpopulation has meant that, in the last thirty or forty years, thousands (8)
___upon_________Thousands of new flats have been built, often with (9)
____scant/little________ regard to architectural beauty and the surrounding countryside has all
(10) ______but_______ disappeared in many cases. It is no longer such an easy matter to escape
the noise and the turmoil of the streets and find a field or a forest where the children can play in
safety.

Exercise 2. Read the following passage and mark letter A, B, C or D to indicate the correct
answer to each of the questions from 26 to 35 (10 pts)

The word ‘laser’ was coined as an acronym for Light Amplification by the Stimulated
Emission of Radiation. Ordinary light, from the Sun or a light bulb, is emitted spontaneously,
when atoms or molecules get rid of excess energy by themselves, without any outside
intervention. Stimulated emission is different because it occurs when an atom or molecule
holding onto excess energy has been stimulated to emit it as light.

Albert Einstein was the first to suggest the existence of stimulated emission in a paper
published in 1917. However, for many years physicists thought that atoms and molecules always
were much more likely to emit light spontaneously and that stimulated emission thus always
would be much weaker. It was not until after the Second World War that physicists began trying
to make stimulated emission dominate. They sought ways by which one atom or molecule could
stimulate many others to emit light, amplifying it to much higher powers.

The first to succeed was Charles H. Townes, then at Columbia University in New
York. Instead of working with light, however, he worked with microwaves, which have a much
longer wavelength, and built a device he called a ‘maser’, for Microwave Amplification by the
Stimulated Emission of Radiation. Although he thought of the key idea in 1951, the first maser
was not completed until a couple of years later. Before long, many other physicists were building
masers and trying to discover how to produce stimulated emission at even shorter wavelengths.

The key concepts emerged about 1957. Townes and Arthur Schawlow, then at Bell
Telephone Laboratories, wrote a long paper outlining the conditions needed to amplify
stimulated emission of visible light waves. At about the same time, similar ideas crystallized in
the mind of Gordon Gould, then a 37-year-old graduate student at Colombia, who wrote them
down in a series of notebooks. Townes and Schawlow published their ideas in a scientific
journal, Physical Review Letters, but Gould filed a patent application. Three decades later,
people still argue about who deserves the credit for the concept of the laser.

26. The word ‘coined’ in paragraph 1 can best be replaced by

A. created B. mentioned C. understood D. discovered

27. The word ‘intervention’ in paragraph 1 can best be replaced by

A. need B. device C. influence D. source

28. The word ‘it’ in paragraph 1 can best be replaced by

A. light bulb B. energy C. molecule D. atom

29. Which of the following statements best describes a laser?

A. A device for stimulating atoms and molecules to emit light

B. An atom in a high-energy state

C. A technique for destroying atoms or molecules

D. An instrument for measuring light waves

30. Why was Townes’ early work with stimulated emission done with microwaves?

A. He was not concerned with light amplification.

B. It was easier to work with longer wavelengths.

C. His partner Schawlow had already begun work on the laser.


D. The laser had already been developed.

31. In his research at Columbia University, Charles Townes worked with all of the following
EXCEPT

A. stimulated emission B. microwaves

C. light amplification D. a maser

32. In approximately what year was the first maser built?

A. 1917 B. 1951 C.1953 D. 1957

33. The word ‘emerged’ in paragraph 4 is closest in meaning to

A. increased B. concluded C. succeeded D. appeared

34. The word ‘outlining’ in paragraph 4 is closest in meaning to

A. assigning B. studying C. checking D. summarizing

35. Why do people still argue about who deserves the credit for the concept of the laser?

A. The researchers’ notebooks were lost.

B. Several people were developing the idea at the same time.

C. No one claimed credit for the development until recently.

D. The work is still incomplete.

Exercise 3. Read the following passage and mark letter A, B, C, or D to indicate the correct
word or phrase to complete each of the numbered blanks from 36 to 45. (5 pts)

Very few of us would admit putting much trust in horoscopes and the fact that the
movements of astronomical bodies (36)____________ to earthly occurrences affecting peoples'
everyday lives.
We all know about the zodiac signs which reflect the position of the sun, the moon
and the planets at the moment of a man's birth and about the peculiar characteristics
(37)___________ to them by astrologers. We say we will take these phenomena with a pinch of
salt while we keep (38) ____________ our eyes over them in every tabloid we lay our hands on.
Most frequently, we expect horoscopes to predict the future, to (39)_____________ our
optimistic mood with a piece of comforting information or to (40)_____________ our ego by
confirming the superlative features that we tend to attribute to our zodiacs.

However, there's no scientific evidence to (41)_____________ the assumption that


human existence is so closely (42)_____________ with the parameters of the celestial bodies.
Our curiosity in horoscopes may, then, (43)____________ our sheer fascination with the
unexplained or the unpredictable as well as in the enticing insight into the future that the
horoscopes offer, thus establishing the sense of our (44)____________ an extreme power over
our own lives. An additional explanation is that humans tend to have a soft (45)_____________
for any form of flattery, which is the fact to which astrologers and the horoscope writers seem to
attach the greatest deal of weight.

36. A. rely B. correspond C. match D. compare

37. A. identified B. associated C. incorporated D. ascribed

38. A. running B. sending C. fixing D. putting

39. A. restore B. adjust C. upgrade D. reassure

40. A. boost B. escalate C. revitalize D. improve

41. A. conclude B. concede C. corroborate D. confound

42. A. fused B. adhered C. coalesced D. intertwined

43. A. stem B. crop C. rear D. dawn

44. A. disposing B. wielding C. effecting D. committing

45. A. pit B. dot C. spot D. nick


Exercise 4. You are going to read an article about returning to work after being away on
holiday. Six sentences have been removed from the article. Choose from the sentences A-G the
one which fits each gap (37-42). There is one extra sentence which you do not need to use.

Just because I’ve been on holiday doesn’t mean I have to be happy

My car. My tortoise. My friends. My bed. The list reads the same every time, but I still write it. I
write it on the last day of every holiday, to convince myself that going home isn’t so bad. Then I
feel utterly miserable. There are plenty of things I’m not great at – driving, maths, returning
library books on time – but the thing I’m worst at is coming back to work after a holiday.

It’s an extreme case of being selfishly miserable. To have had a lovely sunshine break and then
return to the office, where everyone has been working hard without restaurant lunches or
morning swims, with a face like thunder is terribly bad manners. 37 . Given the
number of names for it – ‘holiday hangover’, ‘back-to-work blues’, post-travel depression’ – it’s
a well-known condition.
In a recent survey conducted by a travel website, 82 per cent of the 1,254 people asked
experienced post-holiday misery. 38 . Probably just before they logged on to a job
vacancy website or started fantasising about retraining for work in the countryside.
Even if you manage to avoid end-of-holiday panic, and you feel refreshed, relaxed and ready to
face the world of work, you’re guaranteed to walk into stress, conflict and
injustice. 39 . Or the surprise departmental reorganisation that took place while you
were away. Still, it could be worse. Over three-quarters of people questioned said that their
holiday depression lasted for a month. 40 . Perhaps they should have saved their cash
and nor bothered going.
After years of practice, I’ve come up with a few things that help. A bit. The first is the list
mentioned above. 41 . Unlike some people I know, I can’t just roll off an
intercontinental flight and roll in to the office. The third is concentrating on getting through the
first day back or work without running away, making a grand plan for a new life or spending (too
much) time on my own tearfully looking at my holiday photos saying to myself: ‘I can’t believe
this is my life.’
I feel sorry for my poor colleagues having to look at my long face today, but at least by having
my break now I’m getting my bad mood in early. 42 . Then I can support them in their
hour (month?) of need. I might even lend them one of my pets.
A. By September, on the other hand, when the schools go back and the main summer-holiday
season is over, I’ll be back to normal.
B. The most content, with both their home and working life, appear to be those who stay at home
all summer.
C. For instance, that highly important task you left with a colleague that’s been ignored and later
caused your email inbox to turn toxic.
D. At least, though, I’m not the only fed-up wage slave to feel like this.
E. The next one is making sure I have a day off everything between getting home and going to
work.
F. Also, over two-thirds of them answered the next question, ‘Are you usually glad to be home
after a holiday abroad?’ with a – presumably unhappy sounding – ‘No’.
G. Longer by at least a fortnight, I’d guess, than the holiday they’d taken.

Exercise 5. You are going to read a newspaper article about an Olympic athlete. For
questions 43 – 52, choose from the sections (A – D). The people may be chosen more than once.
Which paragraph

43. gives an example of Jessica having good luck?


44. refers to the role of Jessica’s family in helping her achieve success?
45. suggests it is surprising that Jessica does not understand herself better?
46. mentions a previous sporting disappointment that Jessica had?
47. explains why Jessica is so popular with the local public?
48. explains why another athlete was surprised at Jessica’s performance?
49. mentions a painful childhood memory?
50. suggests that Jessica’s appearance can give a misleading impression?
51. says that Jessica’s relationship with someone can sometimes be difficult?
52. contrasts Jessica’s personality on and off the track?

Jessica Ennis: heptathlon Olympic champion

A
There have been many great Olympic athletes in recent years, but few have been taken to their
country’s heart quite as warmly as gold medal winner Jessica Ennis. Her quiet determination to
succeed, her good humour when faced by setbacks and the absolute joy she showed when finally
becoming Olympic champion have all contributed to this, as has the difficulty of the sport she
has chosen to compete in: the heptathlon. This involves turning in world-class performances in
seven track and field events over two days. At first sight, Jessica – at just 1.65 metres and 57
kilos – may seem an unlikely winner of such a physically demanding sport, but once the action
begins it soon becomes clear she has the speed, strength and endurance to beat anyone.
В
Jessica recognises that her normally easygoing nature is transformed into something much fiercer
when she has to compete. She knows that success only comes from being highly motivated and
totally focused on each event. In her autobiography Unbelievable, she talks of the way she was
picked on at school by bigger girls because of her background and lack of size, and how that has
made her determined to succeed, particularly against taller and stronger athletes. She also points
out that she is not from a particularly sporting family and that her sister ‘absolutely hates sport’,
but says she was introduced to athletics by her parents, who have continued to give her
encouragement and support throughout her career as an athlete. Her mother was born in the UK
and her father in Jamaica.
C
She gets on well with her family, as she does with her husband Andy, saying she dislikes conflict
and wherever possible avoids arguments with people. The only exception is her trainer Chell,
with whom she has a row ‘every day’. And although Jessica is a psychology graduate, she is
unable to explain how she acquired the tremendous self-discipline that has enabled her to keep
training to Olympic gold medal standard while so many others have given up along the way. Of
course, at that level nothing can be taken for granted, as she discovered when a sudden injury put
her out of the Beijing Games. She describes that as the lowest point in her career. Typically,
though, Jessica bounced back, and once fit again began training just as hard as ever.
D
By the time of the London Games in 2012 she was in the best shape of her life, and on this
occasion she was fortunate enough to remain free of injury. Some of the times she recorded in
the heptathlon were so fast that she would have achieved good positions in the finals of track
events such as the 200 metres. That brought to mind a race won two years earlier against the
world champion, who couldn’t believe she had lost to someone who trained for seven different
events. Since the London Olympics, Jessica has continued to take part in competitions, receiving
numerous awards including World Sportswoman of the Year. She has also featured on a special
postage stamp and has had a post box in her home city of Sheffield painted gold in her honour.

PART FOUR: WRITING

Exercise 1. Finish the second sentence in such a way that its meaning is similar to that of the
original one. (10 pts)

1. I’d prefer you to wear something more formal to work.

→ I’d rather __________________________________________.

2. You bought too much food, which was not necessary.

→ You needn’t _________________________________________.

3. Success in the academic field depends on your ability to amass qualifications.

→ The more _________________________________________.


4. Provided your handwriting is legible, the examiners will accept your paper.

→ So long as _________________________________________.

5. Most of the problems arose because there was no leadership on the committee.

→ It _________________________________________.

6. Driving at that speed is dangerous whether you are an experienced driver or not.

→ However _________________________________________.

7. She didn’t cry when the story ended in tragedy.

→ Not a tear _________________________________________.

8. “ There’s no point in writing it all out in longhand if you can use a typewriter, isn’t there?”

→ She _________________________________________..

9. It’s highly unlikely that the meeting will end before 7.00.

→ The chances _________________________________________.

10. He himself admits to a fear of spiders.

→ On his _________________________________________.

Exercise 2. Rewrite each of the sentences below in such a way that its meaning is similar to
that of the original one, using the word given in brackets. Do not change this word in any way.
(5 pts)

1. What has this experience taught you? (DRAWN)

→ What conclusions _____________________________?

2. I need a calculator to arrive at the total. (WORK)

→ I can’t _____________________________________________.

3. As a result of the bad weather, there may be delay to some international flights. (SUBJECT)

→ Due to the bad weather _______________________________________.


4. Students at the school are not allowed to go into the Rainbow Disco. (BOUNDS)

→ The Rainbow Disco _________________________________________

5. I suddenly realized the meaning of a “freebie”. (DAWNED)

→The meaning of _____________________________________

You might also like